Download as pdf or txt
Download as pdf or txt
You are on page 1of 71

XII.

Termination of Employment
1.

While petitioners were only probationary employees who do not


enjoy permanent status, they are still protected by security of
tenure. For a probie, this means they may only be terminated for a
valid and just cause or for failing to qualify as a regular employee in
accordance with the reasonable standards made known to him by the
employer at the time of engagement, and after being accorded due
process.

The petitioners were terminated prior to end of the probationary


period. This is valid. As probationary employees, they may be
terminated anytime if the employer decides he no longer needs the
employee or has failed to reach the standards required for the job.

Espina v Hon Court of Appeals GR No. 164582, 2007

Facts

Petitioners are employees of MY San. MY San sold its business to


Monde.

In the interest of industrial peace, an agreement was had between the


workers and MY San's management, namely that they will be prioritized
for rehiring in case Monde needs new employees.

116 were hired as probationary employees for six months. Only 74 were
regularized. Petitioners were the probationary employees who were not
regularized. (Note that not all failed to reach the standards under the
probationary employment. Some just stopped reporting for work, while
some just resigned. )

Issue

WON the business closure was justified? Yes.

Firing is included in management prerogatives.

Under Art. 283, MY San was justified in its closure of establishment


and reduction of personnel, not only because of serous business
losses, but even without business losses. Only difference is that
where there is no serious business losses, separation pay must be
given to the employees.

Employers have rights and privileges to assure their self


determination and independent and reasonable return of capital,
called management prerogative. This is broad in scope, and limited
only by the state to ensure that the exercise of such prerogatives
does not interfere with the law or protected rights.

Art. 283 requirements for valid cessation:

service of written notice to employees and to DOLE at least 1


month before intended date thereof.

Cessation must be bona fide in character.

Payment to the employees of termination pay amounting to at


least month pay for every year of service. Or one month pay,
whichever is higher.

Ultimate test of validity of closure or cessation of establishment is


that it must be bona fide in character. And the burden of proving
such fails upon the employer.

WON the termination of petitioners was lawful? Yes.

The case at bar falls under Art. 281 on probationary employment.

2.

Alhambra Industries v NLRC 238 SCRA 232 (1994)

ALHAMBRA INDUSTRIES, INC., Petitioner, v. NATIONAL LABOR


RELATIONS COMMISSION and DANILO RUPISAN, Respondents.
FACTS:

On 27 June 1987, petitioner Alhambra Industries, Inc. employed private


respondent Danilo C. Rupisan as salesman on a six-month probationary
basis.

ALHAMBRA conducted a surprise audit of the records of Rupisan. He


was then called to the Head Office where alleged violations of company
rules purportedly committed by him were brought to his attention.
Rupisan was placed under a one-month preventive suspension for serious
violations of company policies, rules and regulations, as reflected in the
results of the surprise audit.

Rupisan protested his suspension, reiterating vehement denial of the


charges against him and exposing the threat of termination at their
meeting. He alleges that the charges against him had become academic
when he was given a clearance of all his accountabilities.

On 6 February 1990, a day before the end of his suspension,


ALHAMBRA wrote Rupisan terminating his services.

Rupisan sued ALHAMBRA for illegal dismissal and unpaid wages or


commissions. The suit was later amended on 24 April 1990 to include
charges of illegal suspension and damages.

ISSUE/S: whether Rupisan was validly terminated or dismissed


HELD:
The Labor Arbiter found a valid ground for dismissal, taking into consideration
the controverting evidence of the parties, The failure of petitioner to give private
respondent the benefit of a hearing before he was dismissed constitutes an
infringement of his constitutional right to due process of law and equal protection
of the laws. The petitioner must nevertheless be held to account for failure to
extend to private respondent his right to an investigation before causing his
1

dismissal. The rule is explicit. The dismissal of an employee must be for just or
authorized cause and after due process (Section 1, Rule XIV, Implementing
Regulations of the Labor Code). Petitioner committed an infraction of the second
requirement. Thus, it must be imposed a sanction for its failure to give a formal
notice and conduct an investigation as required by law before dismissing
petitioner from the employment.
A termination without just cause entitles a worker to reinstatement
regardless of whether he was accorded due process. On the other hand,
termination of a worker for cause, even without procedural due process, does
not warrant reinstatement, but the employer incurs liability for damages.
To order reinstatement and compel the parties to start the procedure from
step one would be circuitous because almost invariably that same issue of
validity of the ground of dismissal would be brought back to the Labor
Arbiter for adjudication. An otherwise justly grounded termination without
procedural due process would only sanction payment of damages.
The decision to dismiss Rupisan did not state the reason for his termination, in
disregard of Sec. 6, Rule XIV, Book V, of the Omnibus Rules. But, having been
found guilty of serious misconduct, private respondent cannot demand
reinstatement nor separation pay. However, he is entitled to damages for
petitioners non-observance of procedural due process which is not only required
by statute but enshrined in the Constitution.
3.

Pantoja v SCA Hygiene Products Corp GR No 163554 (2010)

4.

Coca Cola Bottlers v Valentina Garcia, GR No. 159625 (2008)

COCA COLA BUENAVENTURA COCA-COLA BOTTLERS PHILIPPINES,


INC., petitioner, vs. VALENTINA GARCIA, respondent.
FACTS
On December 1, 1988, Coca-Cola Bottlers Philippines, Inc. (petitioner) hired
Valentina G. Garcia (respondent) as Quality Control Technician on probationary
status. She was assigned at petitioner's Tacloban plant. On June 1, 1989 she
became a regular employee.
In the middle of 1989, petitioner adopted some modernization programs which
resulted in increased efficiency and production. Likewise, the work load of their
employees was substantially reduced. Under the Collective Bargaining
Agreement (CBA) and Article 283 of the Labor Code, respondent, as the most
junior employee of the Department could be validly terminated. However, instead
of terminating respondent on ground of redundancy, petitioner decided to assign
her to its Iloilo plant.
Thus, sometime in April 1990, petitioner informed respondent that she would be
transferred to the Iloilo plant for being an excess or redundant employee in the
Tacloban plant. Respondent refused to be transferred. Through her Union, she
brought the matter to their grievance machinery. Meanwhile, petitioner pushed
through with respondent's transfer. On June 26, 1990, petitioner gave respondent
notice of her transfer to take effect on July 2, 1990. Yet, on said date, respondent
reported for work at the Tacloban plant. The security guard refused her entry.

Almost one year after she was refused entry, respondent filed a complaint for
illegal dismissal.
The Labor Arbiter (LA) rendered a Decision finding that respondent was illegally
dismissed.
NLRC reversed. It held that there was a valid transfer since the mobility clause
in petitioner's employment contract was valid; and because petitioner refused to
be transferred, she was considered to have abandoned her work.
CA rendered a Decision partially granting the petition. While the CA held that
abandonment of work was a just cause to effect respondent's dismissal, it found
that the dismissal was ineffectual since it did not comply with due process
requirements, as petitioner received only the notice of her dismissal on the
ground of abandonment, and she was not given the initial notice of her impending
dismissal or the chance to explain her side. It held petitioner liable for backwages
from the time respondent was dismissed up to the finality of the decision.
ISSUE: whether respondent was accorded procedural due process before her
separation from work.
HELD: NO.
In dismissing an employee, the employer has the burden of proving that the
dismissed worker has been served two notices: (1) the first to inform the employee
of the particular acts or omissions for which the employer seeks his dismissal, and
(2) the second to inform the employee of his employer's decision to terminate
him.17 The first notice must state that the employer seeks dismissal for the act or
omission charged against the employee; otherwise, the notice does not comply
with the rules.
In Maquiling v. Philippine Tuberculosis Society, Inc.,the Court held that the first
notice must inform outright the employee that an investigation will be conducted
on the charges specified in such notice which, if proven, will result in the
employee's dismissal. The Court explained the rationale for this rule, thus: This
notice will afford the employee an opportunity to avail all defenses and exhaust
all remedies to refute the allegations hurled against him for what is at stake is
his very life and limb his employment. Otherwise, the employee may just
disregard the notice as a warning without any disastrous consequence to be
anticipated. Absent such statement, the first notice falls short of the requirement
of due process. One's work is everything, thus, it is not too exacting to impose this
strict requirement on the part of the employer before the dismissal process be
validly effected. This is in consonance with the rule that all doubts in the
implementation and interpretation of the provisions of the Labor Code, including
its implementing rules and regulations, shall be resolved in favor of labor. Article
277 of the Labor Code explicitly provides: ART. 277. Miscellaneous provisions. x x
x (b) Subject to the constitutional right of workers to security of tenure and their
right to be protected against dismissal except for a just and authorized cause and
without prejudice to the requirement of notice under Article 283 of this Code, the
employer shall furnish the worker whose employment is sought to be terminated
a written notice containing a statement of the causes for termination and shall
afford the latter ample opportunity to be heard and to defend himself with the
assistance of his representative if he so desires in accordance with company rules
2

and regulations promulgated pursuant to guidelines set by the Department of


Labor and Employment. x x x
Section 2, Rule XXIII, Book V of the Omnibus Rules Implementing the Labor
Code provides:
Section 2. Standards of due process: requirements of notice. - In all cases of
termination of employment, the following standards of due process shall be
substantially observed:
I. For termination of employment based on just causes as defined in Article 282 of
the Code: (a) A written notice served on the employee specifying the ground or
grounds for termination, and giving to said employee reasonable opportunity
within which to explain his side; (b) A hearing or conference during which the
employee concerned, with the assistance of counsel if the employee so desires, is
given opportunity to respond to the charge, present his evidence or rebut the
evidence presented against him; and (c) A written notice of termination served on
the employee indicating that upon due consideration of all the circumstances,
grounds have been established to justify his termination. x x x
There is no dispute that in cases of abandonment of work, notice shall be served
at the worker's last known address. While petitioner presented the envelopes of
the alleged notices sent to respondent's last known address, the contents thereof
were not offered in evidence. Thus, the records are wanting of proof that
respondent was properly apprised of the charges against her and given an
opportunity to explain her side, as petitioner maintains. Evidently, it is clear that
respondent's dismissal was effected without the notice required by law. Thus,
petitioner failed to satisfy the two-notice requirement.

FACTS: Mansion is a single proprietorship engaged in printing self-adhesive


printables while respondent is petitioners driver, who they first engaged as a
kargador but was later on promoted as the companys sole driver. The nature of
the business requiring swift delivery to clients, petitioners monitored Bitaras
punctuality, wherein the former found out he was late 38 out of 66 times during
that period of observation; for that, petitioners gave him a Memorandum, to
which he promised not to do the same. After the memorandum, Bitara incurred
19 absences without notice to his employers, and Davis Cheng, the proprietor,
gave him (1) a memorandum asking him to explain his absences; and (2) a notice
of termination four days after Bitaras receipt of the memorandum, which the
latter refused to sign or acknowledge.
Bitara filed a complaint for ID against Mansion, saying he took a leave of absence
from March 17-23, 2000, a period which caused financial losses to petitioner.
LA; NLRC: Dismissed for lack of merit; affirmed LAs findings
CA: Reversed NLRC; held that the basis of termination was Bitaras absences and
such basis was insufficient; no procedural due process observed because Bitara
refused to sign the memoranda
ISSUE: Whether or not the CA correctly found that the NLRC committed GAD in
upholding petitioners termination and affirming the denial of his claim for nonpayment of holiday pay, SIL pay and damages (NO)
RESOLUTION: NLRC decision REINSTATED with MODIFICATION

MANSION PRINTING CENTER v. BITARA

Substantive Due Process; Complied with; The notices contained the grounds
for his dismissal, which were his habitual tardiness and his frequent absences,
despite previous memoranda apprising him of such. Despite his claims that his
wife called a certain Delia to inform management of his intent to file a leave to
attend to an urgent family matter. An officer of Mansion even went to his house
to require him to go to work, but his wife said her hubby had left without saying
where he was headed. No basis as to the fact that the company tolerates
offsetting undertime with overtime. The adjustment of work hours from 8:30-5:30
were not proof of respondents claim that the company was not sure of the time its
employees were supposed to go to work; it was proof that they had to adjust work
hours to solve respondents habitual tardiness.

J. Perez

A.

Gross negligence and habitual neglect; Defined; Gross is negligence is


want of care in the performance of ones duties. Habitual neglect is the repeated failure to perform ones duties for a period of time, depending upon
the circumstances. These are not overly technical terms, which, in the first
place, are expressly sanctioned by the LC. Despite the absence of a definition
of GN and HN in the company rule book, respondents omissions qualify as
such warranting his dismissal from service.

B.

Procedural due process; Complied with; CA held that the fact of Bitaras
refusal to sign was a violation of Bitaras right to due process, as such was
self-serving on the part of petitioner, that petitioner should exert effort to observe PDP. In Bughaw v. Treasure Island, the Court looked for:

The Serrano doctrine, which awarded full backwages to "ineffectual dismissal


cases" where an employee dismissed for cause was denied due process, was
applied by the CA. That doctrine has been abandoned by the Court's ruling in
Agabon, where the Court held that if the dismissal was for cause, the lack of
statutory due process should not nullify the dismissal, or render it illegal or
ineffectual; but the employer's violation of the employee's right to statutory due
process warrants the payment of indemnity in the form of nominal damages.
5.

Mansion Printing Center v Bltara Jr GR No. 168120 (2012)

G.R. No. 168120


TOPIC: Termination; Gross negligence is want of care in the performance of
ones duties and habitual neglect as repeated failure to perform ones duties for
a period of time, depending upon the circumstances.. These are not overly
technical terms, which, in the first place, are expressly sanctioned by the Labor
Code of the Philippines. Clearly, even in the absence of a written company rule
defining gross and habitual neglect of duties, respondents omissions qualify as
such warranting his dismissal from the service. We cannot simply tolerate
injustice to employers if only to protect the welfare of undeserving employees.

a.

Affidavit of service stating the reason for failure to serve notice upon
the recipient

b.

A notation to that effect, written on the notice itself

Davis Cheng had both.


C.

Monetary claims; Employer to show that employee intended and


used his service incentive leave pay; Employers have the burden to prove
their employees received these benefits under the law. It is incumbent upon
employers to present the necessary documents to prove such claim. Cheng
was unable to prove he paid the monetary equivalent or allowed Bitara his
SIL. His numerous absences do not offset his SIL.

6.

Golden Ace Builders & Azul v Talde GR No. 187200 (2010)

7.

Functional Inc. v Samel C. Granfll GR No. 176377 Nov 16, 2011

FUNCTIONAL INC. vs SAMUEL GRANFIL


Functional Inc. is a business engaged in the sale and rental of various business
equipments such as photocopying machines. It employed Samuel Granfil as its
key operator to operate the photocopying machine rented by National Bookstore
in SM Megamall. The conflict in this case is rooted in that Granfil pocketed the
payment of the Cavaldeja spouses for the photocopies of their fliers instead of
turning it over to the cashier. The NBS security guard filed an incident report
about this, which prompted FI's Office Supervisor to terminate him.
Granfil thus files this illegal dismissal complaint against FI and its President
(Bautista), Marketing Manager (Tenorio), Office Supervisor (Ballesteros), and
Area Supervisor (Dizon).
ISSUE: w/n Granfil was illegally dismissed
HELD: YES, Granfil was illegally dismissed.
The burden of proof is upon the employer to show that the employee's termination
from service is for a just and valid cause. In this case, FI pleads Granfil's
abandonment as a defense against his illegal dismissal (that after the incident he
was to be transferred back to the main branch but he misconstrued this as
punishment and thus refused to report for work), but fails to adequately prove
such.
Abandonment has 2 elements: (1) failure to report for work/absence without valid
or justifiable reason, and (2) a clear intention to sever the employer-employee
relationship, as shown by overt acts. FI was unable to prove such when:

8.

it merely presented as evidence the NBS branch's request for Granfil's


transfer and FI's acquiescence thereto
defendants (President, Marketing Manager etc) didn't even execute
sworn statements to refute the charge of illegal dismissal
Granfil filed the complaint of illegal dismissal, clearly showing his
intention to continue employer-employee relations.
Pepsi Cola Product Phil. Inc v EV Santos GR No. 165968 (2008)

PEPSI
COLA
PRODUCTS
EMMANUEL V. SANTOS

PHILIPPINES, vs.

FACTS:
Respondent Emmanuel V. Santos was employed by petitioner Pepsi Cola Products
Phils., Inc. sometime in July 1989. In March 1996, he was promoted as Acting
Regional Sales Manager at the Libis Sales Office.
On February 14, 1997, respondent received from petitioner Ernesto F. Gochuico a
memorandum charging him with violation of company rules and regulations and
Article 282 (a) of the Labor Code, as follows:
Group III FRAUD AND ACTS OF DISHONESTY
NO. 12 Falsifying company records or documents or
knowingly using falsified records or
documents.
NO. 8 Breach of trust and confidence.
NO. 4 Engaging in fictitious transactions, fake
invoicing, deals padding and other sales
malpractices.
NO. 5 Misappropriation or embezzlement of company
funds or property and other acts of
dishonesty.
Article 282 (a) Serious misconduct or willful disobedience to
the
lawful orders of his employer.
The charges arose out of alleged artificial sales by the sales personnel of the Libis
Sales Office in March 1996 allegedly upon the instruction of respondent. The
alleged artificial sales resulted in damage to petitioners amounting to
P795,454.54.
Respondent was dismissed on June 27, 1997.
ISSUE: Whether the respondent was validly dismissed
HELD: NO. The respondent was not validly dismissed.
RATIO: Petitioners failed to present evidence to justify respondent's dismissal.
Save for the notice of termination, we could not find any evidence which would
clearly and convincingly show that respondent was guilty of the charges imputed
against him. There appears to be no compelling reason why petitioners would
rather present their witnesses on direct testimony rather than reduce their
testimonies into affidavits. The submission of these affidavits appears to be the
more prudent course of action particularly when the Labor Arbiter informed the
parties that no further trial will be conducted in the case.
In an illegal dismissal case, the onus probandi rests on the employer to prove that
its dismissal of an employee is for a valid cause.
ISSUE: (2) whether a trial on the merits was necessarry
4

HELD: NO. NOT NECESSARY


RATIO: It is not legally objectionable, for being violative of due process, for the
Labor Arbiter to resolve a case based solely on the position papers, affidavits or
documentary evidence submitted by the parties. The holding of a formal hearing
or trial is discretionary with the Labor Arbiter and is something that the parties
cannot demand as a matter of right. The requirements of due process are satisfied
when the parties are given the opportunity to submit position papers wherein
they are supposed to attach all the documents that would prove their claim in
case it be decided that no hearing should be conducted or was necessary.
9.
Jackqui Moreno v San Sebastian College-Recoletos GR No.
175283 (2008)
JACKQUI R. MORENO v. SAN SEBASTIAN COLLEGE-RECOLETOS,
MANILA (2008)
CHICO-NAZARIO, J.:
FACTS: Respondent SSC-R is a domestic corporation and an educational
institution duly registered under the laws of the Philippines, located in C. M.
Recto Avenue, Quiapo, Manila. SSC-R employed petitioner Jackqui R. Moreno
(Moreno) as a teaching fellow. Moreno was appointed as a full-time college faculty
member. Then, Moreno became a member of the permanent college faculty. She
was also offered the chairmanship of the Business Finance and Accountancy
Department of her college.
Subsequently, reports and rumors of Morenos unauthorized external teaching
engagements allegedly circulated and reached SSC-R. The Human Resource
Department of the school thereafter conducted a formal investigation on the said
activities. The Department submitted its report which stated that Moreno indeed
had unauthorized teaching assignments at the Centro Escolar University during
the first semester of the School Year 2002-2003, and at the College of the Holy
Spirit, Manila, during the School Years 2000-2001, 2001-2002 and the first
semester of School Year 2002-2003.
On 27 October 2002, Moreno received a memorandum from the Dean of her
college, requiring her to explain the reports regarding her unauthorized teaching
engagements. The said activities allegedly violated Section 2.2 of Article II of
SSC-Rs Faculty Manual which reads:
Administrative permission is required for all full-time faculty members to teach
part-time elsewhere. If ever teaching permission is granted, the total teaching load
should not exceed the maximum allowed by CHED rules and regulations. Faculty
members are required to report all other teaching assignments elsewhere within
two (2) weeks from start of the classes every semester.
Moreno sent a written explanation in which she admitted her failure to secure
any written permission before she taught in other schools. Moreno explained that
the said teaching engagements were merely transitory in nature as the aforesaid
schools urgently needed lecturers and that she was no longer connected with
them. Moreno further stated that it was never her intention to jeopardize her

work in SSC-R and that she merely wanted to improve her familys poor financial
conditions.
A Special Grievance Committee was then formed in order to investigate and
make recommendations regarding Morenos case. Moreno admitted she did not
formally disclose her teaching loads at the College of the Holy Spirit and at the
Centro Escolar University for fear that the priest administrators may no longer
grant her permission, as prior similar requests had already been declined; that
the Dean of her college was aware of her external teaching loads; that she went
beyond the maximum limit for an outside load in the School Years 2000 until
2002, because she needed to support her mother and sister, her masteral studies,
and her sisters canteen business, all of which coincided with the payment of the
emergency loan from the SSC-R administrators that paid for her mothers illness;
that she did not deny teaching part-time in the aforementioned schools; and that
she did not wish to resign because she felt she deserved a second chance.
On the same day that Moreno sent her letter, the grievance committee issued its
resolution, which unanimously found that she violated the prohibition against a
full-time faculty having an unauthorized external teaching load. SSC-R sent a
letter to Moreno that was signed by the College President, informing her that
they had approved and adopted the findings and recommendations of the
grievance committee and, in accordance therewith, her employment was to be
terminated. Moreno thus instituted with the NLRC a complaint for illegal
termination against SSC-R seeking reinstatement, money claims, backwages,
separation pay if reinstatement is not viable, and attorneys fees.
The LA dismissed Morenos complaint for lack of merit and ruled that
Morenos due acceptance of the appointment as a member of the Permanent
Faculty meant that she was bound to the condition therein not to accept any
outside teaching assignments without permission. The NLRC reversed the
rulings of the Labor Arbiter in a decision saying that the four applications for
leave of absence adduced in evidence by the respondent are all undated. If the
absences indicated in the said documents were the only absences incurred by the
Moreno in her four-year tenure, it cannot be said that she had a poor attendance.
The CA reinstated the decision of the LA saying that there was willful
disobedience of the employers lawful orders, a just cause for dismissal of an
employee.
ISSUE: WON THE DISMISSAL OF MORENO WAS PROPER AND LAWFUL.
HELD: IT WAS NOT PROPER. SSC-R failed to observe substantive due process.
RATIO: On the basis of the evidence on record, the Court finds that Moreno has
indeed committed misconduct against respondent SSC-R. Her admitted failure to
obtain the required permission from the school before she engaged in external
teaching engagements is a clear transgression of SSC-Rs policy. However, said
misconduct falls below the required level of gravity that would warrant dismissal
as a penalty.
Under Art. 282(a) of the Labor Code, willful disobedience of the employers lawful
orders as a just cause for termination of employment envisages the concurrence of
at least two requisites: (1) the employees assailed conduct must have been willful
or intentional, the willfulness being characterized by a "wrongful and perverse
5

attitude"; and (2) the order violated must have been reasonable, lawful, made
known to the employee and must pertain to the duties which he has been engaged
to discharge.
In order to constitute serious misconduct which will warrant the dismissal of an
employee under paragraph (a) of Article 282 of the Labor Code, it is not sufficient
that the act or conduct complained of has violated some established rules or
policies. It is equally important and required that the act or conduct must have
been performed with wrongful intent.
After examining the records of the case, the Court finds that SSC-R miserably
failed to prove that Morenos misconduct was induced by a perverse and wrongful
intent as required in Art. 282(a) of the Labor Code. SSC-R merely anchored
Morenos alleged bad faith on the fact that she had full knowledge of the policy
that was violated and that it was relatively easy for her to secure the required
permission before she taught in other schools. SSC-R failed to adduce any
concrete evidence to prove that Moreno indeed harbored perverse or corrupt
motivations in violating the aforesaid school policy. In her letter of explanation to
the grievance committee, Moreno explained in detail her role as the breadwinner
and the grave financial conditions of her family. As previous requests for
permission had already been denied, Moreno was thus prompted to engage in
illicit teaching activities in other schools, as she desperately needed them to
augment her income. Instead of submitting controverting evidence, SSC-R simply
dismissed the above statements as nothing more than a lame excuse and are
clearly an afterthought, considering that no evidence was offered to support
them and that Morenos salary was allegedly one of the highest among the
universities in the country. Finally, the Court noted that in Morenos contract of
employment, one of the provisions therein categorically stated that should a
violation of any of the terms and conditions thereof be committed, the penalty
that will be imposed would either be suspension or dismissal from employment.
Thus, contrary to its position from the beginning, SSC-R clearly had the
discretion to impose a lighter penalty of suspension and was not at all compelled
to dismiss Moreno under the circumstances, just because the Faculty Manual said
so.
In light of the foregoing, the Court holds that the dismissal of petitioner Moreno
failed to comply with the substantive aspect of due process. Despite SSC-Rs
observance of procedural due process, it nonetheless failed to discharge its burden
of proving the legality of Morenos termination from employment. Thus, the
imposed penalty of dismissal is hereby declared as invalid.
10.
Nagkakaisang Lakas ng Manggagawa sa Keihin v Keihin Phil
Corp GR No. 171115
NAGKAKAISANG LAKAS NG MANGGAGAWA SA KEIHIN (NLMK-OLALIAKMU) and HELEN VALENZUELA, v. KEIHIN PHILIPPINES CORPORATION,
Aug 9, 2010
Facts:

Helen Valenzuela (Helen) was a production associate in respondent Keihin


Philippines Corporation (Keihin), a company engaged in the production of intake
manifold and throttle body used in motor vehicles manufactured by
Honda.
It is a standard operating procedure of Keihin to subject all its employees to
reasonable search before they leave the company premises. On September
5, 2003, while Helen was about to leave the company premises, she saw a
packing tape near her work area and placed it inside her bag because it
would be useful in her transfer of residence. When the lady guard on duty
inspected Helens bag, she found the packing tape inside her bag. The guard
confiscated it and submitted an incident report to the HR Dept.
The following day the company issued a show cause notice to Helen accusing her
of violating the companys Code of Conduct, which says, Any act constituting
theft or robbery, or any attempt to commit theft or robbery, of any
company property or other associates property. Penalty: D (dismissal).
Helens supervisor, called her to his office and directed her to explain in writing
why no disciplinary action should be taken against her.
Helen admitted the offense. She, however, did not reckon that respondent
company would terminate her services for her admitted offense.
On September 26, 2003, Helen received a notice of termination.
On October 15, 2003, petitioners filed a complaint for illegal dismissal.
Petitioners alleged that Helens act of taking the packing tape did not
constitute serious misconduct, because the same was done with no
malicious intent. They believed that the tape was not of great value and of
no further use to respondent company since it was already half used.
Although Helen admitted that she took the packing tape, petitioners claimed that
her punishment was disproportionate to her infraction.
LA dismissed the complaint. LA held that she indeed committed a serious
violation of the companys policies amounting to serious misconduct, a just cause
for terminating an employee under Article 282 of the Labor Code.
NLRC affirmed in toto the decision of the LA.
CA dismissed the petition outright for not having been filed by an indispensable
party in interest under Section 2, Rule 3 of the Rules of Court.
6

Issue: Whether, in taking the packing tape for her own personal use, Helen
committed serious misconduct, which is a just cause for her dismissal from
service. YES. Helen is guilty of serious misconduct in her act of taking the
packing tape.
Ruling:
Article 282 of the Labor Code enumerates the just causes for termination. It
provides:
ARTICLE 282. Termination by employer. An employer may terminate an
employment for any of the following causes:
(a) Serious misconduct or willful disobedience by the employee of the lawful
orders of his employer or representative in connection with his work;
(b) Gross and habitual neglect by the employee of his duties;

employee had no violations in his eight years of service and the value of the
lighter fluid x x x is very minimal compared to his salary x x x.
After a closer study of both cases, we are convinced that the case of Caltex is
different from the case at hand. Although both Clarete and Helen had no prior
violations, the former had a clean record of eight years with his employer. On the
other hand, Helen was not even on her second year of service with Keihin
when the incident of theft occurred. And what further distinguishes the
instant case from Caltex is that respondent company was dealing with several
cases of theft, vandalism, and loss of company and employees property
when the incident involving Helen transpired.
Re: procedural due process
In the dismissal of employees, it has been consistently held that the twin
requirements of notice and hearing are essential elements of due process. The
employer must furnish the employee with two written notices before termination
of employment can be legally effected:
(a) a notice apprising the employee of the particular acts or omissions for
which his dismissal is sought, and

(c) Fraud or willful breach by the employee of the trust reposed in him by his
employer or duly authorized representative;
(d) Commission of a crime or offense by the employee against the person of his
employer or any immediate member of his family or his duly authorized
representative; and
(e) Other causes analogous to the foregoing.
Misconduct is defined as the transgression of some established and
definite rule of action, a forbidden act, a dereliction of duty, willful in
character, and implies wrongful intent and not mere error in judgment.
For serious misconduct to justify dismissal under the law,
(a) it must be serious,

(b) a subsequent notice informing the employee of the employers decision


to dismiss him.
With regard to the requirement of a hearing, the essence of due process lies in an
opportunity to be heard. Such opportunity was afforded the petitioner when she
was asked to explain her side of the story. In Metropolitan Bank and Trust
Company v. Barrientos, we held that, the essence of due process lies simply in an
opportunity to be heard, and not that an actual hearing should always and
indispensably be held. Similarly in Philippine Pasay Chung Hua Academy v.
Edpan,[44] we held that, [e]ven if no hearing or conference was conducted, the
requirement of due process had been met since he was accorded a chance to
explain his side of the controversy.

(b) must relate to the performance of the employees duties; and

Petition Denied.

(c) must show that the employee has become unfit to continue working
for the employer.

11.
2013

Helen, by her own admission, there was intent on her part to benefit herself when
she attempted to bring home the packing tape in question.
Firestone Tire and Rubber Company of the Philippines v. Lariosa: theft
committed by an employee constitutes a valid reason for his dismissal.
The petitioners also argue that the penalty of dismissal is too harsh and
disproportionate to the offense committed since the value of the thing taken is
very minimal. Petitioners cite the case of Caltex Refinery Employees Association v.
NLRC where Arnelio M. Clarete (Clarete) was found to have willfully breached
the trust and confidence reposed in him by taking a bottle of lighter fluid. In
said case, we refrained from imposing the supreme penalty of dismissal since the

SKM Art Craft Corp. v. Efren Bauca et al., G.R. No. 171282 Nov 27,

SKM Art Craft Corp. v. Efren Bauca et al. (2013).


Private respondents are all employees of SKM Art Craft Corp. On April 8, 2000 a
fire burned down the packing area of SKM including container vans and trailer
truck. On May 8, 2000, SKM informed respondents that it would be suspending
operations for 6 months. Employees immediately filed complaint for illegal
dismissal. They claimed that SKM was selective of the employees that were
suspended, all of whom were from a newly formed union.
DEFENSE: LC 286 provides for bona fide suspension of a business or
undertaking for a period not exceeding six months. Fire caused significant losses
from which they could not recover quickly. Also, the complaint was premature
because they had not yet been terminated.
7

Intervening fact: They were not reinstated during pendency of the case through
LA, NLRC, CA, and SC.

NLRC: affirmed the Labor Arbiter but reduced moral damages and deleted
attorneys fees.

SC: In general, case filed before lapse of 6 months in bona fide suspension is
premature. However, in this case, they were not reinstated after the 6 month
period, which becomes constructive dismissal. Even if the company offered to
reinstate them one year after as was the case here, there was already a violation
of the law when they failed to reinstate employees within six months of the
suspension.

CA: Set aside the NLRC decision for lack of jurisdiction. The case being an intracorporate dispute and under the SECs jurisdiction.
Issues:

12.

The NLRC had jurisdiction.

Gilles v CA Schema Konsult GR No. 149273 (2009)

Gilles vs CA, Schema Konsult


Nachura, J.
Facts:

Respondent Schema Konsult, Inc. (SKI) is a company engaged in project


consulting and management. Respondent Edgardo Abores was the President of SKI.
Petitioner Bienvenido Gilles was an incorporator, stockholder, a VP for
Finance and Principal Engineer.
SKI entered into an agreement with Carl Bro International (CBI) where
SKI will provide staff to CBIs projects.
CBI was awarded a shrimp farm project in India which needed a qualified Aquaculture Engineer. Gilles was accepted as the Engineer for the
project for a period of 2 years (Jan 1993-Jan 1995). He was to remain an
employee of SKI during that period and his salary was to come from SKI.
Gilles was sent to India for the project however in May 1993, Gilles tendered his resignation to CBI and departed for the Philippines the day after he tendered. CBI faxed the resignation letter to SKI.
In the Philippines, a board meeting was held to discuss Gilles resignation. The Board decided to terminate Gilles services and a notice was
sent to him.
Gilles filed a complaint for illegal dismissal.

Gilles version: There was a deliberate scheme to ease him out of the Project and
ultimately out of SKI. He believed that Pres. Abores was behind it. He tried to
communicate the difficulties he encountered in the India project but his calls were
ignored. Regarding his difficulties, he alleged that the project lead in India
committed tight timelines which necessitated that he work 18-hour days, 7 days a
week. Furthermore, he alleged that he was not paid his salary for 3.5 months.
Gilles said that he had an 80-year old mother to support and bills to pay. These
pushed him to resign and depart for the Philippines despite having no
replacement yet.
SKIs version: The allegations of Gilles are mere fabriations. SKI averred that
Gilles was well provided in India; that his resignation from CBI and his
departure from India were not known nor approved by SKI.
Labor Arbiter: found for Gilles and ordered his reinstatement and awarded
moral damages and attorneys fees.

1.) W/N the NLRC had jurisdiction. Yes


2.) W/N Gilles was illegally dismissed. Yes
Article 217 of the Labor Code vests in Labor Arbiters and the NLRC exclusive
jurisdiction to hear and decide cases involving termination disputes and all other
cases arising from employer-employee relations.
The case is not an intra-corporate dispute but a labor controversy. Gilles sought
reinstatement; he wanted to recover his position as Principal Engineer of SKI. It
was the Labor Arbiter that made the mistake of ordering the reinstatement of
Gilles as VP for Finance. Gilles never sought to regain his seat in the Board of
Directors; he actually claimed reinstatement as Principal Engineer of SKI.
Gilles was illegally dismissed (constructive dismissal).
A valid termination of employment by the employer must comply with two
requisites, namely:
(1) the dismissal must be for any of the causes provided under Article 282 of
the Labor Code which must be supported by clear and convincing evidence.
(2) the employee must be afforded an opportunity to be heard and to defend
himself (notice and hearing).
SKI maintains that Gilles was terminated for willful disobedience and gross
neglect of his duties.
Willful disobedience has two requisites which are present in this case:
1)
2)

conduct is willful (wrongful and perverse attitude)


The order violated was reasonable, lawful, made known to the employee
and must pertain to his duties.

Gilles resignation from CBI and sudden departure from India was not approved
by SKI. management instructed him to stay in India. When he left the Project,
despite the clear and lawful instructions of the management for him to stay, his
act constituted willful disobedience.
However SKI was guilty of violating Art 103 of the Labor Code. SKI was remiss in
paying the compensation of Gilles as Aquaculture Engineer of the Project on time.
Gilles was not paid his salaries for the three and half (3) months. Gilles
departure from India, despite the instruction of SKI for him to stay, was impelled
by the financial difficulties he encountered thereat. Clearly, Gilles had a valid
reason to leave India. This is a case of constructive dismissal. Constructive
dismissal exists when the employee involuntarily resigns due to the harsh,
hostile, and unfavorable conditions set by the employer.
8

In regards to SKIs allegation of gross neglect, the court held that as a just cause
for an employees dismissal, neglect of duty must not only be gross but also
habitual. A single or isolated act of negligence does not constitute a just cause for
the dismissal of the employee.
SKI was ordered to pay Gilles separation pay. President Abores was absolved for
insufficient proof of bad faith as director.
13.

Yabut v MECO GR No. 190436 (2012)

backwages and attorney's fees. NLRC rendered its Resolution dismissing the
herein respondents' appeal for lack of merit. CA rendered the now assailed
Decision reversing the rulings of the NLRC.
Issue
Whether or not the CA committed an error of law in annulling and setting aside
the resolutions of the NLRC that declared the herein petitioner illegally
dismissed by the respondents.

NORMAN YABUT VS. MANILA ELECTRIC COMPANY

Ruling

AND MANUEL M. LOPEZ

The petitioner's violation of the company rules was evident. While he


denies any involvement in the installation of the shunting wires which Meralco
discovered, it is significant that said SIN 708668501 is registered under his name,
and its meter base is situated within the premises of his property. Said meter
registered electric consumption during the time his electric service was officially
disconnected by Meralco. It was the petitioner and his family who could have
benefited from the illegal connection, being the residents of the area covered by
the service. His claim that he failed to know or even notice the shunted wires fails
to persuade as we consider the meter located in the front of his house, the nature
of his work as branch field representative, his long-time employment with
Meralco and his familiarity with illegal connections of this kind.

Facts
The petitioner had worked with Meralco from February 1989 until his dismissal
from employment on February 5, 2004. At the time of said dismissal, he was
assigned at the Meralco Malabon Branch Office as a Branch Field Representative
tasked, among other things, to conduct surveys on service applications, test
electric meters, investigate consumer-applicants' records of Violations of Contract
(VOC) and perform such other duties and functions as may be required by his
superior. The Inspection Office claimed discovering shunting wires installed on
the meter base for Service Identification Number (SIN) 708668501, registered
under petitioner Yabut's name. These wires allegedly allowed power transmission
to the petitioner's residence despite the fact that Meralco had earlier disconnected
his electrical service due to his failure to pay his electric bills. Meralco's Head of
Investigation-Litigation Office issued to the petitioner a notice4 dated November
3, 2003, received by the petitioner's wife on the same day.
Yabut admitted being the registered customer of Meralco at No. 17 Earth Street,
Meralco Village 8, Batia, Bocaue, Bulacan. The petitioner claimed that his
electrical service was disconnected sometime in July 2003 for unpaid electric bills.
He confirmed that the inspected meter base was installed within his lot's
premises. Claiming that he had been obtaining electricity from a neighbor, he
argued that shunting wires in his meter base could have caused an electrical
malfunction. As to Meralco's allegation that Yabut's wife had admitted the
petitioner's authorship of the illegal connection, Yabut denied knowing of such
admission.
Yabut wife admitted that he were the one who installed the shunted wires on
your meter base to have power because she and his two children were sick. The
illegal connection enabled you to defraud the company by consuming unregistered
electricity which makes him liable for violation of Section 7, par. 3 of the
Company Code on Employee Discipline, defined as (d)irectly or indirectly
tampering with electric meters or metering installations of the Company or the
installation of any device, with the purpose of defrauding the Company,
penalized therein with dismissal from the service. Yabut filed with the National
Labor Relations Commission (NLRC) a complaint11 for illegal dismissal and
money claims against Meralco and Lopez.
Labor Arbiter Antonio R. Macam rendered his Decision, declaring the petitioner
illegally dismissed from the service and hence, entitled to reinstatement plus

It is reasonable that its commission is classified as a severe act of dishonesty,


punishable by dismissal even on its first commission, given the nature and
gravity of the offense and the fact that it is a grave wrong directed against their
employer.
To reiterate, Article 282 (a) provides that an employer may terminate an
employment because of an employee's serious misconduct, a cause that was
present in this case in view of the petitioner's violation of his employer's code of
conduct. Misconduct is defined as the transgression of some established and
definite rule of action, a forbidden act, a dereliction of duty, willful in character,
and implies wrongful intent and not mere error in judgment. For serious
misconduct to justify dismissal, the following requisites must be present: (a) it
must be serious; (b) it must relate to the performance of the employee's duties;
and (c) it must show that the employee has become unfit to continue working for
the employer. To consider the petitioner's duties and powers as a Meralco
employee. And we conclude that he committed a serious misconduct. Installation
of shunting wires is without doubt a serious wrong as it demonstrates an act that
is willful or deliberate, pursued solely to wrongfully obtain electric power through
unlawful means. The act clearly relates to the petitioner's performance of his
duties given his position as branch field representative who is equipped with
knowledge on meter operations, and who has the duty to test electric meters and
handle customers' violations of contract. Instead of protecting the companys
interest, the petitioner himself used his knowledge to illegally obtain electric
power from Meralco. His involvement in this incident deems him no longer fit to
continue performing his functions for respondent-company.
Loss of confidence as a just cause for termination of employment is premised from
the fact that an employee concerned holds a position of trust and confidence. This
situation holds where a person is entrusted with confidence on delicate matters,
9

such as the custody, handling, or care and protection of the employer's property.
But, in order to constitute a just cause for dismissal, the act complained of must
be work-related such as would show the employee concerned to be unfit to
continue working for the employer. Taking into account the results of its
investigations, Meralco cannot be expected to trust Yabut to properly perform his
functions and to meet the demands of his job. His dishonesty, involvement in
theft and tampering of electric meters clearly prejudice respondent Meralco, since
he failed to perform the duties which he was expected to perform.

process requirements mandated by law. Hence, employers are barred from


arbitrarily removing their workers whenever and however they want. The law
sets the valid grounds for termination as well as the proper procedure to take
when terminating the services of an employee.

14.
Jonas Michael R. Garza v. Coca-Cola Bottlers Phils., Inc., et al.,
G.R. No. 180972 (2014)

The Labor Code, as amended, enumerates several just and authorized causes for
a valid termination of employment. An employee asserting his right and asking
for minimum wage is not among those causes. Dismissing an employee on this
ground amounts to retaliation by management for an employees legitimate
grievance without due process. Such stroke of retribution has no place in
Philippine Labor Laws.

15.
Alert Security & investigation Agency Inc. v Saidali Pasawilan et
al GR No. 182397 (2011)
Alert Security v. Pasawilan
Facts:
Respondents Saidali Pasawilan, Wilfredo Verceles and Melchor Bulusan were all
employed by petitioner Alert Security and Investigation Agency, Inc. (Alert
Security) as security guards beginning March 31, 1996, January 14, 1997, and
January 24, 1997, respectively. They were paid 165.00 pesos a day as regular
employees, and assigned at the Department of Science and Technology (DOST)
pursuant to a security service contract between the DOST and Alert Security.
Respondents aver that because they were underpaid, they filed a complaint for
money claims against Alert Security and its president and general manager,
petitioner Manuel D. Dasig, before Labor Arbiter Ariel C. Santos. As a result of
their complaint, they were relieved from their posts in the DOST and were not
given new assignments despite the lapse of six months. On January 26, 1999,
they filed a joint complaint for illegal dismissal against petitioners.
Petitioners, on the other hand, deny that they dismissed the respondents.
Petitioners presented "Duty Detail Orders" that Alert Security issued to show
that respondents were in fact assigned to LRTA. Respondents, however, failed to
report at the LRTA and instead kept loitering at the DOST and tried to convince
other security guards to file complaints against Alert Security. Thus, on August 3,
1998, Alert Security filed a "termination report" with the Department of Labor
and Employment relative to the termination of the respondents.
Issue:
Whether respondents were illegally dismissed
Held:
SC rules in the affirmative.
As a rule, employment cannot be terminated by an employer without any just or
authorized cause. No less than the 1987 Constitution in Section 3, Article 13
guarantees security of tenure for workers and because of this, an employee may
only be terminated for just or authorized causes that must comply with the due

Although the Court recognizes the right of employers to shape their own work
force, this management prerogative must not curtail the basic right of employees
to security of tenure. There must be a valid and lawful reason for terminating the
employment of a worker. Otherwise, it is illegal and would be dealt with by the
courts accordingly.

On the element of the failure of the employee to report for work, the Court also
cannot accept the allegations of petitioners that respondents unjustifiably refused
to report for duty in their new posts. A careful review of the records reveals that
there is no showing that respondents were notified of their new assignments.
Granting that the "Duty Detail Orders" were indeed issued, they served no
purpose unless the intended recipients of the orders are informed of such.
The employer cannot simply conclude that an employee is ipso facto notified of a
transfer when there is no evidence to indicate that the employee had knowledge
of the transfer order. Hence, the failure of an employee to report for work at the
new location cannot be taken against him as an element of abandonment.
The Court acknowledges and recognizes the right of an employer to transfer
employees in the interest of the service. This exercise is a management
prerogative which is a lawful right of an employer. However, like all rights, there
are limitations to the right to transfer employees. As ruled in the case of Blue
Dairy Corporation v. NLRC:
x x x The managerial prerogative to transfer personnel must be exercised without
grave abuse of discretion, bearing in mind the basic elements of justice and fair
play. Having the right should not be confused with the manner in which that right
is exercised. Thus, it cannot be used as a subterfuge by the employer to rid himself
of an undesirable worker. In particular, the employer must be able to show that the
transfer is not unreasonable, inconvenient or prejudicial to the employee; nor does
it involve a demotion in rank or a diminution of his salaries, privileges and other
benefits. x x x
In addition to these tests for a valid transfer, there should be proper and effective
notice to the employee concerned. It is the employers burden to show that the
employee was duly notified of the transfer. Verily, an employer cannot reasonably
expect an employee to report for work in a new location without first informing
said employee of the transfer. Petitioners insistence on the sufficiency of mere
issuance of the transfer order is indicative of bad faith on their part.

10

*Digest culled from Scribd. Cross checked with case, facts and doctrine are sound.
Credit to the owner.
16.

Mapili v Phil Rabbit Bus Lines GR No. 172506 (2011)

Mapili vs Phil. Rabbit


Del Castillo, J.
Facts:

Jerry Mapili was hired by Philippine Rabbit as a bus conductor. While on


duty in a Manila-Pangasinan trip, he was caught by Phil. Rabbits field
inspector extending a free ride to a lady, who was a wife of a coemployee. On order from the inspector, a ticket was issued and was paid
for by the lady.
Mapili was preventively suspended. Thereafter, a hearing was conducted
and he was afforded the chance to explain himself. He was terminated
for committing a serious irregularity by extending a free ride to a
passenger in violation of company rules. This was the 3rd time that
Mapili committed such violation.
He filed a complaint for illegal dismissal with the NLRC.
Mapilis argument: termination without cause and due process.
Infraction was trivial and done without malice. He honestly believed that
family members of Phil. Rabbits employees are entitled to free ride. His
two previous infractions cannot be considered because they were trivial:
1) giving free ride to a police officer, 2) giving free ride to a an exemployee who misrepresented himself to be currently working for Phil.
Rabbit by presenting a company ID. He further argued that he was
already penalized for previous violations and to consider them anew
would be tantamount to penalizing him twice for the same offense.
Considering his length of service (8 years), petitioner advanced that his
violations are not sufficient to merit the penalty of dismissal.
Phil. Rabbits argument: Mapilis admission that he gave a free ride and
that this was the 3rd offense justified his termination considering that his
position is imbued with trust and confidence. The present violation
coupled with his past record of serious offenses was a ground for valid
dismissal. They also argued that due process was observed.

Issue: Whether or not dismissal was valid. Yes.


Petitioners violation of company rules was intentional, willful, serious and a just
cause for dismissal.
In the CBA between employees and Phil. Rabbit, there is a provision for free rides
but this is only for employees. For relatives, passes must first be requested.
Mapili should be conversant of this provision considering previous infractions of
the same nature. Mapilis claim of good faith is belied by his admission that he
knew the free ride was a violation yet extended it to the lady nonetheless because
she helped him financially in the past. There was deliberate intent to commit the
violation in order to repay a personal debt at the expense of the company.
His infraction was not merely trivial. As a conductor, his duty is the collection of
fares which is the lifeblood of Phil. Rabbit. and his habitual failure to exercise the
same cannot be taken for granted. Mapilis series of irregularities when put
together may constitute serious misconduct.
This is the third time that Mapili refused to collect fares from passengers.
Although Mapili already suffered the corresponding penalties for his past
misconduct, those infractions are still relevant and may be considered in
assessing his liability for his present infraction. Mapilis termination is proper.
17.

Aboc v Metro Bank & Trust Co GR No. 170542-43 (2010)

Aboc v Metro Bank


FACTS:
o

Aboc, started working as a loan officer.

He was given merit increases and awarded promotions during his


employment because of his highly satisfactory performance

For 9 years, he maintained an unblemished employment record

On January 1998, he received a letter requiring him to explain in writing


the charges that he had actively participated in the lending activities of
his immediate supervisor, Wynster Y. Chua (Chua)

Aboc wrote back and explained that he has no interest

LA: Found for Mapili. No intention to defraud the company by his failure to issue
a ticket. The ride was given out of gratitude for the lady and under impression
that relatives can be given free rides. The fare was subsequently collected
anyway. Due process was observed.

He admitted, however, that he did some acts for Chua in connection with
his lending activity. He did so because he could not say no to Chua
because of the latters influence and ascendancy over him and because of
his utang na loob (debt of gratitude)

NLRC: Reversed the LA and found for Phil. Rabbit. Mapilis violation was a
deliberate act which prejudiced the companys interests. Petitioners record of
committing the violation and his propensity to commit similar infractions do not
merit the compassion of law.

His participation in the lending activity was limited to ministerial acts


such as the preparation of deposit and withdrawal slips and the typing of
statement of accounts for some clients of Chua.

Metrobank alleged that Chua, Judith Eva Cabrido (assistant


manager), Arthur Arcepi (accountant), and Aboc organized a
credit union known as Cebu North Road Investment (CNRI). Said
officers and employees used Metrobanks premises, equipment and
facilities in their lending business.

CA: Upheld the NLRCs ruling. Mapili has a history of committing violations of
company rules, the last one being a repeat violation against extending free rides
to passengers.

11

Thus, Aboc was terminated

ISSUE: WON Aboc was validly terminated?


HELD/RATIO: YES

Art. 282 LC. (a) Serious misconduct or willful disobedience by the


employee of the lawful orders of his employer or representative in
connection with his work; (c) Fraud or willful breach by the employee of
the trust reposed in him by his employer or duly authorized
representative;

the burden of proof rests on the employer to show that the dismissal was
for a just cause or authorized cause

An employee's dismissal due to serious misconduct and loss of trust and


confidence must be supported by substantial evidence.

Substantial evidence is that amount of relevant evidence as a reasonable


mind might accept as adequate to support a conclusion, even if other
minds, equally reasonable, might conceivably opine otherwise

Metrobanks evidence clearly shows that the acts of Aboc in helping Chua
organize the CNRI and FFA credit unions

Aboc submitted his Explanation[23] dated February 6, 1998, admitting


having committed said acts but claiming that he was only an unwilling
participant doing a ministerial job.

18.

Consequently, according to management, about 65% of Nissans employees


declined rendering overtime work, angered by the low salaries. Nissan suffered
massive losses.
First notice (charge) was received by Angelo on May 8, 2000 on grounds of serious
misconduct, willful disobedience, and gross neglect of duties. He was preventively
suspended and given 3 days to respond. Hearing was scheduled. Angelo
responded by letter-explanation, partly reading:
Again, its not negligence on my part and Im not alone to be blamed. Its
negligence on your part [Perla Go] and AA Del Rosario kasi, noong pang April
1999 ay alam niyo na hindi ako ang dapat may responsibilidad ng payroll kundi
ang Section Head eh bakit hindi ninyo pinahawak sa Section Head noon pa. Pati
kaming dalawa sa payroll, kasama ko si Thelma. Tinanggal nyo si Thelma. Hindi
nyo ba naisip na kailangan dalawa ang tao sa payroll para pag absent ang isa ay
may gagawa. Dapat noon nyo pa naisip iyan. Ang tagal kong gumawa ng
trabahong hindi ko naman dapat ginagawa.
Management concluded that Angelos explanation was untrue and insufficient.
Notice of termination (2nd notice) was issued on June 13, 2000.
ISSUES & RULING:
1)

Nissan Motors Phil. V Victorino Angelo GR No. 164 181 (2011)

NISSAN v ANGELO (2011)


FACTS:
Victorino Angelo was employed by Nissan Motor Philippines, Inc as one of its
payroll staff. On April 7 to 17, 2000, respondent was on sick leave. He would leave
the office early on certain days and would be absent without notice. He was not
able to prepare the payroll for April 15, 2000. IT tried to extract the payroll from
Angelos office computer but failed. Thus, payroll was released only on April 15,
which was a Saturday (in practice, Nissan does not release salary on Saturdays
but on Fridays in advance). Payroll was erratic because Nissan based it on the
March 15 payroll. Some employees got more than what they were supposed to
receive; others got less. Employees became angry.
Again, on April 27 to 28, 2000, Angelo was on vacation leave for his sons
graduation, approved upon condition that he will finish the payroll on time and
will make a proper turnover to his immediate superior before the leave. He again
was not able to prepare the payroll. He turned over the payroll task via diskette
but there were no bank account numbers, deductions were not finished, and
salaries of contractuals and apprentices were not finished. IT successfully
accessed the payroll, but salaries were again lower than what they were supposed
to be.

WON there was serious misconduct


YES. Misconduct is improper or wrong conduct. It is the transgression of
some established and definite rule of action, a forbidden act, a dereliction
of duty, willful in character, and implies wrongful intent and not mere
error in judgment. For misconduct to be a just cause:
a) It must be serious;
b) It must relate to the performance of employees duties; and
c) It must show that the employee has become unfit to continue
working for the employer.
The letter-explanation was grossly discourteous in content and tenor.
Accusatory and inflammatory language used by an employee to the
employer or superior can be a ground for dismissal.

2)

WON there was willful disobedience


YES. Willful disobedience of a rule, order, or instruction justifies
dismissal only when the rule, order, or instruction is:
a) Reasonable and lawful;
b) Sufficiently known to the employee; and
c) Connected with the duties which the employee has been engaged to
discharge.
Willful disobedience can be adduced and proven from the same letterexplanation.

3)

4)

WON there was gross neglect of duties


YES. Neglect of duty, to be a ground for dismissal, must be both gross
and habitual.
Failure to turn over the payroll task constitutes gross neglect.
WON Angelo is entitled to separation pay
12

YES. In general, separation pay is not awarded if termination is due to a


just cause. However, the Court, inspired by compassionate and social
justice, has in the past awarded financial assistance to dismissed
employees when circumstances warranted such an award.
Angelo is entitled to separation pay as financial assistance, considering
his length of service and poor physical condition which was one of the
reasons he filed a leave.

The case does not in any way show that they were even remotely negligent of
their duties so as to cause the loss of NBSs funds. They were able to illustrate
with candor and sincerity the procedure they took prior to the loss which was
witnessed by an employee of NBS. They were subjected to a thorough body search
before leaving their place of work on the date in issue. Moreover, it was not even
shown that they had access to the vault where the money was kept.

National Bookstore, Inc., and Alfredo C. Ramos vs. CA Special 8th Div.,
NLRC, Marietta M. Ymasa, and Edna L. Gabriel

Significantly, in order to constitute a just cause for the employees dismissal, the
neglect of duties must not only be gross but also habitual. Thus, the single or
isolated act of negligence does not constitute a just cause for the dismissal of the
employee. Verily, assuming arguendo that they were negligent, although we find
otherwise, it could only be a single or an isolated act that cannot be categorized as
habitual, hence, not a just cause for their dismissal.

G.R. No. 146741 | 27 February 2002 | 2nd Division | Bellosillo, J.

Loss of trust and confidence

FACTS:

Loss of trust and confidence to be a valid ground for dismissal must be based
on a willful breach of trust and founded on clearly established facts. A breach is
willful if it is done intentionally, knowingly and purposely, without justifiable
excuse, as distinguished from an act done carelessly, thoughtlessly, heedlessly or
inadvertently.

19.

National Book store v CA 278 SCRA 194 (2002)

NATIONAL BOOKSTORE, INC. VS. COURT OF APPEALS (CELEBRADO)

National Bookstore (NBS) employed Ymasa and Gabriel as its Cash


Custodian and Head Cashier, respectively. In NBS (SM North EDSA branch),
Ymasa counted the money to be deposited with INTERBANK while Gabriel
counted that for PCIB, in the presence of a watcher, Cupcupin. The money was
placed inside 2 separate sealed plastic bags. Ymasa put the plastic bags inside
her locked cabinet.
Asst. Mgr. Tagalog retrieved the money to be picked up by the banks for deposit,
but the amount for deposit to PCIB was found short of P42K.
NBS asked them to explain in writing why they should not be dismissed for the
loss of company funds and placed them under preventive suspension. However,
they denied responsibility because they had no access to the vault and that before
leaving the office, the guard subjected them to a thorough body search. They
asserted that theyve been in the service of the company for the past 13 years and
it has been their practice to turn over their collection to their supervisor without
any proof of receipt.
NBS terminated them for gross neglect of duty and loss of confidence.
Thus, they filed a complaint for illegal dismissal before LA against NBS. LA,
NLRC, and CA, ruled in their favor.
ISSUE:
Whether or not there are just causes, i.e. gross neglect of duty and loss of
confidence, for their dismissal? [No]
HELD:
NO.
Gross negligence
Gross negligence has been defined as the want or absence of or failure to
exercise slight care or diligence, or the entire absence of care. It evinces a
thoughtless disregard of consequences without exerting any effort to avoid them.

The Labor Arbiter, the NLRC and the Court of Appeals were unanimous in
declaring that there was no willful breach of confidence in the instant case as
NBS failed to establish with certainty the facts upon which it could be based.
Indeed, NBS lost some funds but that their dismissed employees were responsible
therefor was not supported by any substantial evidence.
20.

PNB v Dan Padao GR No. 180849, 187143 (2011)

Dan Padao was a loan and credit officer of the Philippine National Bank (PNB),
Dipolog City Branch. He was charged for having presented a deceptively positive
status of the business, credit standing/rating and financial capability of loan
applicants Reynaldo and Luzvilla Baluma and 11 others. It was later found that
either said borrowers business was inadequate to meet their loan obligations, or
that the projects they sought to be financed did not exist.
Padao reasoned out that his reports were done in compliance with the apparently
lawful orders of his supervisors. It was a management-sanctioned deviation,
which did not amount to fraud or illegal activities. Does his explanation find
merit?
Ruling: No.
Padao, in affixing his signature on the fraudulent reports, attested to the
falsehoods contained therein. Moreover, by doing so, he repeatedly failed to
perform his duties as a credit investigator.
Article 11(6) of the Revised Penal Code requires that any person, who acts in
obedience to an order issued by a superior, does so for some lawful purpose in
order for such person not to incur criminal liability. The succeeding article
exempts from criminal liability any person who acts under the compulsion of an
irresistible force (Article 12, paragraph 6) or under the impulse of an
uncontrollable fear of an equal or greater injury (Article 12, paragraph 7).
13

Assuming solely for the sake of argument that these principles apply by analogy,
even an extremely liberal interpretation of these justifying or exempting
circumstances will not allow Padao to escape liability.
Gross negligence connotes want of care in the performance of ones duties, while
habitual neglect implies repeated failure to perform ones duties for a period of
time, depending on the circumstances. In the case at bench, Padao was accused of
having presented a fraudulently positive evaluation of the business, credit
standing/rating and financial capability of Reynaldo and Luzvilla Baluma and
eleven other loan applicants. Some businesses were eventually found not to exist
at all, while in other transactions, the financial status of the borrowers simply
could not support the grant of loans in the approved amounts. Moreover, Padao
over-appraised the collateral of spouses Gardito and Alma Ajero, and that of
spouses Ihaba and Rolly Pango. Padaos repeated failure to discharge his duties
as a credit investigator of the bank amounted to gross and habitual neglect of
duties under Article 282 (b) of the Labor Code. He not only failed to perform what
he was employed to do, but also did so repetitively and habitually, causing
millions of pesos in damage to PNB. Thus, PNB acted within the bounds of the
law by meting out the penalty of dismissal, which it deemed appropriate given the
circumstances.
21.

Jumad v HI Flyer Food Inc GR. No. 187887 (2011)

22.

MECO v Ma Luisa Beltran GR No. 173774 (2012)

Meralco v. Beltran
FACTS:
-

Beltran was employed by Meralco as Senior Branch Clerk


While rendering OT, she received from customer Andy Chang the
payment in lieu of his returned check the latter earlier gave in the
amount of 15k
Beltran was hesitant since it was not part of her duties to accept
payments from customers but was persuaded by Chang
She issued an Auxiliary Receipt dated 2 days after the actual payment
date to show that it was an accommodation, an accepted practice in the
office
All pertinent docs, including the money, was placed in her drawer
Beltran was however only able to remit said payment after four months
after her immediate supervisor called her attention
She was given a memo placing her under preventive suspension pending
investigation
Meralco considered as misappropriation or withholding of company funds
her failure to immediately remit said payment in violation of its Code on
Employee Discipline
Investigation by Meralco adjudged Beltran guilty noting that on the day
she was reminded, she failed to remit; failed to remit the day after; and
took a leave of 2 days before managing to submit the money and docs
Recommendation was dismissal which Meralco subsequently
implemented
Beltran filed complaint for illegal dismissal and argued that:

she had no intention to withhold said funds and claimed good


faith;
o that it was not her customary duty to collect and remit
payments;
o her accommodation of Chang was in fact goodwill in favor of
Meralco and its customers;
o if any, she was only guilty of delay in remitting the payment;
o she had 9 years of unblemished service which should not
warrant her dismissal as penalty
LA: dismissal not commensurate with infraction committed; backwages
forfeited
NLRC: dismissal is valid
CA: agreed with LA, failure was unintentional and not attended by any
ill motive
o

ISSUE(S):
w/n Beltran was validly dismissed
w/n dismissal is warranted under the circumstances
HOLDING:
No, for loss of trust and confidence to be a valid ground of dismissal it must be
based on a willful breach of trust and founded on clearly established facts. A
breach is willful if it is done intentionally, knowingly and purposely, without
justifiable excuse, as distinguished from an act done carelessly, thoughtlessly,
heedlessly or inadvertently. In addition, loss of trust and confidence must rest on
substantial grounds and not on employers arbitratriness, whims, caprices or
suspicion.
Beltran attributes her delay to marital problems and from the fact that her ill
child was suffering an illness. Beltran admits having been reminded but denied
having been ordered to effect the remittance that same day. She took her leave in
order to attend to her child who was suffering from asthma.
Her having taken a leave of absence after having been reminded of the
unremitted funds cannot be the basis for concluding misappropriation. This is
mere suspicion. Although the reasons Beltran proffered are mere allegations, the
onus probandi of providing clear and convincing facts upon which the alleged loss
of trust and confidence still lies on Meralco.
Beltran was remiss in her duties, but such negligence is not sufficient to warrant
separation from employment. To justify removal from service, the negligence
should be gross and habitual. No concrete evidence was presented by Meralco to
show that Beltrans delay was done intentionally.
Moreover, her negligence did not result in any loss.
Under such circumstances, Meralcos dismissal will not be commensurate to her
failure to remit not only because there was no clear showing of bad faith and
malice but also in consideration of her untainted record of long and dedicated
service to Meralco.
14

The forfeiture of salary is an equitable punishment for the simple negligence


committed.
Reinstatement without backwages.
23.

Lima Land Inc v Cuevas GR No. 169523 (2010)

2010 Lima Land, et al v Cuevas

ISSUE - whether petitioners validly dismissed respondent from her employment.


Held: NO

Marlyn Cuevas was Finance and Administration Manager of Lima Land.

There was compliance with due process requirement and but dismissal was not
for valid cause.

1996, Lima entered into lease agreements known as arriendo contracts


with different persons whereby itits right to harvest coconuts, other
fruits to lessees, for consideration.

There was no sufficient evidence to prove that petitioners are justified


in terminating respondent's employment on the basis of loss of trust and
confidence.

The collection of the proceeds were under the direct supervision of Jonas
Senia, Operation and Estate Manager at Lima Land, Batangas, assisted
by Flor San Gabriel, Site Assistant and Imelda Melo, Liaison Assistant.
The arriendo collections were, thereafter, remitted to the Head Office in
Makati.
2000, irregularities in the arriendo collections were discovered.
Investigations revealed fraudulent activities and irregularities
committed by the Cuevas relative to the Company funds.
She was served two notices and was given a chance to appear at hearings
before she was terminated
The grounds cited for her termination were: 1)
reasonable diligence to inquire about the status
arriendo collections; 2) approving a patently
reimbursement of representation expenses; and 3)
sufficient accounting standards.

failure to exercise
of the unremitted
false request for
failure to institute

Loss of trust and confidence must be based not on ordinary breach by


the employee of the trust reposed in him by the employer, but, in the
language of Article 282 (c) of the Labor Code, on willful breach. A breach
is willful if it is done intentionally, knowingly and purposely, without justifiable
excuse, as distinguished from an act done carelessly, thoughtlessly, heedlessly or
inadvertently.
Cuevass negligence or carelessness in handling the arriendo collections,
however, are not justifiable grounds for petitioners' loss of trust and
confidence in her, especially in the absence of any malicious intent or
fraud on respondents part.
Cuevass dismissal also inspires suspicion of ill motive on the part of
petitioners considering that Senia, the Operations and Estate Manager
in their Batangas estate, was cleared of any accountability and allowed
to resign when he should be the first to be made liable, considering that
he was the one who had direct and immediate control and supervision
over the arriendo transactions and collections. Conversely, if there was
indeed no basis to hold Senia liable, then the Court agrees with
respondent that with more reason should she be exonerated of the
charges of

Cuevas filed a Complaint for illegal dismissal.

Petition is DENIED. CA affirmed.

LA dismissed complaint

24.

Elmer Lopez v Keppel Bank Phil. Inc. et al. GR No. 176800 (2011)

Lopez v. Keppel Bank


NLRC reversed

J. Brion

CA affirmed NLRC there was illegal dismissal

Facts:

Cuevas wasnt responsible for monitoring the arriendo collections and,


therefore, cannot be dismissed on the ground of loss of trust and
confidence

Elmer Lopez was the Branch Manager of the Keppel Bank Philippines,
Inc. in Iloilo City. Allegedly, through his efforts, Hertz Exclusive Cars,
Inc. (Hertz) became a client of the bank.
15

By notice dated August 12, 2003,[4] the bank asked Lopez to explain in
writing why he should not be disciplined for issuing, without authority,
two purchase orders (POs) for the Hertz account amounting to a total of
P6,493,000.00, representing the purchase price of 13 Suzuki Bravo and
two Nissan Exalta vehicles.
Lopez submitted his written explanation on the same day, but the bank
refused to give it credit. Through respondents Manuel Bosano III (VicePresident and Head of Retail Banking Division/Consumer Banking
Division) and Stefan Tong Wai Mun (Vice-President/Comptroller), the
bank terminated Lopezs employment effective immediately.
Lopez asked the bank for reconsideration. In response, the bank, through
the respondent officers, met with Lopez at its headquarters in Cubao,
Quezon City on September 25, 2003. Lopez came with his lawyer (Atty.
Edmundo V. Buensuceso) and a military man (one Col. Flordeliza).
After the meeting, the bank found no reason to reconsider and reiterated
its decision to dismiss Lopez. Lopez filed a complaint for illegal dismissal
and money claims against the bank, Bosano and Tong.

Issue:

Is Lopez liable for loss of trust and confidence for issuing the two
disputed POs?

25.

Cathedral School of Technology v NLRC 214 SCRA 551 (1992)

CATHEDRAL SCHOOL OF TECHNOLOGY vs. NATIONAL LABOR


RELATIONS COMMISSION
G.R. No. L-101438; October 13, 1992
FACTS:

February 1981- Teresita Vallejera sought admission as an aspirant to the


Congregation of the Religious of Virgin Mary (RVM)

During the period of her aspirancy and in return for her


accommodations, she volunteered to assist as a library aide in the library
section of the Cathedral School of Technology, an educational institution
run by the RVM sisters (monthly allowance of 200.00 given to her)

She had a change of heart and no longer wanted to be a nun. But she
requested to be allowed to continue living with the sisters, which was
granted. (her voluntary duties also continued)

January 1988-She was appointed to the position of library aide with a


monthly salary of P1,171.00. (BUT: sisters then began receiving
complaints' from students and employees about private respondent's
difficult personality and sour disposition at work)

June 1989- She was asked to explain why the Chief Librarian resigned
due to irreconcilable differences with her plus about the other negative
reports about her (she was reprimanded)

She resented the observations and was completely unreceptive to the


advice given by her superior. She reacted violently to remarks and
angrily offered to resign, repeatedly saying, "OK, I will resign. I will
resign." AND without waiting to be dismissed from the meeting, she
stormed out of the office in discourteous disregard and callous defiance of
authority

After trying to settle the differences, the school then decided to accept
her resignation and gave her a month to look for a new job.

Teresita filed complaint for Illegal Dismisal

Held:

Yes. The right of an employer to freely select or discharge his employee is


a recognized prerogative of management; an employer cannot be
compelled to continue employing one who has been guilty of acts inimical
to its interests.
When this happens, the employer can dismiss the employee for loss of
confidence. At the same time, loss of confidence as a just cause of
dismissal was never intended to provide employers with a blank check
for terminating employment. Loss of confidence should ideally apply only
to cases involving employees occupying positions of trust and confidence,
or to situations where the employee is routinely charged with the care
and custody of the employers money or property. To the first class belong
managerial employees, i.e., those vested with the powers and
prerogatives to lay down management polices and/or to hire, transfer,
suspend, lay-off, recall, discharge, assign or discipline employees, or
effectively recommend such managerial actions. To the second class
belong cashiers, auditors, property custodians, or those who, in the
normal and routine exercise of their functions, regularly handle
significant amounts of money or property.
As branch manager, Lopez clearly occupies a "position of trust." As a
bank official, the petitioner must have been aware that it is basic in
every sound management that people under ones supervision and
direction are bound to follow instructions or to inform their superior of
what is going on in their respective areas of concern, especially regarding
matters of vital interest to the enterprise. What appears clear is that the
bank cannot in the future trust the petitioner as a manager who would
follow directives from higher authorities on business policy and

directions. The bank can be placed at risk if this kind of managerial


attitude will be repeated, especially if it becomes an accepted rule among
lower managers.
In Nokom v. NLRC, we reiterated the guidelines for the application of
loss of confidence as follows: (1) loss of confidence, should not be
simulated; (2) it should not be used as a subterfuge for causes which are
improper, illegal or unjustified; (3) it may not be arbitrarily asserted in
the face of overwhelming evidence to the contrary; and (4) it must be
genuine, not a mere afterthought to justify an earlier action taken in
bad faith.

She claims she was hired as a library aide since 1981

And that she was forced to tender resignation


16

LA: Illegally Dismissed for lack of due process

NLRC affirmed LA

ISSUES:
1.

W/N lack of due process is a valid ground for illegal dismissal. YES

HELD:
The Court held, the existence of an employer-employee relationship is essentially
a factual question and the respondent commission's findings thereon are accorded
great weight and respect and even finality when the same are supported by
substantial evidence. There was found no reason to overrule the same.
An evaluative review of the records of this case nonetheless supports a finding of
a just cause for termination. The reason for which private respondent's services
were terminated, namely, her unreasonable behavior and unpleasant deportment
in dealing with the people she closely works with in the course of her
employment, is analogous to the other "just causes" enumerated under the Labor
Code, as amended:
Art. 282. Termination by employer. An employer may terminate an employment
for any of the following just causes:
(a) Serious misconduct or willful disobedience by the employee of the lawful orders
of his employer or representative in connection with his work;
(b) Gross and habitual breach by the employee of his duties;
(c) Fraud or willful breach by the employee of the trust reposed in him by his
employer or duly authorized representative;
(d) Commission of a crime or offense by the employee against the person of his
employer or any immediate member of his family or his duly authorized
representative; and
(e) Other causes analogous to the foregoing.
On the matter of illegal dismissal, petitioners do not dispute the findings, and in
effect admit, that private respondent was denied her right to due process. As
found by the labor arbiter, no hearing on the impending dismissal was conducted
as would have afforded private respondent an opportunity to explain her side and,
if need be, to defend herself.

Nevertheless, we find no merit in public respondent's ratiocination, quoting the


labor arbiter, that: It is likewise the finding (of) and this Branch so holds that
complainant was illegally dismissed. This finding is anchored on the lack of due
process. In this case complainant was not afforded the opportunity to defend
herself in a hearing called for the purpose. She was just summarily dismissed
contrary to the provision(s) of Batas Pambansa Bilang 130 and its Implementing
Rules.
Clearly, therefore, its ruling that private respondent was illegally dismissed was
premised solely on the fact of alleged lack of procedural due process, without
regard to whether or not there was lawful cause for such dismissal, which latter
aspect constitutes the element of substantive due process. We accordingly proceed
to resolve the issue that is thereby presented.
It is the contention of petitioners that dismissal for cause but without due process
does not warrant an order for reinstatement or separation pay, as the case may be
nor of backwages, for these are sanctions that pertain to dismissals without just
cause. On the other hand arbitrary dismissal for just cause only warrants an
award of indemnity for the dismissed employee.
We grant our imprimatur to this submission of petitioners, just as we view with
disfavor public respondent's intransigence on the matter in this and other cases
despite our pronouncements thereon.
It is true that, exceptionally and as an equitable concession, separation pay may
be allowed as a measure of social justice but only in those instances where the
employee is validly dismissed for causes other than serious misconduct or those
reflecting on his moral character. However, such exceptional circumstance does
not obtain in the present case.
In Section 7, Book VI of the Implementing Rules of the Labor Code which
expressly states that:
Sec. 7. That just causes for terminating the services of an employee shall be those
provided in Article 282 of the Code. The separation from work of an employee for a
just cause does not entitle him to the termination pay provided in the Code,
without prejudice, however, to whatever rights, benefits and privileges he may
have under the applicable individual or collective bargaining agreement with the
employer or voluntary employer policy or practice.
26.

Lim v NLRC 259 SCRA 485 (1996)

True, petitioners notified her of the school's decision to terminate her services.
But notice alone, without the requisite hearing does not suffice. Albeit with some
ambiguity which will hereafter be clarified, this Court has held that:

SIXTA

C. LIM, petitioner, vs. NATIONAL LABOR RELATIONS


COMMISSION
and
PEPSI-COLA
FAR
EAST
TRADE
DEVELOPMENT CO., INC.,respondents.

Under the Labor Code, as amended, the requirements of lawful dismissal of an


employee by his employer are two-fold: the substantive and the procedural. Not
only must the dismissal be for a valid or authorized cause as provided by law
(Arts. 279, 281, 282-284), but the rudimentary requirements of due process
notice and hearing must also be observed before an employee may be dismissed
(Art. 277 [b]). One cannot go without the other, for otherwise the termination
would, in the eyes of the law, be illegal.

G.R. No. 118434. July 26, 1996 DAVIDE, JR., J.:


Facts

PEPSI employed petitioner on 15 June 1983, but she had been with
the Pepsi Group since 1 January 1981 as a secretary for Pepsi
Bottling Co. (Phils.), Inc.

17

At the time of her dismissal, she held the position of Staff


Accountant.
o She assisted and worked closely with the Plant Accountant
to carry out the accounting departments tasks
o In particular, her work involved:

(1) Cost Accounting-Production Reporting (40%)

(2) Cost Accounting-Financial Reporting (20%)

(3) Payroll Reporting (15%)

(4) Statutory Reporting (15%)

(5) Preparation of daily trade accounts receivable


reports, petty cash fund custodianship, and check
preparation (10%).
Over time, the petitioners overall performance appraisals rated as
follows: (a) S (Superior) as of 1 May 1984; (b) C (Commendable)
for the period for 1 December 1987 to 31 August 1988; and (c) C-
(C minus), quantified as 81.10% for the period from 1 September
1988 to 31 May 1989.
In the latter part of 1989, PEPSI changed the rating nomenclatures
for the performance evaluation of its employees.
For the period beginning 1 July 1989 until 31 December 1989, the
petitioner received an overall rating of BT or Below Target in
the management performance appraisal. This rating was
heavily influenced by her performance in production
reporting, which accounted for forty percent (40%) of the overall
rating.
She likewise obtained a BT rating for Cost Accounting-Financial
Reporting, which was weighted at twenty percent (20%).
The overall performance appraisal of the petitioner stated
that she was below target.
Petitioner asked for reevaluation, and later wrote another letter
asking to change the weights of the grades, and made objections to
certain findings on her report.
PEPSI conducted another appraisalof the petitioners performance
for the period from 1 January 1990 to 31 December 1990 and gave
her an overall rating of below target.
Unsatisfied, the petitioner wrote a letter to Mr. Yasuyuki Mihara of
Pepsi Co, Inc., Japan, pointing out that her irregular procedures of
her superiors in handling her case i.e. they did not furnish her a
copy of the memorandum requesting to analyze her case.
PEPSI asked the petitioner to voluntarily resign and offered to pay
her termination benefits.
Petitioner was verbally informed of her termination. She filed with
the labor arbiter. She later received a termination letter on grounds
of gross inefficiency
LA: for petitioner
NLRC: reversed LA.

Issue/Holding/Ratio

WON a company could terminate an employee on the ground of


gross inefficiency? Yes, it is a just cause.

Under Article 282 of the Labor Code (P.D. No. 442), as amended,
the following are deemed just causes to terminate an employee:
(a) Serious misconduct or willful disobedience by the employee
of the lawful orders of his employer or representative in
connection with his work;
(b) Gross and habitual neglect by the employee of his duties;
(c) Fraud or willful breach by the employee of the trust reposed
in him by his employer or duly authorized representative;
(d) Commission of a crime or offense by the employee against
the person of his employer or any immediate member of his
family or his duly authorized representative; and
(e) Other causes analogous to the foregoing.

Gross inefficiency falls within the purview of other


causes analogous to the foregoing, and constitutes, therefore,
just cause to terminate an employee under Article 282 of the Labor
Code.
The test if one is analogous to another is if it is susceptible of
comparison with the latter either in general or in some specific
detail; or has a close relationship with the latter. Gross
inefficiency is closely related to gross neglect, for both
involve specific acts of omission on the part of the employee
resulting in damage to the employer or to his business.

WON the petitioner was guilty of gross inefficiency? No, she


was not.

Prior to the issuance of the Termination Letter on 15 May 1991,


PEPSI never called the petitioners attention to any alleged
gross inefficiency on her part.
The evaluation report merely indicated her areas for
improvement.
o Moreover, in PEPSIs brochure entitled Managing
Performance For the 90s, a BT rating does not merit
dismissal from the service; as a matter of fact, the lower
rating - Significantly Below Target (SB) - is not even a
ground for termination of employment, but may only justify
putting the employee on probation and [telling him] that
improvement is a necessity.
o Undoubtedly, the petitioner obtained an unfavorable
rating, but not to the extent, under the companys
standards, to warrant even a probationary measure which
is given to the lowest rating of Significantly Below Target
(SB).

18

PEPSI had not characterized as gross inefficiency whatever


failures, shortcomings, or deficiencies may have been attributable to
the petitioner. The rule is of course doctrinally entrenched
that in termination cases, the burden of proving that the
employees dismissal from employment was for just cause
rests upon the employer.

Dr. Gorospe informed SLMC's Corporate President, Jose Ledesma, who


called up the WPO to inquire if it was its policy to reject patients. The WPO
staff denied that they declined Dr. Gorospe and said that her request was
already being processed for scheduling.

Fadrigo meanwhile was not at the office while this happened, she was
enjoying her rest day when at 5 pm she received a call from SLMC's
Associate Director for Corporate Affairs, Marilen Lagniton, who informed
her what happened at the WPO. She was directed to tell Tingzon and Rillo
not to report for duty the next day.

Fadrigo immediately called the WPO and was able to talk to Gail
Manalastas, a senior associate, who confirmed what happened. However,
she was not able to talk to Tingzon and Rillo as they already left. She tried
to reach their mobile phones to no avail, and thus she instructed
Manalastas to tell both of them the next day not to work and wait for her
at the office.

The next day (a Sunday), Lagniton called the WPO and found out that
Tingzon and Rillo were in the office. She talked to them and instructed
them to go home. Thus when Fadrigo got to the office the two already left.

On 27 Apr Fadrigo received a memo requiring her to show cause why no


DA should be taken against her for insubordination, gross inefficiency and
incompetence due to the incident. The memo stated that she allowed a
trainee and a casual employee to man the WPO during official business
hours, and that she failed to comply with management order to
immediately pull out Rillo and Tingzon.

In her letter-reply she denied the charges against her, explaining that
Manalastas, a senior associate, was present at the time of the incident. She
also denied that she failed to comply with the management order regarding
Rillo and Tingzon. She also requested for a bill of particulars to state the
particular acts or omissions that amounted to insubordination and gross
negligence.

Fadrigo was issued another memo by SLMC's Committee on Values Ethics


and Discipline (COVED) thru its chair, Fe Muit. She was also summoned to
a COVED conference set on 6 May.

During the conference, Fadrigo reiterated her request for a bill of


particulars but was denied. COVED also denied her request to summon Dr.
Gorospe.

She received another memo from COVED informing her of the decision to
terminate her employment effective 18 May. In the presence of several
employees, she was subjected to a thorough search by security officers
pursuant to SLMC's directive.

She filed a complaint with the LA for illegal dismissal with prayer for
reinstatement, as well as full backwages, moral damages and attorney's
fees.

WON PEPSI denied petitioner of due process? Yes, it did.

The right of security of tenure is implemented by the requirements


of twin notice and hearing prescribed in Article 277 of the Labor
Code, as amended, and in Sections 2 to 7, Rule XIV, Book V of the
Omnibus Rules Implementing the Labor Code.
The first notice apprises the employee of the particular acts or
omissions for which his dismissal is sought, which may be loosely
considered as the proper charge; while the second informs the
employee of the employers decision to dismiss him.
The latter must come only after the employee is given a
reasonable period from receipt of the first notice within
which to answer the charge, and ample opportunity to be
heard and defend himself with the assistance of his
representative, if he so desires.
Non-compliance therewith is fatal as these requirements are
conditions sine qua non before dismissal may be validly effected.
After the petitioner wrote a letter to Mr. Yasuyuki Mihara of
Pepsico, Inc., Japan, she was twice verbally asked to
voluntarily resign, albeit with separation pay. When she
rejected the proposal, she was verbally informed of her
termination, as a consequence of which, she filed her complaint for
dismissal without due process on 14 May 1991. The formal
Letter of Termination was only prepared and served on her
on 15 May 1991.

Holding: Reinstatement with backwages


27.

St Luke's Medical Center Inc v Fabrlgo GR No. 185933 (2009)

ST. Lukes Medical Center v. Fadrigo


GR No. 185933 / 25 Nov 2009 / J. Nachura
FACTS

Jennifer Lynne Fadrigo was Customer Affairs Department Manager of


SLMC. She supervised the Wellness Program Office (WPO) which
administers SLMC's check up packages.
On 23 Apr 2005 a Dr. Gorospe called up the WPO to refer a patient for
immediate check up. The call was answered by a trainee at the front desk,
Michelle Rillo, who transferred it to a casual employee, Hazel Tingzon.
Tingzon explained to Dr. Gorospe the mechanics of a check up which could
not be administered immediately as she wanted.

19

LA - found dismissal illegal, as SLMC failed to substantiate allegations of


insubordination, gross inefficiency and incompetence. Thus her termination
was without just cause.

NLRC - reversed LA, found Fadrigo was remiss in her duties as


Department Manager, thus losing the trust and confidence of her employer.

CA - reinstated LA ruling, but in lieu of reinstatement granted separation


pay.

her. Being their immediate supervisor, the act of making them wait for her
to be personally informed of their dismissal is reasonable.

Art. 282(c) talks about a willful breach as a just cause for termination. A
breach is willful if it is done intentionally, knowingly and purposely,
without justifiable excuse, as distinguished from an act done carelessly,
thoughtlessly, heedlessly, or inadvertently. It must rest on substantial
grounds and not on the employers arbitrariness, whims, caprices or
suspicion; otherwise, the employee would eternally remain at the mercy of
the employer. It should be genuine and not simulated; nor should it appear
as a mere afterthought to justify an earlier action taken in bad faith or as a
subterfuge for causes that are improper, illegal or unjustified. It has never
been intended to afford an occasion for abuse because of its subjective
nature. There must, therefore, be an actual breach of duty committed by
the employee, which must be established by substantial evidence.

Here SLMC failed to establish the requirements. Burden of proof is on the


employer that the dismissal is for a just cause.

ISSUE
W/N Fadrigo was illegally dismissed. YES
HELD

SLMC - argues that as a managerial employee, the mere existence of a


basis for believing that Fadrigo breached the trust of her employer would
suffice for dismissal.

But the lack of confidence is attributed by SLMC to gross inefficiency,


incompetence and insubordination.

Gross inefficiency - as a just cause, the inefficiency must not only be gross
but also habitual. Thus a single incident does not constitute just cause.

SLMC has not cited any specific policy prohibiting the assignment of
casuals and trainees, meanwhile Fadrigo's reply is reasonable, in that it is
a practice resorted to due to lack of manpower and management's
reluctance to hire regular employees. Also a senior staff, Manalastas, was
assigned that particular day from 7 am to 4 pm

Likewise her failure to "document WPO policies, to orient new staff, and to
act on the incident of April 6, 2005" was not included in the show cause
memo given to Fadrigo, which gives the Court reason to believe these
reasons are just afterthoughts to justify the dismissal.

Assuming this inefficiency is true, it is not habitual, as during her 5 year


stay with SLMC, she has shown exemplary performance, evidenced by
testimonials and commendations given her.

Insubordination - needs two requisites:

a.

the employee's assailed conduct must have been willful, that is,
characterized by a wrongful and perverse attitude

a.

the order violated must have been reasonable, lawful, made known
to the employee, and must pertain to the duties which he has been
engaged to discharge

The facts show that she tried her best to comply with management orders.
She tried to call Rillo and Tingzon on their mobiles, she gave instructions
to the staff to make the two stop working the next day and just wait for

Dispositive:

Full backwages, moral damages, however no reinstatement due to strained


relations, thus separation pay in lieu of.

28.
Flordeliza Marla Reyes-Rayel v. Philippine Luen Thai Holdings
Corporation, et al.
Flordeliza Maria Reyes-Rayel v. Philippine
Corporation/L&T (Digest by Robby Solis)

Luen

Thai

Holdings

(Del Castillo, Jr.; G.R. 174893)


(CAUTION: As we have 3 cases each on short notice, I utilized a Sun-Star Digest
to help make this. Credit to the source, and please note that the digest may not be
as comprehensive. Thanks.)
Facts:

Feb 2000 - PLTHC hired petitioner as Corporate Human Resources (CHR)


Director for Manufacturing for its subsidiary/affiliate company, L&T. In the
employment contract, petitioner was tasked to perform functions in relation
to administration, recruitment, benefits, audit/compliance, policy development/structure, project plan, and such other works as may be assigned by her
immediate superior, Frank Sauceda, PLTHCs Corporate Director for Human
Resources.

Sep. 12, 2001 - Reyes-Rayel, through a Prerequisite Notice, was dismissed


from the service by respondents Philippine Luen Thai Holdings Corp. and
L&T International Group Philippines, Inc. for loss of confidence in her ability
to promote their interests.

LA: Reyes-Rayel illegally dismissed; to be paid backwages, 13th month pay


and VL.
20

NLRC: MR of Luen Thai dismissed


CA:

1st

Appeal of Luen Thai dismissed

EG Construction Corporation v Sato


Facts
.Petitioner corporation is engaged in the construction business. They hired
respondents as technical laborers, steelmen, etc. eventually respondents
were dismissed.

However, on MR, CA modified and Reversed, dismissal is legal but


Reyes-Rayel to be paid P240,000 as compensation for monthly salary

She filed a complaint for illegal dismissal, praying for separation pay, 13th
month pay, moral and exemplary damages and attorneys fees.

The petitioner claimed, among others, that her dismissal was effected without the observance of due process since she was not afforded a hearing. Does
this claim find merit?

Issue: WON Reyes-Rayel was illegally dismissed as she was not properly
informed of the charges against her and was not affored a proper hearing.

.With regard to Sato, petitioner claimed that Sato was disciplined for having
affairs with a woman, and in retaliation Sato filed a complaint with the
SSS against the corporation.
.With regard to the other respondents, it was alleged that they abandoned
their jobs.
Issues
.WON respondents abandoned their jobs? No.
.petitioner corporation failed to prove that respondents were dismissed
for just or authorized cause.

Held/Ratio: No.

The following are the guiding principles in connection with the hearing requirement in dismissal cases:

.For abandonment to exist, essential that:


.employee must have failed to report for work or must have been
absent without valid or justifiable reason;

(a) ample opportunity to be heard means any meaningful opportunity


(verbal or written) given to the employee to answer the charges against him
and submit evidence in support of his defense, whether in a hearing,
conference or some other fair, just and reasonable way.

.that there must have been a clear intention to sever the employeremployee relationship manifested by some overt act.

(b) a formal hearing or conference becomes mandatory only when requested


by the employee in writing or substantial evidentiary disputes exist or a
company rule or practice requires it, or when similar circumstances justify it.

.Mere absence is not equal to abandonment. There must be unequivocal


intent on the part of employee to discontinue his employment.
.Reason why respondents failed to report for work was because petitioner
barred them from entering the construction sites.

(c) the ample opportunity to be heard standard in the Labor Code prevails
over the hearing or conference requirement in the implementing rules and
regulations.

In this case, petitioners written response to the Prerequisite Notice provided


her with an avenue to explain and defend her side and thus served the purpose of due process.

That there was no hearing, investigation or right to appeal, which petitioner


opined to be violation of company policies, is of no moment since the records
are bereft of any showing that there is an existing company policy that requires these procedures with respect to the termination of a CHR Director
like petitioner or that company practice calls for the same. There was also no
request for a formal hearing on the part of petitioner.

As she was served with a notice apprising her of the charges against her and
also a subsequent notice informing her of the managements decision to terminate her services after respondents found her written response to the first
notice unsatisfactory, petitioner was clearly afforded her right to due process.

Dispositive: Petition DENIED. Assailed CA Decision AFFIRMED.


29.

EG & Const Corp v Sato GR No. 182070 (2011)

.Failure to report for work after notice of return does not constitute
abandonment when there is no proof.
.Corporation's defense that it paid the money claims is not tenable either.
The party that pleads payment has the burden of proving the same.
The corporation is the one who has access to files and records that
can prove payment. It was not, however, able to prove that it did pay
the respondents.
30.

Mario Dlmagan v Dacworks United GR No. 191053 (2011)

Dimagan vs. Dacworks


FACTS:

Petitioner Mario B. Dimagan is a stockholder of respondent DACWORKS


UNITED, INC., which is engaged in the business of installing,
maintaining and repairing airconditioning systems. In July 1997, he
started working for respondent company as OIC) for mechanical
installation with a monthly salary of P8,000.

21

Sometime in 2002, petitioner was downgraded from his post as OIC to


supervisor. Then, in March of the following year, he was made to work as
a mere technician. When he vocally expressed his concerns regarding his
assignments, one Loida Aquino, who was in charge of servicing/personnel
under the direct supervision of respondent Dean A. Cancino, told him not
to report for work anymore. This prompted petitioner to file a complaint
for illegal dismissal, non-payment of overtime pay, holiday pay, service
incentive leave and separation pay against respondents.
Respondents denied that petitioner was illegally dismissed arguing that,
since April 4, 2003 up to the time of the filing of the complaint, petitioner
never reported for work and continuously violated the company policy on
absence without official leave. They allegedly sent a total of 4
memoranda informing petitioner of his offenses, including being AWOL,
but he nonetheless unjustifiably refused to return to work.

ISSUE/S: whether petitioner was illegally or constructively dismissed.


HELD: Yes.

Constructive dismissal is defined as a quitting because continued


employment is rendered impossible, unreasonable or unlikely; when
there is a demotion in rank or a diminution of pay.24 The test of
constructive dismissal is whether a reasonable person in the employee's
position would have felt compelled to give up his position under the
circumstances. It is an act amounting to dismissal but is made to appear
as if it were not. Constructive dismissal is therefore a dismissal in
disguise. The law recognizes and resolves this situation in favor of
employees in order to protect their rights and interests from the coercive
acts of the employer.
Similarly, it cannot be said that it was petitioner who abandoned his
employment by failing to report for work or having gone AWOL.
o Abandonment is the deliberate and unjustified refusal of an
employee
to
resume
his employment. To
constitute
abandonment of work, two elements must concur: (1) the
employee must have failed to report for work or must have been
absent without valid or justifiable reason; and (2) there must
have been a clear intention on the part of the employee to sever
the employer-employee relationship manifested by some overt
act. The employer bears the burden of proof to show the
deliberate and unjustified refusal of the employee to resume his
employment without any intention of returning.
In this case, petitioner's failure to report for work was caused by the
unwarranted demotion in rank that was imposed upon him by
respondents, not by any intention to sever employment ties with them.
And his filing of the instant complaint for illegal dismissal indubitably
negates the allegation of abandonment. Had petitioner intended to
forsake his job, then he would not have found it necessary to institute
this case against respondents.

31.

Batongbacal v Associated Bank 168 SCRA 7600 (1988)

32.

Manlimos v NLRC 242 SCRA 145 (1995)

MANLIMOS - BUENAVENTURA
FACTS
The petitioners were among the regular employees of the Super Mahogany
Plywood Corporation, hired as patchers, taper-graders, and receivers-dryers. On 1
September 1991, a new owner/management group acquired complete ownership of
the corporation. The petitioners were advised of such change of ownership;
however, they continued to work for the new owner and were considered
terminated, with their conformity, only as of December 1991 when they received
their separation pay, 13th month pay, and all other benefits due them computed
as of the said month. Each of them then executed on 17 December 1991 a Release
and Waiver. On 27 December 1991, the new owner caused the publication of a
notice for the hiring of workers, indicating therein who of the separated
employees could be accepted on probationary basis. The petitioners then filed
their applications for employment. Except for Rosario Cuarto, they were hired on
probationary basis for six months as patchers or tapers, but were compensated on
piece-rate or task basis. For their alleged absence without leave, Perla Cumpay
and Virginia Etic were considered, as of 4 May 1992, to have abandoned their
work. The rest were dismissed on 13 June 1992 because they allegedly committed
acts prejudicial to the interest of the new management. Petitioners Ronald Booc,
Jaime Timbal, German Gista, Federico Amper, Francisco Evale, and Renante
Yacapin then filed against the private respondent for "non-payment of wages,
underpayment of wages, incentive leave pay, non-payment of holiday pay,
overtime pay, 13th month pay, separation pay, reinstatement with back wages,
illegal termination and damages." Petitioners Ronald Manlimos, Froilan Pagalan,
Merlita Duhay Lungsod, Elizabeth Andagan, Doris Serdan, Leonora Bibiano,
Perla Cumpay, Virginia Etic, Remegia Noel, and Rosario Cuarto also filed against
the private respondent for "illegal termination; reinstatement with back wages;
non-payment of wages; underpayment of wages; non-payment of incentive leave
pay, overtime pay, 13th month pay; and damages." The petitioners maintained
that they remained regular employees regardless of the change of management
and their execution of the Release and Waiver. They argue that being a
corporation, the private respondent's juridical personality was unaffected even if
ownership of its shares of stock changed hands. Their signing of the Release and
Waiver was of no moment not only because the consideration was woefully
inadequate, but also because employees who receive their separation pay are not
barred from contesting the legality of their dismissal and quit claims executed by
laborers are frowned upon for being contrary to public policy. Respondent
contended that the petitioners were deemed legally terminated from their
previous employment as evidenced by the execution of the Release and Waiver
and the filing of their applications for employment with the new owner; that the
new owner was well within its legal right or prerogative in considering as
terminated the petitioners' probationary/temporary appointment; and that the
petitioners were not illegally dismissed. The Labor Arbiter found for petitioners
and held that the transfer of ownership partook of a cessation of business
operation not due to business reverses under Article 283 of the Labor Code.
NLRC reversed.
Issue: WON petitioners were illegally dismissed.
22

HELD: NO. A change of ownership in a business concern is not proscribed by law.


In Central Azucarera del Danao vs.Court of Appeals, this Court-stated: There can
be no controversy for it is a principle well-recognized, that it is within the
employer's legitimate sphere of management control of the business to adopt
economic policies or make some changes or adjustments in their organization or
operations that would insure profit to itself or protect the investment of its
stockholders. As in the exercise of such management prerogative, the employer
may merge or consolidate its business with another, or sell or dispose all or
substantially all of its assets and properties which may bring about the dismissal
or termination of its employees in the process. Such dismissal or termination
should not however be interpreted in such a manner as to permit the employer to
evade the very concept of social justice. In a number of cases on this point, the
rule has been laid down that the sale or disposition must be motivated by good
faith as an element of exemption from liability. Indeed, an innocent transferee of
a business establishment has no liability to the employees of the transfer or to
continue employing them. Nor is the transferee liable for past unfair labor
practices of the previous owner Where such transfer of ownership is in good
faith, the transferee is under no legal duty to absorb the transferor employees as
there is no law compelling such absorption. The most that the transferee may do,
for reasons of public policy and social justice, is to give preference to the qualified
separated employees in the filling of vacancies in the facilities of the purchaser.
14 Since the petitioners were effectively separated from work due to a bona fide
change of ownership and they were accordingly paid their separation pay, which
they freely and voluntarily accepted, the private respondent corporation was
under no obligation to employ them; it may, however, give them preference in the
hiring. The private respondent in fact hired, but on probationary basis, all the
petitioners, except Rosario Cuarto. The non-hiring of Cuarto was legally
permissible. As for Perla Cumpay and Virginia Etic the private respondent not
only failed to prove intent to abandon, it as well violated the due process rule in
dismissal of employees. The requirements of lawful dismissal of an employee by
his employer are two-fold, viz., notice and hearing. These requirements constitute
the essential elements of due process and not having been met, their dismissal
was, consequently, illegal.
33.

Elcee Farms v NLRC GR No. 126428 (2007)

ELCEE FARMS and SAGUEMUELLER v. NLRC


J. Valdez
G.R. No. 505050
TOPIC: Just causes; Moral damages are recoverable when the dismissal of an
employee is attended by bad faith or fraud or constitutes an act oppressive to labor,
or is done in a manner contrary to good morals, good customs or public policy.
Exemplary damages, on the other hand, are recoverable when the dismissal was
done in a wanton, oppressive, or malevolent manner. Bad faith on the part of Elcee
Farms is shown by the act of simulating a lease agreement with Garnele in order
to evade paying private respondents the proper amount of separation benefits
based on the number of years they worked in the hacienda, as provided by the
Labor Code.

FACTS: Pampelo Semillano and 143 other complainants, represented by Sugar


Agricultural Industrial Labor Organization (SAILO) were employees of Elcee
Farms and Corazon Saguemueller (later on revealed that priv resps mistook her
for the company president). Elcee Farms entered into a lease contract with
Garnele, though private respondents continued to work in the hacienda, while
Elcee remained as their employer, as indicated in the payroll. Garnele sublet the
property to Daniel Hilado, owner of HILLA, the sublease contract of which
indicated the continued employment of 120 of the formers employees, though it
was silent as to their benefits as a result of their employment with Elcee.
HILLA entered into a closed shop agreement with the workers, which stipulated
that those not part of the union shall be dismissed in 30 days following the
agreement, upon recommendation of the Union. Due to their refusal to join the
Union, private respondents lost their jobs after HILLA dismissed them. SAILO
and 144 complainants filed a complaint for ILLEGAL DISMISSAL against Elcee
Farms before HILLA took possession of the farm, pointing out that Elcee failed to
present proof they were employed by Garnele to substantiate a valid lease
agreement.
LA: Awarded separation pay
NLRC: Awarded moral damages and separation pay; held that lease contract
between Elcee and Garnele was simulated as payrolls indicated it was Elcee who
paid their wages; SSS Forms E-4 indicated Elcee as employer; simulation of lease
smacks of bad faith and was held as ground for MD; ELCEE should have apprised
its employees of the lease to HILLA, as Elcee was obligated to pay separation pay
and other benefits due since the lease to HILLA was a termination of ER-EE
relationship; HILLA no liability as termination was due to a valid closed shop
agreement
ISSUE: Whether or not the NLRC erred in awarding separation pay and moral
damages to private respondents (NO)
RESOLUTION: NLRC decision AFFIRMED
Lease agreement; Contract between Elcee and Garnele was simulated;
Payrolls, SSS E-4 forms indicated Elcee was still private respondents employer.
Lease document wa a haphazardly prepared two-page document which did not
provide for private respondents employment status. Employees were not
informed of the lease agreement.
D. Separation pay; Private respondents entitled; Liability for separation
pay is provided for in LC Art. 283, and three requirements need to be shown:
a.

Service of a written notice to the employees and the SOLE

b.

Cessation of or withdrawal from business operations must be


bona fide in character

c.

Payment to employees of termination pay amounting to at least


for every year in service, or one month pay, whichever is
higher
23

E.

F.

Moral damages; When awarded; In cases of dismissal, moral damages are


awarded when dismissal is tainted with fraud, bad faith, or constitutes an act
oppressive to labor, or is done in a manner contrary to good morals, good customs, or public policy. Exemplary damages are recoverable when dismissal
was done in a wanton, oppressive, or malevolent manner. Bad faith is deducible from the simulated lease agreement between Elcee and Garnele, as
well as their failure to apprise the employees of the lease to HILLA. When
the employees filed their complaint, Elcee tried to deny liability by using the
lease with Garnele as a subterfuge to deny their claims due to prescription.
Elcee said that their ER-EE relationship was severed in 1987, and that it
complained was filed in 1990, > 3 years.
Liability of incorporator; Corazon not liable; It was in fact one of Corazons sons who was the president of Elcee, despite private respondents averments that they regularly came to Corazon for help. As a general rule, corporations are not solidarily liable with their incorporators, as the former is
vested with a personality separate and distinct from those persons comprising it.

34.

Manila Electric Co. v NLRC 263 SCRA 531 (1996)

Manila Electric Co v. NLRC


24 October 1996
J. Hermosisima

Complaint for illegal dismissal


Jeremias C. Cortez, Jr. was employed on probationary status by petitioner Manila Electric Company (Meralco) on September 15, 1975 as a lineman driver. Six months later, he was regularized as a 3rd class linemandriver. In 1977, and until the time of his dismissal, he worked as 1st
class lineman-driver. Characteristics, however, of private respondents
service with petitioner is his perennial suspension from work.
Due to his numerous infractions, private respondent was
administratively investigated for violation of Meralcos Code on
Employee Discipline, particularly his repeated and unabated absence
from work without prior notice his superior. After the administrative
investigation by Meralco, it was concluded by Meralco that Cortez have
grossly neglected his duties by not attending to his work as lineman
from Aug. 2, 1989 to September 19, 1989 without notice to his superiors.
In a letter dated January 19, 1990, Cortez was notified of the
investigation result and consequent termination of his services
effective January 19, 1990.

Issue: W/N Cortez was illegally terminated. YES


Ratio:
Article 283 of the Labor Code enumerates the just causes for termination.
Amongsuchcauses are the following:

a) Serious misconduct or willful disobedience by the employee of the


lawful orders of his employers or representatives in connection with his
work.
b) Gross and habitual neglect by the employee of his duties.
This cause includes gross inefficiency, negligence and carelessness. Such just
causes is derived from the right of the employer to select and engage his
employees. This court had defined a valid exercise of management prerogative as
one which covers: hiring work assignment, working methods, time, place and
manner of work, tools to be used, processes to be followed, supervision of workers,
working regulations, transfer of employees, work supervision, lay-off of workers,
and the discipline, dismissal and recall of workers. Except as provided for, or
limited by, special laws, an employer is free to regulate, according to his own
discretion and judgment, all aspects of employment
This Court has upheld a companys management prerogatives so long as they are
exercised in good faith for the advancement of the employers interest and not for
the purpose of defeating or circumventing the rights of the employees under
special laws or under valid agreements.
In the case at bar, the service record of private respondent with petitioner is
perpetually characterized by unexplained absences and unauthorized sick leave
extensions. The nature of his job i.e. as a lineman-driver requires his physical
presence to minister to incessant complaints often faulted with electricity. The
penchant of private respondent to continually incur unauthorized absences and/or
a violation of petitioners sick leave policy finally rendered his dismissal as
imminently proper.
35.

RB Michael Press v Galit GR No. 153510 (2008)

R.B. MICHAEL PRESS vs GALIT


Nicasio Galit was an offset machine operator of defendant. He committed several
offenses during his employment for which he was issued a memo on Feb. 22, 1999:
To : Mr. Nicasio Galit
From : ANNALENE REYES-ESCOBIA
Re : WARNING FOR DISMISSAL; NOTICE OF
HEARING
This warning for dismissal is being issued for the following
offenses:
(1) habitual and excessive tardiness
24

(2) committing acts of discourtesy, disrespect in addressing


superiors
(3) failure to work overtime after having been instructed to do
so
(4)
Insubordination
willfully
or disregarding company authority

disobeying,

defying

The offenses youve committed are just causes for termination of


employment as provided by the Labor Code. You were given
verbal warnings before, but there had been no improvement on
your conduct.
Further investigation of this matter is required, therefore, you
are summoned to a hearing at 4:00 p.m. today. The hearing wills
determine your employment status with this company.
During his hearing on the 23rd, Galit admitted to the charges. The following day,
he was terminated from employment and was issued a termination letter.

Please contact Ms. Marly Buita to discuss 13th-Month Pay


disbursements.
Cordially,(SGD) Mrs. Annalene Reyes-Escobia
Galit thus files this case for illegal dismissal.
ISSUE: w/n Galit's dismissal is illegal
HELD: NO, Galit's dismissal was valid and legal.
However, there were procedural infirmities in his dismissal, making Rural
Michael Press and Escobin solidarily liable to Galit in the amount of P30,000 as
nominal damages.
Substantive:
Habitial tardiness
- is a form of neglect of duty, inimical to the general productivity and business of
employer

February 24, 1999

- was not proven by EE to be condoned by ER. That his tardiness was not
immediately sanctioned, absent any other proof, does not discharge EE's burden
to prove condonation.

Dear Mr. Nicasio Galit,

- CA's judgment that Galit's tardiness and absences are punished already by
deductions on his salary is incorrect; such is not a penalty as it is merely a
consequence of the principle: "a day's pay for a day's work."

I am sorry to inform you that your employment with this


company has been terminated effective today, February 24,
1999. This decision was not made without a thorough and
complete investigation.
You were given an office memo dated February 23, 1999
warning you of a possible dismissal. You were given a chance to
defend yourself on a hearing that was held in the afternoon of
the said date.
During the hearing, Mrs. Rebecca Velasquez and Mr. Dennis
Reyes, were present in their capacity as Production Manager
and Supervisor, respectively.
Your admission to your offenses against the company and the
testimonies from Mrs. Velasquez and Mr. Reyes justified your
dismissal from this company,

Insubordination or willful disobedience


- the 2 elements of willful disobedience were met in this case: (1) conduct is willful
(characterized by a wrongful/perverse attitude), and (2) violated order is
reasonable, lawful, made known to employee, and must pertain to EE's duties.
- EE refused to render OT work even with the knowledge that it was needed to
meet a production deadline. His excuse of not feeling well was obviously an
afterthought, as he would've taken the day off if he weren't feeling well.
- The order to render OT work is reasonable (business is a printing press), lawful
(Art. 89 of labor code on emergency overtime work), made known to him and
pertains to his duties.
Twin notice and hearing requirement
- While the two notices (1. apprising EE of charges and 2. notice of termination)
were complied with and there was a hearing for Galit, there was no reasonable
opportunity for him to defend himself. There was undue haste in his termination
(1 day hearing only, when jurisprudence provides that there must be a 5 day
period from receipt of notice for EE to prepare for hearing).
- Moreover, the two notices were inadequate in substance
25

1. Notice of charge: mere general description, no detailed narration of


facts and circumstances as basis of charge
2. Notice of termination: did not specify details surrounding offenses and
specific company rules/Labor code provisions violated
36.

Century Canning Corp v Ramil GR No. 171630 (2010)


CENTURY CANNING V. RAMIL

FACTS:
Petitioner Century Canning Corporationemployed respondent Vicente Randy
Ramil in August 1993 as technical specialist. Prior to his dismissal on May 20,
1999, his job included, among others, the preparation of the purchase requisition
(PR) forms and capital expenditure (CAPEX) forms, as well as the coordination
with the purchasing department regarding technical inquiries on needed products
and services of petitioner's different departments.
On March 3, 1999, respondent prepared a CAPEX form for external fax modems
and terminal server, per order of Technical Operations Manager Jaime Garcia, Jr.
and endorsed it to Marivic Villanueva, Secretary of Executive Vice-President
Ricardo T. Po, for the latter's signature. The CAPEX form, however, did not have
the complete details and some required signatures. The following day, March 4,
1999, with the form apparently signed by Po, respondent transmitted it to
Purchasing Officer Lorena Paz in Taguig Main Office. Paz processed the paper
and found that some details in the CAPEX form were left blank. She also doubted
the genuineness of the signature of Po, as appearing in the form. Paz then
transmitted the CAPEX form to Purchasing Manager Virgie Garcia and informed
her of the questionable signature of Po. Consequently, the request for the
equipment was put on hold due to Po's forged signature. However, due to the
urgency of purchasing badly needed equipment, respondent was ordered to make
another CAPEX form, which was immediately transmitted to the Purchasing
Department.
Suspecting him to have committed forgery, respondent was asked to explain in
writing the events surrounding the incident. He vehemently denied any
participation in the alleged forgery. Respondent was, thereafter, suspended on
April 21, 1999. Subsequently, he received a Notice of Termination from Armando
C. Ronquillo, on May 20, 1999, for loss of trust and confidence.
ISSUE: Whether or not Ramil has breached the trust and confidence of his
employer sufficient to warrant his dismissal
HELD: NO. Dismissal is invalid.
RATIO:
The record of the case is bereft of evidence that would clearly establish Ramil's
involvement in the forgery. They did not even submit any affidavit of witness or
present any during the hearing to substantiate their claim against Ramil.
Ramil alleged in his position paper that after preparing the CAPEX form on
March 3, 1999, he endorsed it to Marivic Villanueva for the signature of the
Executive Vice-President Ricardo T. Po. The next day, March 4, 1999, respondent

received the CAPEX form containing the signature of Po. Petitioner never
controverted these allegations in the proceedings before the NLRC and the CA
despite its opportunity to do so.
Thus, if Ramil retrieved the form on March 4, 1999 with the signature of Po, it
can be correctly inferred that he is not the forger. Had the CAPEX form been
returned toRamil without Po's signature, Villanueva or any officer of the
petitioner's company could have readily noticed the lack of signature, and could
have easily attested that the form was unsigned when it was released to
respondent Ramil. TIEHDC
Further, as correctly found by the NLRC in its original decision dated August 26,
2002, if respondent was the one who forged the signature of Po in the CAPEX
form, there was no need for him to endorse the same to Villanueva and transmit
it the next day. He could have easily forged the signature of Po on the same day
that he prepared the CAPEX form and submitted it on the very same day to
petitioner's main office without passing through any officer of petitioner.
ISSUE: Whether or not Century Canning fulfilled the burden of proving that the
dismissal was for a valid of authorized cause
HELD: No. It failed to fulfill the burden.
RATIO: The burden of proving the validity of the termination of employment
rests with the employer. Failure to discharge this evidentiary burden would
necessarily mean that the dismissal was not justified and, therefore, illegal.
Unsubstantiated suspicions, accusations, and conclusions of employers do not
provide for legal justification for dismissing employees. In case of doubt, such
cases should be resolved in favor of labor, pursuant to the social justice policy of
labor laws and the Constitution.
The termination letter addressed to respondent, dated May 20, 1999, provides
that:
We also conducted inquiries from persons concerned to get
more information in (sic) this forgery. Some of your
statements do not jibe with theirs. . . .
However, this information which petitioner allegedly obtained from the "persons
concerned" was not backed-up by any affidavit or proof. Petitioner did not even
bother to name these resource persons.
Petitioner based respondent's dismissal on its unsubstantiated suspicions and
conclusion that since respondent was the custodian and the one who prepared the
CAPEX forms, he had the motive to commit the forgery. However, as correctly
found by the NLRC in its original Decision, respondent would not be benefited by
the purchase of the subject equipment. The equipment would be for the use of
petitioner company.
ISSUE: Whether or not mere existence of basis for believing that the employee
has breached the trust of the employer is sufficient to warrant a dismissal.
HELD: YES. But the basis must be clearly and convincingly established.
RATIO:
26

Employers are allowed a wider latitude of discretion in terminating the services


of employees who perform functions which by their nature require the employers'
full trust and confidence and the mere existence of basis for believing that the
employee has breached the trust of the employer is sufficient, this does not mean
that the said basis may be arbitrary and unfounded. DACTS
The right of an employer to dismiss an employee on the ground that it has lost its
trust and confidence in him must not be exercised arbitrarily and without just
cause. Loss of trust and confidence, to be a valid cause for dismissal, must be
based on a willful breach of trust and founded on clearly established facts. The
basis for the dismissal must be clearly and convincingly established, but proof
beyond reasonable doubt is not necessary. It must rest on substantial grounds
and not on the employer's arbitrariness, whim, caprice or suspicion; otherwise,
the employee would eternally remain at the mercy of the employer.
There is neither direct evidence nor substantial documentary evidence pointing to
respondent as the one liable for the forgery of the signature of Po.
ISSUE: Whether or not respondents tardiness in reporting for work can be a
valid ground for dismissal
HELD: NO. Cannot be a valid ground for dismissal.
RATIO:
The correct rule has always been that such previous offense of tardiness tardiness
in reporting for work may be used as valid justification for dismissal from work
only if the infractions are related to the subsequent offense upon which the basis
of termination is decreed. His previous offenses were entirely separate and
distinct from his latest alleged infraction of forgery. Hence, the same could no
longer be utilized as an added justification for his dismissal. IEAHca

pay for the two (2) days, as included in the next payroll covering the period of
December 16-31, 1993.
It is a company rule of respondent Jardine that an OT authorization slip must
pertain to only one (1) date when the overtime was rendered. The OT
authorization slip must also contain the following instructions: (1) that the
department supervisor must forward the OT authorization slip to the guard on
duty not later than 5:00 oclock in the afternoon of the working day before the
authorized overtime; (2) that the guard on duty in turn is required to transmit the
OT authorization slip to the HRD not later than 9:00 oclock in the morning of the
following day; and (3) that no payment for OT work may be made unless the OT
authorization slip is properly accomplished.
On December 18, 1993, Yolanda S. Carreon, HRD Clerk, together with Amelia F.
Castillo, HRD Assistant, in the course of preparing and post-auditing payroll
payments, noticed some irregularities in the overtime slips, dated December 6
and December 14, 1993, submitted by petitioner.
The OT authorization slip, dated December 6, 1993, covered alleged overtime
work on six (6) days, November 16, 17, 18, 22, 23, and 24, 1993, as appearing from
the entries under the headings "Overtime Date" and "Actual Time". Said slip was
prepared only on December 6, 1993 and signed by the security guard on
December 7, 1993, or long after the stated days on which petitioner had
supposedly worked overtime. The OT authorization slip dated December 14, 1993
appeared to have been tampered with. Robles, petitioners immediate supervisor,
consistently denied having signed and approved petitioners irregular OT
authorization slips. He maintained that he did not authorize petitioner to render
overtime work on those questioned dates and that petitioners OT authorization
slip dated December 14, 1993 had erasures which do not bear his initials.

Besides, respondent had already been sanctioned for his prior infractions. To
consider these offenses as justification for his dismissal would be penalizing
respondent twice for the same offense.

On April 4, 1994, petitioner was terminated from employment for violation of


Rule 32 of the Company Rules and Regulations penalizing with dismissal, the
offense of "falsification of personnel, medical and other company records" in
pursuit of personal gain.

37.

ISSUE: WON the petitioner was illegally dismissed.

Ramoran v Jrdine CMG 326 SCRA 208 (2000)

VIRGINIA G. RAMORAN,
COMPANY, INC., (2000)

vs.

JARDINE

CMG

LIFE

INSURANCE

DE LEON, JR., J.:


FACTS: Petitioner Ramoran started working with Jardine as an accounting
clerk. She rose thru the ranks and held the position of junior accountant in 1994.
Antonio Robles, then Manager of the Accounting Department of respondent
Jardine, was her immediate supervisor. The Human Resources Development
(hereafter, "HRD") of Jardine, received from petitioner an overtime (OT)
authorization slip dated December 6, 1993, covering her alleged overtime work on
November 16, 17, 18, 22, 23 and 24, 1993. Jardine paid petitioner overtime pay
for the said days as included in the payroll for the period of December 1-15,1993.
On December 15, 1993, the HRD received an OT authorization slip dated
December 14, 1993, prepared by petitioner, covering the overtime work allegedly
rendered by her on December 13 and 14, 1993. The HRD paid petitioner overtime

HELD: The dismissal was proper.


RATIO: The essence of due process is to be found in the reasonable opportunity
to be heard and submit any evidence one may have in support of ones defense.
Due process does not necessarily require conducting an actual hearing but simply
giving the party concerned due notice and affording an opportunity or right to be
heard.
In the instant case, petitioner was apprised of the charges against her. During the
administrative investigation scheduled on February 1 and 8, 1994, petitioner
attended and was given an opportunity to give her side. She consented to
resorting to voluntary arbitration and participated in the selection of arbitrators.
The labor union, of which petitioner is a member and which represented the
petitioner, nominated Atty. Aranzamendez; respondent Jardine nominated Atty.
Jimenez, and these two arbitrators chose Atty. Martinez as the chairman of the
Panel of Voluntary Arbitrators. Petitioner submitted herself to the jurisdiction of
27

the Panel of Voluntary Arbitrators, by presenting her evidence and sought


affirmative relief therein; hence, she cannot now validly question the latters
jurisdiction. It is an undesirable practice for a party, after encountering an
adverse judgment, to complain and question the proceedings where she had
submitted her own evidence and claimed affirmative relief.
There is no basis to mitigate petitioners liability inasmuch as this is not the first
time that petitioner was charged with falsification by respondent Jardine. She
had in fact earlier served a five (5)-day suspension from November 8-13, 1993 for
admittedly tampering with the entries in Official Receipt No. 1013 issued by
Limited Vision Center on July 7, 1993, in support of her application for the optical
loan she had obtained from respondent Jardine. Petitioner made it appear in the
said official receipt that she had paid P6,980.00 for two (2) pairs of Rodenstoch
Eyeglasses when the truth was, as admitted by her, the official receipts pertained
to her purchase of contact lens solution from Limited Vision Center, for which she
only paid P100.00. Under its company rules, respondent Jardine was, strictly
speaking, entitled to dismiss petitioner on that ground. However, due to
petitioners voluntary admission of the offense and for humanitarian reasons, she
was only given a five (5)-day suspension. In a memorandum, respondent Jardine
warned the petitioner that "any future violation of the same nature, irrespective
of the time frame [sic] that it is repeated or committed, would result in the
imposition of the maximum penalty of dismissal." As aptly remarked by the
appellate court "the seeds of mistrust had been sown, awaiting only the proper
occasion for it to grow and fester." Petitioner, thus, had only herself to blame
when she was finally dismissed, for cause, by respondent Jardine for the reason
that she falsified her subject overtime authorization slips dated December 6 and
14, 1993 in violation of the Company Rules and Regulations.
38.

Gustllo v Wyeth Phil Inc 440 SCRA 67 (2004)

Alan Gustilo v. Wyeth Philippines Inc., Filemon Verzano, Jr. Aurelio Mercado,
and Edgar Epilepsia
October 4, 2004
Facts:
Alan D. Gustilo was employed by Wyeth Philippines, Inc., as a
pharmaceutical territory manager in Metro Bacolod City and Negros
Occidental. He performed various functions, such as visiting hospitals,
pharmacies, drugstores and physicians concerned; and submitting
periodic reports of his daily call visits, monthly itinerary, and weekly
locator and incurred expenses.
Petitioners employment records show that company, on various dates,
reprimanded and suspended him for habitually neglecting to submit his
periodic reports.
Petitioner submitted to respondent company a plan of action dated February 6,
1996 where he committed to make an average of 18 daily calls to physicians;
submit promptly all periodic reports; and ensure 95% territory program
performance for every cycle.

However, petitioner failed to achieve the above objectives, prompting


respondent company to send him two (2) separate notices dated, charging him
with willful violation of company rules and regulations and directing him to
submit a written explanation.
In his explanation, petitioner stated that he was overworked and an object of
reprisal by his immediate supervisor.
Respondent company terminated the services of petitioner.
LA: petitioner was illegally dismissed
NLRC affirmed with modification the decision of the LA.
CA reversed the decision of the NLRC but grated him separation pay (P106,890)
considering the mitigating factors of length of service, the loyalty awards he
received, and respondent Verzanos grudge against him. The cited the ff
violations of the company rules:
1.
2.
3.
4.
5.
6.
7.

he was suspended for falsifying, tampering and/or altering the gasoline


receipt
he was warned for false reporting of his trade outlet calls
he was guilty of unauthorized availment of sick leaves, emergency
leaves, vacation leaves and unauthorized absences
he was cited for his repeated delay in submitting his expense reports
he was cited for failure to submit his expense report on time
he was charged with breach of the rule on submission of required reports
he was cited for unauthorized absence on October 19, 1995 and other
violations of company rules

Issue #1: WON Petitioner was illegally dismissed?


In Family Planning Organization of the Philippines, Inc. vs. NLRC, we held: It is
the employer's prerogative to prescribe reasonable rules and regulations
necessary or proper for the conduct of its business or concern, to provide certain
disciplinary measures to implement said rules and to assure that the same be
complied with. At the same time, it is one of the fundamental duties of the
employee to yield obedience to all reasonable rules, orders, and instructions of the
employer, and willful or intentional disobedience thereof, as a general rule,
justifies rescission of the contract of service and the preemptory dismissal of the
employee."
Records show the various violations of respondent companys rules and
regulations committed by petitioner. His dismissal from the service is, therefore,
in order. Indeed, in Piedad vs. Lanao del Norte Electric Cooperative, Inc., we
ruled that a series of irregularities when put together may constitute
serious misconduct, which under Article 282 of the Labor Code is a just cause
for dismissal.

28

But the Court of Appeals still awarded him separation pay of P106,890.00 by
reason of several mitigating factors mentioned in its assailed Decision.
Issue #2: The issue for our determination now is whether he is entitled to
such an award.
Financial assistance may be allowed as a measure of social justice in
exceptional circumstances and as an equitable concession. Financial
assistance is allowed only in those instances where the employee is validly
dismissed for causes other than serious misconduct or those reflecting
on his moral character (Zenco Sales, Inc. vs. National Labor Relations
Commission, 234 SCRA 689).
In the case at bar, we find no exceptional circumstances to warrant the
grant of financial assistance or separation pay to petitioner. Petitioner
falsified his employment application form by not stating therein that he is
the nephew of Mr. Danao, Wyeths Nutritional Territory Manager. Also,
he was guilty of falsifying a gasoline receipt, unauthorized availment of sick,
vacation and emergency leaves.
Neither can petitioner find reliance on the policy of social justice. T]hose who
invoke social justice may do so only if their hands are clean and their motives
blameless (PLDT v. NLRC)
Petitioner was legally dismissed from employment and is, therefore, not entitled
to reinstatement or an award of separation pay or other benefits. Unfortunately,
respondent company did not interpose an appeal to this Court. Hence, no
affirmative relief can be extended to it. A party in a case who did not appeal
is not entitled to any affirmative relief. Thus, respondent company has to comply
with the Appellate Courts mandate to grant petitioner his separation pay.
WHEREFORE, the petition is DENIED. Costs against petitioner.
39.

Santos v NLRC 287 SCRA 117 (1998)

Jose Santos Jr. v. NLRC, Hagonoy Institute, Directress, and Principal


(1998).
Jose Santos Jr, married and a teacher at Hagonoy Institute started an affair with
Mrs. Martin, a co-teacher and likewise married to another man. Rumors of the
affair spread among faculty and staff. Mrs. Martin was asked by officials to take a
leave, which she ignored. She was subsequently dismissed. Mrs. Martin filed a
separate case for illegal dismissal which was upheld by the NLRC for lack of due
process. Meanwhile, an internal investigation concluded that there was in fact an
affair. Jose Santos Jr. was given the opportunity to air his side, then dismissed.
He also filed complaint for illegal dismissal.

Law / Rules Art 282 + Manual of Regulations for Private Schools which
provided for Disgraceful or Immoral Conduct as a grounds for termination.
DEFENSE: He did not live up to the moral conduct expected of teachers.
SC: VALID DISMISSAL. RE Immorality -- To constitute immorality, the
circumstances of each particular case must be holistically considered and
evaluated in the light of prevailing norms of conduct and the applicable law. (1)
Extramarital affair is an affront to sanctity of marriage (2) Teachers stand in loco
parentis. They are held to high standards of integrity and honesty because of
their role in formation of children. (3) IN THIS CASE, testimonies by student,
security guard, janitor, and six co-teachers established that there was an affair.
(4) The fact that Mrs. Martin received a letter of support from the faculty does not
redound to the benefit of Jose Santos Jr., neither does the verdict in favor which
was grounded on the fact that her dismissal was illegal for lack of due process.
Unlike her, he was afforded due process through a fair administrative process.
COMPARE WITH:
Evelyn Chua Qua v. Hon. Jacobo Clave and Tay Tung High School (1990)
In this case, teacher Evelyn Chua Qua (30 years old) married her student Bobby
Qua (16 years old) who she first met when he was in the 6th grade. Consent was
given by Quas mother and marriage was solemnized according to law. She was
dismissed by the school For abusive and unethical conduct unbecoming of a
dignified school teacher and that her continued employment is inimical to the
best interest, and would downgrade the high moral values, of the school.
SC: (1) there was no denial of her due process in this case (2) BUT, there was also
no competent evidence of immoral acts leading up to the marriage, just that they
were seen in classroom together which is normal between a student and his
teacher. The fact of marriage was itself not immoral. If the two eventually fell in
love, despite the disparity in their ages and academic levels, this only lends
substance to the truism that the heart has reasons of its own which reason does
not know.
40.

Chua-Qua v Clave, G.R. No. 49549

41.

Bughaw v Treasure lsaland Industrial, GR No. 173151 (2008)

EDUARDO BUGHAW V TREASURE ISLAND INDUSTRIAL


FACTS:
-Eduardo Bughaw (Bughaw) was employed as production worker by Treasure
Island Industrial (TII), respondent.
-Erlito Loberanes (Loberanes), an employee of TII was caught in flagrante delicto
by the police officers while in possession of shabu.
-In the course of police investigation, Loberanes admitted the commission of the
crime and implicated petitioner, Bughaw, by stating that part of the money used
for buying the illegal drugs was given by Bughaw, and the illegal drugs
purchased were for their consumption for the rest of the month.

29

-TII sent a memo to Bughaw. The memo contains the ff: (1) notice of the 30-day
preventive suspension (2) An instruction requiring him to explain within 120
hours why no disciplinary action should be imposed against him for his alleged
involvement in illegal drug activities. (3) An instruction requiring him to appear
at the office of respondent's legal counsel for the hearing on the matter.

process is not violated where one is given the opportunity to be heard but he
chooses not to explain his side

-Bughaw failed to appear before the TII's legal counsel on the scheduled hearing
date.

HELD:

-TII sent a second letter to petitioner directing him to attend another


administrative hearing but petitioner once again failed to show up.
-In a third letter addressed to Bughaw, TII terminated the latter's employment
for using illegal drugs within company premises during working hours, and for
refusal to attend the administrative hearing and submit written explanation on
the charges hurled against him.
-Thereafter, Bughaw filed a complaint for illegal dismissal against TII and its
President, Emmanuel Ong, before the Labor Arbiter. He argues that:
-He had been working for the respondent for 15 years and he was very
conscientious with his job.
-He was suspended for 30 days on the unfounded allegation of his coworker that he used illegal drugs within company premises.
-When he reported back to work after the expiration of his suspension, he
was no longer allowed by respondent to enter the work premises and was
told not to report back to work.
-LA: Rendered a Decision in favor of Bughaw based on the ff:
(1) TII failed to present substantial evidence to establish the charge
leveled against the Bughaw. Apart from Loberanes's statements on
petitioner's alleged illegal drug use, no other corroborating proof was
offered by respondent to justify petitioner's dismissal.
(2)TII failed to comply with due process when it immediately suspended
petitioner and eventually dismissed him from employment. Bughaws
immediate suspension was not justified since no evidence was submitted
by the TII to establish that Bughaws continued employment pending
investigation poses a serious and imminent threat to respondent's life or
property or to the life or property of petitioner's co-workers.
(3)The notices of hearing sent by TII to Bughaw were not duly received
by the latter.
-NLRC: Affirmed the Labor Arbiter's Decision.
-CA: Reversed the Decisions of the Labor Arbiter and NLRC on the grounds of
patent misappreciation of evidence and misapplication of law. CA found that
Bughaw was afforded the opportunity to explain and defend himself from the
accusations against him when TTI gave him notices of hearing. The essence of
due process in administrative proceedings is simply an opportunity to explain
one's side or to seek reconsideration of the action or ruling complained of. Due

ISSUE:
WON Bughaw was illegally dimissed.

No, but Bughaw is entitled to nominal damages.


RATIONALE:
- ART 282 provides:
An employer may terminate an employment for any of the following
causes:
(a) Serious misconduct or willful disobedience by the employee of
the lawful orders of his employer or representative in connection
with
his
work;
(b) Gross and habitual neglect by the employee of his duties;
(c) Fraud or willful breach by the employee of the trust reposed in him by
his
employer
or
his
duly
authorized
representative;
(d) Commission of a crime or offense by the employee against the person
of his employer or any immediate member of his family or his duly
authorized
representative;
and
(e) Other causes analogous to the foregoing.
-The charge of drug abuse inside the company's premises and during
working hours against Bughaw constitutes serious misconduct, which is
one of the just causes for termination. Misconduct is improper or wrong
conduct. It is the transgression of some established and definite rule of action, a
forbidden act, a dereliction of duty, wilful in character, and implies
wrongful intent and not merely an error in judgment. The misconduct to be
serious within the meaning of the Act must be of such a grave and aggravated
character and not merely trivial or unimportant. Such misconduct, however
serious, must nevertheless, in connection with the work of the employee,
constitute just cause for his separation. This Court took judicial notice of
scientific findings that drug abuse can damage the mental faculties of
the user. It is beyond question therefore that any employee under the
influence of drugs cannot possibly continue doing his duties without
posing a serious threat to the lives and property of his co-workers and
even his employer.
- In administrative proceedings, technical rules of procedure and evidence are not
strictly applied and administrative due process cannot be fully equated with due
process in its strict judicial sense. Hence, the statement of Loberanes that
Bughaw was a shabu user is substantial evidence which the latter failed
30

to refute. Records show that the first and second letters addressed to Bughaw
were duly received by him.
- However, not only must the dismissal be for a just or authorized cause, the
rudimentary requirements of due process - notice and hearing - must, likewise, be
observed before an employee may be dismissed. There must also be observance of
the requirements of due process, otherwise known as the two-notice rule.
-The first notice, which may be considered as the proper charge, serves to apprise
the employee of the particular acts or omissions for which his dismissal is sought.
The second notice on the other hand seeks to inform the employee of the
employer's decision to dismiss him. This decision, however, must come only after
the employee is given a reasonable period from receipt of the first notice within
which to answer the charge and ample opportunity to be heard and defend
himself with the assistance of a representative if he so desires. This is in
consonance with the express provision of the law on the protection to labor and
the broader dictates of procedural due process. Non-compliance therewith is fatal
because these requirements are conditions sine qua non before dismissal may be
validly effected.

*Digest not mine, taken from Working Conditions section.


Facts

Private respondents (Cristina Rodiel, Jesus Paracale, Romeo Tee,


Benjamin Lakandula, Avelino Acha, Ignacio Dela Cerna and Guillermo
Domegillo) were employees of PT & T Corp., assigned in different
branches all over the country.

In 1997, PT & T Corp. adopted its Relocation and Restructuring


Program designed to sustain its retail operations; decongest surplus
workforce in some branches; promote efficiency and productivity; lower
expenses incidental to hiring and training new personnel; and avoid
retrenchment of employees occupying redundant positions.

Private respondents were among the employees affected by the said


program. They were informed of their relocation and directed to
report to their respective relocation assignments. Moreover, the
employees who would agree to the transfer would be considered
promoted, thus:

-There is no dispute that TII fully complied with the first-notice requirement
apprising Bughaw of the cause of his impending termination and giving him the
opportunity to explain his side, however, we find that it failed to satisfy the need
for a second notice informing petitioner that he was being dismissed from
employment.
- The law mandates that it is incumbent upon the employer to prove the validity
of the termination of employment. The burden therefore is on TII to present clear
and unmistakable proof that Bughaw was duly served a copy of the notice of
termination but he refused receipt. There is nothing on record that would indicate
that respondent even attempted to serve or tender the notice of termination to
Bughaw.
-The violation of the petitioners' right to statutory due process by the private
respondent warrants the payment of indemnity in the form of nominal damages.
(cited Agabon v. NLRC) The amount of such damages is addressed to the sound
discretion of the court, taking into account the relevant circumstances. Tthis form
of damages would serve to deter employers from future violations of the statutory
due process rights of employees. At the very least, it provides a vindication or
recognition of this fundamental right granted to the latter under the Labor Code
and its Implementing Rules.
-Conformably,

the award of backwages by the Labor Arbiter and the NLRC should
be deleted and, instead, Bughaw should be indemnified in the amount of
P30,000.00 as nominal damages.
42.
Philippine Telegraph and Telephone Company vs. NLRC, 272
SCRA 596, May 23, 1997
Philippine Telegraph & Telephone Corp. (PT&T Corp.) vs. Court of
Appeals, et al.
G.R. No. 152057; 29 Sept. 2003; Callejo, Sr., J.

Name

From
Position/Job
Grade

Work
Location
(from)

To
Position/Jo
b Grade

New
location

work

Acha

Jr.
Counter/JG
2

Legaspi

Courier/JG
3

Romblon/Odiong
an

Rodel

Jr. Counter
Clerk/JG2

Cabanatua
n

Clerk/JG4

Baguio

Dela
Cerna

Jr.
CW
Operator/JG
2

Cotabato
City

Clerk/JG4

Kidapawan

Demigillo

Jr.
CW
Operator/JG
2

Midsayap
North

Courier/JG
3

Lebak

Lakandul
a

Counter/JG
3

Iligan

Clerk/JG4

Butuan

Paracale

Jr.
CW
Operator/JG
2

Makar,
Gen.
Santos

Clerk/JG4

Butuan

Tee

TTY
OperatorGen/JG4

Zamboang
a City

Clerk/JG4

Jolo

Private respondents rejected the offer. Dissatisfied with their


explanation for refusing the offer, PT&T Corp. considered their
31

he has a right to refuse, and the exercise of this right cannot be


considered in law as insubordination or willful disobedience of a lawful
order of the employer.

refusal as insubordination and willful disobedience to a lawful


order; hence, they were dismissed from work.

Private respondents bargaining agent, PT&T Workers Union-NAFLUKMU filed a complaint against PT&T Corp. for illegal dismissal and
unfair labor practice for and in behalf of the dismissed employees before
the arbitration branch of the NLRC.

The contention of the private respondents is that their refusal to


transfer could not possibly give rise to a valid dismissal on the ground of
willful disobedience, as their transfer was prejudicial and
inconvenient; thus unreasonable. Moreover, since their transfer
resulted in their promotion, they had the right to refuse or decline the
positions being offered them. It follows that the refusal to accept the
transfer could not have amounted to insubordination or willful
disobedience to lawful orders.

For its part, PT&T Corp. maintains that the transfers were made in
lawful exercise of its management prerogative and were done in good
faith. The transfers were aimed at decongesting surplus employees and
detailing them to a more demanding branch.

Labor Arbiter dismissed the complaint of the private respondents;

NLRC reversed the LA and declared that there was illegal dismissal;

CA dismissed the petition for certiorari filed by PT&T and affirmed the
decision of the NLRC.

43.

Star Paper Corporation vs. Simbol, 487 SCRA 228, April 12, 2006

Star Paper v. Simbol


Facts:
According to the respondents, Simbol and Comia allege that they did not resign
voluntarily; they were compelled to resign in view of an illegal company policy. As
to respondent Estrella, she alleges that she had a relationship with co-worker
Zuiga who misrepresented himself as a married but separated man. After he got
her pregnant, she discovered that he was not separated. Thus, she severed her
relationship with him to avoid dismissal due to the company policy. On November
30, 1999, she met an accident and was advised by the doctor at the Orthopedic
Hospital to recuperate for twenty-one (21) days. She returned to work on
December 21, 1999 but she found out that her name was on-hold at the gate. She
was denied entry. She was directed to proceed to the personnel office where one of
the staff handed her a memorandum. The memorandum stated that she was
being dismissed for immoral conduct. She refused to sign the memorandum
because she was on leave for twenty-one (21) days and has not been given a
chance to explain. The management asked her to write an explanation. However,
after submission of the explanation, she was nonetheless dismissed by the
company. Due to her urgent need for money, she later submitted a letter of
resignation in exchange for her thirteenth month pay.

Issue/Holding/Ratio
WON the private respondents were illegally dismissed Yes. The dismissal is
not based on any of the just or valid grounds under Art. 282 of the Labor
Code, the NLRC correctly ordered the private respondents reinstatement
without loss of seniority rights and the payment of backwages from the time of
their dismissal up to their actual reinstatement.

As maintained by PT&T Corp. when the private respondents were


transferred they were also promoted.

An employee cannot be promoted without his consent. A transfer


that results in promotion or demotion, advancement or
reduction or a transfer that aims to lure the employee away
from his permanent position cannot be done without the
employees consent.

A promotion is the advancement from one position to another with an


increase in duties and responsibilities as authorized by law, and usually
accompanied by an increase in salary. The indispensable element of
promotion is the advancement from one position to another or an
upward vertical movement of the employees rank or position. Any
increase in salary should only be considered incidental.

Considering further that a promotion is in the nature of a gift or reward,


an employee cannot be compelled to accept a promotion. In other words,

Respondents later filed a complaint for unfair labor practice, constructive


dismissal, separation pay and attorneys fees. They averred that the
aforementioned company policy is illegal and contravenes Article 136 of the Labor
Code.
Issue:
WON there was just cause in the dismissal of the concerned parties.
Held:
No, just cause was not proven.
The Labor Code is the most comprehensive piece of legislation protecting labor.
The case at bar involves Article 136 of the Labor Code which provides:
Art. 136. It shall be unlawful for an employer to require as a condition of
employment or continuation of employment that a woman employee shall not get
married, or to stipulate expressly or tacitly that upon getting married a woman
employee shall be deemed resigned or separated, or to actually dismiss, discharge,
discriminate or otherwise prejudice a woman employee merely by reason of her
marriage.
In denying the contention of the petitioner company, the SC applied the two
factors to justify a bona fide occupational qualification:
32

Since the finding of a bona fide occupational qualification justifies an employers


no-spouse rule, the exception is interpreted strictly and narrowly. There must be a
compelling business necessity for which no alternative exists other than the
discriminatory practice. To justify a bona fide occupational qualification, the
employer must prove two factors: (1) that the employment qualification is
reasonably related to the essential operation of the job involved; and, (2) that there
is a factual basis for believing that all or substantially all persons meeting the
qualification would be unable to properly perform the duties of the job.
The SC does not find a reasonable business necessity in the case at bar.
It is significant to note that in the case at bar, respondents were hired after they
were found fit for the job, but were asked to resign when they married a coemployee. Petitioners failed to show how the marriage of Simbol, then a Sheeting
Machine Operator, to Alma Dayrit, then an employee of the Repacking Section,
could be detrimental to its business operations. Neither did petitioners explain
how this detriment will happen in the case of Wilfreda Comia, then a Production
Helper in the Selecting Department, who married Howard Comia, then a helper
in the cutter-machine. The policy is premised on the mere fear that employees
married to each other will be less efficient.
The failure of petitioners to prove a legitimate business concern in imposing the
questioned policy cannot prejudice the employees right to be free from arbitrary
discrimination based upon stereotypes of married persons working together in
one company. Decision of the CA affirmed.
*Digest culled from Scribd. Cross checked with case, facts and doctrine are sound.
Credit to the owner.
44.

Yrasuegui vs. Philippine Airlines, Inc., 569 SCRA 467

Yrasuegui vs. PAL


REYES, R.T., J.
Facts:

Yrasuegui was an international flight steward for PAL. He stands 58


and as mandated by the Cabin and Crew Administration Manual of PAL,
his ideal weight is 166 pounds. He has a weight problem (SCs words not
mine). PAL advised him to go on extended leaves to address his weight
concerns.
o December 1984 to March 1985
o March 1985 to November 1985
o October 1988 February 1989
By April 1989, he was 43 pounds over his ideal weight. He was removed
from flight duty and was formally requested to trim down and report for
weight checks. He was told that he may avail of the services of the
company physician. During weight checks, he was found to have gained 8

pounds. He made a written commitment to PAL that he will meet his


weight requirement.
He failed to report for weight checks several time and was formally
warned that a repeated refusal to report for weight check would be dealt
with accordingly.
By 1992, he was still at 205 pounds. PAL finally served petitioner a
Notice of Administrative Charge for violation of company standards on
weight requirements. He did not deny being overweight. There was also
a clarificatory hearing where he manifested that he was undergoing a
weight reduction program.
In 1993, PAL terminated his employment.
He filed a complaint for illegal dismissal.

LA: Found for Yrasuegui. The weight standards of PAL are reasonable in view of
the nature of the job but the weight did not hamper the performance of his duties.
Assuming that it did, he could have been reassigned. Other overweight employee
instead of being terminated were in fact promoted.
NLRC: Upheld the LA ruling. Obesity, or the tendency to gain weight
uncontrollably regardless of the amount of food intake, is a disease in itself. As a
consequence, there can be no intentional defiance or serious misconduct by
petitioner to the lawful order of PAL for him to lose weight.
CA: Found for PAL. The NLRC looked at wrong and irrelevant considerations.
The weight standards of PAL are meant to be a continuing qualification for an
employees position. The failure to adhere to the weight standards is an analogous
cause for the dismissal of an employee under Article 282(e) of the Labor Code in
relation to Article 282(a). The relevant question to ask is not one of willfulness
but one of reasonableness of the standard and whether or not the employee
qualifies or continues to qualify under this standard.
Issue: Whether or not dismissal was valid. Yes.
The obesity of petitioner is a ground for dismissal under Article 282(e) [44] of the
Labor Code.
The weight standards of PAL constitutes a continuing qualification of an
employee in order to keep the job. An employee may be dismissed the moment he
is unable to comply with his ideal weight as prescribed by the weight standards.
The dismissal of the employee would thus fall under Article 282(e) of the Labor
Code (other causes analogous to the foregoing). By its nature, these qualifying
standards are norms that apply prior to and after an employee is hired. They
apply prior to employment because these are the standards a job applicant must
initially meet in order to be hired. They apply after hiring because an employee
must continue to meet these standards while on the job in order to keep his job.
The SC does not agree, contrary to Yrasueguis argument, that obesity is a
disease. That he was able to reduce his weight during his extended leaves clearly
show that it is possible for him to lose weight given the proper attitude,
determination, and self-discipline. During the clarificatory hearing, Yrasuegui
even claimed that he can bring his weight down. His fluctuating weight indicates
absence of willpower rather than an illness.
33

The dismissal of Yrasuegi can be predicated on the bona fide occupational


qualification defense.
Employment in particular jobs may not be limited to persons of a particular sex,
religion, or national origin unless the employer can show that sex, religion, or
national origin is an actual qualification for performing the job. The qualification
is called a bona fide occupational qualification (BFOQ).
There are provisions similar to BFOQ in our laws:

CONSTITUTION (1987), Art. XIII, Sec. 3.


Labor Code, ART. 3. Declaration of Basic Policy
RA 7277, SEC. 32. Discrimination of Employment.

The test of reasonableness of the company policy is used because it is parallel to


BFOQ. BFOQ is valid provided it reflects an inherent quality reasonably
necessary for satisfactory job performance. In Star Paper Corporation v. Simbol,
the SC held that in order to justify a BFOQ, the employer must prove that
(1) The employment qualification is reasonably related to the essential
operation of the job involved; and
(2) That there is factual basis for believing that all or substantially all
persons meeting the qualification would be unable to properly perform
the duties of the job
The weight standards of PAL show its effort to comply with the exacting
obligations imposed upon it by law by virtue of being a common carrier. In other
words, the primary objective of PAL in the imposition of the weight standards for
cabin crew is flight safety. The most important activity of the cabin crew is to care
for the safety of passengers and the evacuation of the aircraft when an emergency
occurs. On board an aircraft, the body weight and size of a cabin attendant are
important factors to consider in case of emergency.
The biggest problem with an overweight cabin attendant is the possibility of
impeding passengers from evacuating the aircraft, should the occasion call for it.
Being overweight necessarily impedes mobility. Indeed, in an emergency
situation, seconds are what cabin attendants are dealing with, not minutes. Three
lost seconds can translate into three lost lives. Evacuation might slow down just
because a wide-bodied cabin attendant is blocking the narrow aisles. These
possibilities are not remote.
45.
St. Luke's Medical
(SLMCEA-AFW) v NLRC

Center

Employee's

Association-AFW

ST. LUKE'S MEDICAL CENTER EMPLOYEE'S ASSOCIATION- AFW vs


NLRC (SAME LANG TO SA LAST TIME NA DIGEST)
FACTS
Maribel S. Santos is an X-ray technician at the Radiology Dept. of St. Luke's
Medical Center, hired in Oct. 13, 1984. On April 22, 1992: Congress passed RA
7431, "Radiologic Technology Act of 1992", which prohibits a radiology/x-ray
technologist from practicing in the Philippines without a certificate of registration
from the Board of Radiologic Technology.

Despite several notices from the HR and the Director of the Institute of
Technology, Santos never took the Board examination. In Nov. 23, 1998, she was
finally given notice of retirement in lieu of separation pay, which Santos declined.
She was then issued a Notice of Separation from the company effective Dec. 30,
1998, which also states that her qualifications do not fit any of the present vacant
positions in the hospital.
The President of Philippine Association of Radiologic Technologists (Jack Lappay)
then asked Personnel Manager of SLMC (Judith Betita) to give due consideration
to the organization's 3 regular members to be reassigned to any department in
SLMC awaiting their chance to pass the Board Exam in the future
Since Maribel didn't appeal for rechecking to the PRC for the recent board
examinations, the SLMC Personnel Manager issued again a Notice of Separation
from the Company to Santos effective Feb. 5, 1999.
Maribel Santos then filed a complaint for illegal dismissal and non-payment of
salaries, allowances and other monetary benefits against SLMC. Pending this,
the Alliance of Filipino Workers (AFW) wrote to SLMC HR Director Rita
Marasigan to accommodate Maribel as CSS Aide in the hospital since an
employee died 2 months earlier.
Marasigan replied that Santos would have to go through the usual application
period to procure the position given that she did not look for an appropriate
vacant position at the hospital within 30 days from issuance of termination
notice. A month later, Marasigan offered Santos the position of secretary in the
Dietary Department, to which Santos has not responded. SLMC's Personnel
Manager then wrote to AFW informing it that SLMC is giving Ms.Santos until
the end of the month (December) to reply otherwise the position will be taken up
by someone else.
The Labor Arbiter granted Maribel Santos P115,500 as separation pay, but
dismissed all her other claims. Santos appealed to the NLRC, which affirmed the
Labor Arbiter, hence this petition for certiorari.
ISSUE: w/n Santos was illegally dismissed
HELD: YES, Santos was illegally dismissed
RATIO:
" While the right of workers to security of tenure is guaranteed by the
Constitution, its exercise may be reasonably regulated pursuant to the police
power of the State to safeguard health, morals, peace, education, order, safety,
and the general welfare of the people. Consequently, persons who desire to
engage in the learned professions requiring scientific or technical knowledge may
be required to take an examination as a prerequisite to engaging in their chosen
careers...
The rationale behind regulating the practice of radiologic and x-ray technology is
stated in Section 2 of RA 7431:
Sec. 2. Statement of Policy. - It is the policy of the State to upgrade the practice of
radiologic technology in the Philippines for the purpose of protecting the public
34

from the hazards posed by radiation as well as to ensure safe and proper
diagnosis, treatment and research through the application of machines and/or
equipment using radiation.

2)

No malice or ill will can be imputed to SLMC as it dismissed Santos in conformity


to a statute. An employer should not be penalized for situations where it had no
participation or control. Additionally, SLMC gave Santos ample opportunity to
qualify for the position and was sufficiently warned that she would be dismissed
should she fail to comply with the requirement of having a certificate of
registration from the Board, which can only be obtained after passing the
licensure exam.
Not all labor disputes will be resolved in the favor of the workers, as management
also has rights to be protected, such as the right of the employer to conduct his
business as he wishes, barring unlawful discrimination or other legal limitations
which do not exist in this case.
46.

3)

Malig-on v Equitable General Services Inc GR No. 185269 (2010)

MALIG-ON v EQUITABLE (2010)


FACTS:
Elsa Malig-on was hired by Equitable General Services, Inc. (subcontractor) as
janitress in 1996.
According to Malig-on: Six years later, on February 15, 2002, Malig-ons
immediate supervisor told her that Equitable will assign her to another client.
This never happened. Eight months later, on Oct 15, 2002, Equitable told Maligon that she had to file a resignation letter before reassignment. Malig-on
complied but was not reassigned. Three days later, on Oct 18, Malig-on filed for
illegal dismissal.
According to Equitable: Malig-on just stopped reporting for work on Feb 16, 2002.
Equitable wrote her two letters on Aug 23, 2002 and Sept 2, 2002, asking her to
explain her absence. On Oct 15, 2002, Malig-on showed up and filed resignation.
ISSUES & RULING:
1)

WON Malig-ons resignation was freely made


NO. Employer bears the burden of proving that dismissal was legal.
When employer claims resignation by employee, burden of proof is on
employer that employee did so willingly. Circumstances surrounding
such alleged resignation must be consistent with the employees intent to
give up work.
Here, the circumstances do not sound consistent with resignation freely
made:
First, Equitable should have immediately written Malig-on upon
supposedly vanishing and asked her to explain in writing why she should
not be considered to have abandoned her job so the company may be
cleared of its responsibility as employer.
Second, if Malig-on had abandoned work, there was no reason for her to
suddenly show up after 8 months and file her resignation.

Finally, that Malig-on filed for illegal dismissal just 3 days after filing
her alleged resignation letter is inconsistent with genuine resignation.
WON Malig-on was constructively dismissed
YES. Off-detailing an employee is not equivalent to dismissal as long as
her floating status did not continue beyond a reasonable time. When
floating status ran up to more than 6 months, the employer may be
considered to have constructively dismissed the employee from work.
More than 6 months from Feb 15, 2002 is Aug 16, 2002. On that date,
Malig-on was constructively dismissed. Thus:
a) Her purported resignation on Oct 15, 2002 could not have been
legally possible
b) Alleged notices on Aug 23 and Sept 2, 2002 were given after
constructive dismissal
WON Malig-on is entitled to reinstatement
NO. Malig-on did not appear persistent in being rehired and, under the
circumstances, reinstatement would only result in a highly hostile work
environment. Separation pay should be awarded instead.

47.
Barroga v Data Center College of the Philippines GR No. 174158
(2011)
BARROGA VS. DATA CENTER COLLEGE OF THE PHILIPPINES (CELEBRADO)
William Endeliseo Barroga vs. Data Center College of the Philippines
and Wilfred Bactad
G.R. No. 174158 | June 27, 2011 | 1st Division | Del Castillo, J.
FACTS:
Barroga was employed as an Instructor by Data Center College (Laoag) and
the latter transferred him to University of Northern Philippines where it had
a tie-up program. He was informed through a letter that he would be receiving, in
addition to his monthly salary, a P1.2k allowance for board and lodging during
his stint as instructor in UNP.
He is transferred to Data Center College (Bangued) as Head for
Education/Instructor due to an urgent need for an experienced officer and
computer instructor. However, he declined citing the deteriorating health
condition of his father and the absence of additional remuneration to defray
expenses for board and lodging.
He filed for constructive dismissal against Data Center College alleging that
his proposed transfer to Abra constitutes a demotion in rank and diminution in
pay and would cause personal inconvenience and hardship. He will also be reassigned merely as an instructor, thereby relegating him from an administrative
officer to a rank-and-file employee and the elimination of his allowance for board
and lodging will result to an indirect reduction of his salary which is prohibited
by labor laws.
LA and NLRC found no constructive dismissal. CA denied reconsideration due to
procedural lapses.
ISSUE:
35

Whether or not there is constructive dismissal and diminution of benefits? [No]


HELD.
NO.
Constructive Dismissal
Constructive dismissal is quitting because continued employment is rendered
impossible, unreasonable or unlikely, or because of a demotion in rank or a
diminution of pay. It exists when there is a clear act of discrimination,
insensibility or disdain by an employer which becomes unbearable for the
employee to continue his employment.
Barroga did not deny having been designated as Head for Education in a
temporary capacity for which he cannot invoke any tenurial security. Hence, being
temporary in character, such designation is terminable at the pleasure of
respondents who made such appointment. Moreover, the schools right to transfer
him rests not only on contractual stipulation but also on jurisprudential
authorities.
LA and NLRC both relied on the condition laid down in his employment contract
that the school has the prerogative to assign him in any of its branches or tie-up
schools as the necessity demands. In any event, it is management prerogative for
employers to transfer employees on just and valid grounds such as genuine
business necessity.
His re-assignment was not tainted with bad faith. As a matter of fact, the school
displayed commiseration over the health condition of his father when they
suggested that he take an indefinite leave of absence to attend to this personal
difficulty. Also, during the time when the school directed all its administrative
officers to submit courtesy resignations, his letter of resignation was not accepted.
Diminution of Benefits
Barroga failed to present any other evidence that the school committed to provide
the additional allowance or that they were consistently granting such benefit as
to have ripened into a practice which cannot be peremptorily withdrawn.
Moreover, there is no conclusive proof that his basic salary will be reduced as it
was not shown that such allowance is part of his basic salary. Hence, there will be
no violation of the rule against diminution of pay.
48.

Nippon Housing Phil. Inc. v Leynes, GR No. 177816 (2011)

NIPPON HOUSING PHIL. INC., and/or TADASHI OTA, HOROSHI TAKADA,


YUSUHIRO KAWATA, MR. NOBOYUSHI and JOEL REYES
vs.
MAIAH ANGELA LEYNES
The bona fide suspension of the operation of a business undertaking for a period
not exceeding six (6) months, or the fulfillment by the employee of a civic duty
shall not terminate employment. In all such cases the employer shall reinstate

the employee to his former position without loss of seniority rights if he indicates
his desire to resume his work not later than one (1) month from the resumption of
operations of his employer or from his relief from the military or civic duty.
The rule is settled that "off-detailing" is not equivalent to dismissal, so long as
such status does not continue beyond a reasonable time and that it is only when
such a "floating status" lasts for more than six months that the employee may be
considered to have been constructively dismissed. A complaint for illegal
dismissal filed prior to the lapse of said six-month and/or the actual dismissal of
the employee is generally considered as prematurely filed.
FACTS:
Petitioner Nippon Housing Philippines, Inc. (NHPI) ventured into building
management. Having gained the Bay Gardens Condominium Project of the Bay
Gardens Condominium Corporation (BGCC) as its first and only building
maintenance client, petitioner NHPI hired respondent Maiah Angela Leynes for
the position of Property Manager.
During the project, respondent Leynes had a misunderstanding with Engr.
Cantuba regarding the extension of the latters working hours. Because of this,
respondent Leynes instructed the security guards to bar Engr. Cantuba from
entry into the Project.
However, Hiroshi Takada, NHPIs Vice President attributed the incident as
"simple personal differences" and directed respondent Leynes to allow Engr.
Cantuba to report back for work.
Disappointed with the foregoing management decision, respondent Leynes
submitted to NHPIs President a letter asking for an emergency leave of absence
for the supposed purpose of coordinating with her lawyer regarding her
resignation letter.
While petitioner NHPI offered the Property Manager position to Engr. Carlos
Jose as a consequence respondent Leynes signification of her intention to resign,
respondent Leynes sent another letter to NHPI's HR Head on the same day,
expressing her intention to return to work and to call off her planned resignation.
Having subsequently reported back for work and resumed performance of her
assigned functions, respondent Leynes was constrained to send out a written
protest upon being informed that a substitute has already been hired for her
position.
Respondent Leynes was further served by NHPIs Senior Manager and
JanitorialManager, with a letter from NHPI's HR Head, relieving her from her
position and directing her to report to NHPIs main office while she was on
floating status.
Claiming that her being relieved from her position without just cause and
replacement by one Carlos Jose amounted to an illegal dismissal from
employment, respondent Leynes filed a complaint for illegal dismissal.
Petitioner NHPI and its officers, on the other hand, asserted that the
managements exercise of the prerogative to put an employee on floating status
36

for a period not exceeding six months was justified in view of her threatened
resignation from her position and BGCCs request for her replacement.
The Labor Arbiter found that petitioner NHPIs act of putting respondent Leynes
on floating status was equivalent to termination from employment without just
cause and compliance with the twin requirements of notice and hearing. It also
found that NHPIs officers acted with bad faith in effecting Leynes termination.
The NLRC reversed the labor arbiter's decision. NLRC ruled that petitioner
NHPIs placement of Leynes on floating status was necessitated by the clients
contractually guaranteed right to request for her relief.
The CA, however, reversed the NLRC's decision. It held that absent any showing
that there was a bona fide suspension of petitioner NHPIs business operations,
respondent Leynes relief from her position even though requested by the client
was tantamount to a constructive dismissal. It further held that the bad faith of
petitioner NHPI and its officers is evident from the hiring of Engr. Jose as
respondent Leynes replacement prior to her being relieved from her position.
Petitioners argue that the CA erred in finding that respondent Leynes was
constructively dismissed when she was placed on floating status. Petitioners
claim that she was validly placed on floating status pursuant to Article 286 of the
Labor Code of the Philippines.
ISSUES:
Whether or not petitioners' decision to place respondent Leynes on floating status
is tantamount to constructive dismissal.
RULING:
No. Article 286 of the Labor Code (When employment not deemed terminated)
states that the bona fide suspension of the operation of a business undertaking for
a period not exceeding six (6) months, or the fulfillment by the employee of a civic
duty shall not terminate employment. In all such cases the employer shall
reinstate the employee to his former position without loss of seniority rights if he
indicates his desire to resume his work not later than one (1) month from the
resumption of operations of his employer or from his relief from the military or
civic duty.
Futhermore, the rule is settled that "off-detailing" is not equivalent to dismissal,
so long as such status does not continue beyond a reasonable time and that it is
only when such a "floating status" lasts for more than six months that the
employee may be considered to have been constructively dismissed. A complaint
for illegal dismissal filed prior to the lapse of said six-month and/or the actual
dismissal of the employee is generally considered as prematurely filed.
In the case at bar, in view of the sensitive nature of respondent Leynes position
and the critical stage of the Projects business development, petitioner NHPI was
constrained to relay the situation to BGCC which, in turn, requested the
immediate adoption of remedial measures from Takada, including the
appointment of a new Property Manager for the Project. Upon BGCCs

recommendation, petitioner NHPI consequently hired Engr. Jose as respondent


Leynes replacement. Far from being the indication of bad faith the CA construed
the same to be, these factual antecedents suggest that petitioner NHPIs
immediate hiring of Engr. Jose as the new Property Manager for the Project was
brought about by respondent Leynes own rash announcement of her intention to
resign from her position. Although she subsequently changed her mind and sent
Reyes a letter announcing the reconsideration of her planned resignation and her
intention to return to work, respondent Leynes evidently had only herself to
blame for precipitately setting in motion the events which led to petitioner
NHPIs hiring of her own replacement.
The record, moreover, shows that petitioner NHPI simply placed respondent
Leynes on floating status "until such time that another project could be secured"
for her. Traditionally invoked by security agencies when guards are temporarily
sidelined from duty while waiting to be transferred or assigned to a new post or
client, Article 286 of the Labor Code has been applied to other industries when, as
a consequence of the bona fide suspension of the operation of a business or
undertaking, an employer is constrained to put employees on floating status for a
period not exceeding six months. In brushing aside petitioner NHPI's reliance on
said provision to justify the act of putting respondent Leynes on floating status,
the CA ruled that no evidence was adduced to show that there was a bona fide
suspension of petitioner NHPIs business. What said court clearly overlooked,
however, is the fact that petitioner NHPI had belatedly ventured into building
management and, with BGCC as its only client in said undertaking, had no other
Property Manager position available to respondent Leynes.
Finally, the record shows that respondent Leynes filed the complaint for actual
illegal dismissal from which the case originated on 22 February 2002 or
immediately upon being placed on floating status as a consequence of petitioner
NHPIs hiring of a new Property Manager for the Project. Thus, there was no
constructive dismissal and that her complaint was prematurely filed.
49.
Salvador 0. Mojar, et al. v Agro Commercial Security Service
Agency
50.

Pharmacia & Upjohn v Albayda Jr GR No. 172724, Aug 23, 2010

Pharmacia and Upjohn (now Pfizer Phils Inc.) vs. Albayda


FACTS:
-

Albayda is a District Sales Manager of Pharmacia & Upjohn


Pursuant to a district territorial configuration for the new marketing and
sales direction, Albayda was informed in a memo that he will be
reassigned to northern Mindanao
He refused for reasons of personal inconvenience and dislocation from
family
The company tried to convince by saying that his skills and expertise are
needed by the dismally performing district office

37

The HR manager also met with him and indicated his entitlement to
relocation benefits and allowance; he was also offered an option to
transfer to Metro Manila but refused
Albayda viewed the scheme as a guise to terminate his employment
He was given a memo directing him to report for work within 5 days
from receipt, and for failure to do so, was issued another memo notifying
him of his dismissal for being AWOL and insubordination
He filed complaint for constructive dismissal
LA: dismissed
NLRC:
dismissed
appeal
CA: remanded to CA

ISSUE(S):
w/n the transfer from Western Visayas to Cagayan de Oro City was a valid
exercise of companys management prerogative

inconvenience or hardship that will be caused to the employee by reason of the


transfer, the disobedience is not valid.
Dismissal valid. Awarded separation pay as a measure of social justice as this
was his first infraction and considering his 22 years of services with the company.
51.

Morales v Harbor Center Port Terminal Inc. GR No. 174208 (2012)

2012 Morales v Harbor Centerport Terminal


2000, Morales was hired by HCPTI as an Accountant and Acting Finance Officer.
He was later promoted to Division Manager of the Accounting Department.
After HCPTI transferred to its new offices at Tondo, Morales received a memo
reassigning him to Operations Cost Accounting

w/n dismissal was valid

2003, Morales wrote the management protesting that his reassignment was a
clear demotion since the position to which he was transferred was not even
included in HCPTIs plantilla.

HOLDING:

Defence: management prerogative

1. Yes, jurisprudence recognizes the exercise of management prerogative to


transfer or assign employees from one office or area to another, provided there is
no demotion in rank, diminution of salary, benefits and other privileges, and the
action is not motivated by discrimination, made in bad faith, or effected as a form
of punishment or demotion without sufficient cause.

Morales filed a complaint for constructive dismissal

Albaydas transfer in this case was for a legitimate purpose to maximize


business opportunities, growth and development of personnel and the expertise of
Albayda being used to assist the CDO office. He will be holding the same position,
and there was no diminution of emoluments. He was even entitled to relocation
benefits and allowance.

CA reversed NLRC

Moreover, in his employment contract, he agreed that he was willing to be


assigned to any work or workplace during the period of his employment as may be
determined by the company whenever the operations require such assignment.
There was no evidence showing that the restructuring of the company was done
with ill motives or with malice and bad faith purposely to constructively
terminate respondent. The CA failed to recognize the very nature of a salesman
that it is mobile and ambulant.
2. Yes, while no actual hearing was held, the same is not fatal, as only an ample
opportunity to be heard is what the law requires in order to satisfy due process
of law. The twin notice were complied with when respondent was sent a memo
which served as a final warning directing him to report for work within 5 days
otherwise he will be terminated on the ground of being AWOL. Upon receipt of
this first memo, respondent could have asked for a conference with the company
which he failed to do, hence the second memo informing him of the companys
decision to terminate his services for being AWOL and for insubordination for
deliberately ignoring the defying the lawful orders of his employer which is a
valid ground for termination under Art. 282 (a) of the LC.
Where the objection to the transfer is grounded solely upon personal

LA dismissed complaint
NLRC reversed - reassignment was a clear demotion despite lack of showing of
diminution of salaries and benefits.
WON THE CHANGE IN THE DESIGNATION/POSITION
OF PETITIONER CONSTITUTED CONSTRUCTIVE
DISMISSAL.
Held: yes
Constructive dismissal exists where there is cessation of work because "continued
employment is rendered impossible, unreasonable or unlikely, as an offer
involving a demotion in rank or a diminution in pay" and other benefits.
The employer is charged with the burden of proving that its conduct and action
are for valid and legitimate grounds such as genuine business necessity and that
the transfer is not unreasonable, inconvenient or prejudicial to the employee. If
the employer cannot overcome this burden of proof, the employees transfer shall
be tantamount to unlawful constructive dismissal.
HCPTI failed to discharge this burden. While there was a lack of showing that the
transfer or reassignment entailed a diminution of salary and benefits, one fact
that must not be lost sight of was that Morales was already occupying the
position of Division Manager at HCPTIs Accounting Department as a
consequence of his promotion in 2002 so his reassignment was a clear demotion.
HCPTI argues that Morales transfer was brought about by the reorganization of
its corporate structure but Morales was able to prove that HCPTIs existing

38

plantilla did not include an Operations Cost Accounting Department and/or an


Operations Cost Accountant.
PETITION GRANTED. CA reversed. NLRC reinstated.
52.

Tee

Phil. Telegraph v CA, 412 SCRA 263 ( 2003)

Philippine Telegraph & Telephone Corp. (PT&T Corp.) vs. Court of


Appeals, et al.
G.R. No. 152057; 29 Sept. 2003; Callejo, Sr., J.

Private respondents (Cristina Rodiel, Jesus Paracale, Romeo Tee, Benjamin Lakandula, Avelino Acha, Ignacio Dela Cerna and Guillermo
Domegillo) were employees of PT & T Corp., assigned in different
branches all over the country.

In 1997, PT & T Corp. adopted its Relocation and Restructuring


Program designed to sustain its retail operations; decongest surplus
workforce in some branches; promote efficiency and productivity; lower
expenses incidental to hiring and training new personnel; and avoid retrenchment of employees occupying redundant positions.

Private respondents were among the employees affected by the said program. They were informed of their relocation and directed to report to their respective relocation assignments. Moreover, the employees who would agree to the transfer would be considered promoted,
thus:
Name

From
Position/Job
Grade

Work
Location
(from)

To
Position/Jo
b Grade

New
location

Acha

Jr.
Counter/JG
2

Legaspi

Courier/JG
3

Romblon/Odiong
an

Rodel

Jr. Counter
Clerk/JG2

Cabanatua
n

Clerk/JG4

Baguio

Dela
Cerna

Jr.
CW
Operator/JG
2

Cotabato
City

Clerk/JG4

Kidapawan

Demigillo

Jr.
CW
Operator/JG
2

Midsayap
North

Courier/JG
3

Lebak

Lakandul
a

Counter/JG
3

Iligan

Clerk/JG4

Butuan

Paracale

Jr.

Makar,

Clerk/JG4

Butuan

CW

Gen.
Santos

TTY
OperatorGen/JG4

Zamboang
a City

Clerk/JG4

Jolo

Private respondents rejected the offer. Dissatisfied with their explanation for refusing the offer, PT&T Corp. considered their refusal
as insubordination and willful disobedience to a lawful order;
hence, they were dismissed from work.

Private respondents bargaining agent, PT&T Workers Union-NAFLUKMU filed a complaint against PT&T Corp. for illegal dismissal and unfair labor practice for and in behalf of the dismissed employees before the
arbitration branch of the NLRC.

The contention of the private respondents is that their refusal to


transfer could not possibly give rise to a valid dismissal on the ground of
willful disobedience, as their transfer was prejudicial and inconvenient; thus unreasonable. Moreover, since their transfer resulted in
their promotion, they had the right to refuse or decline the positions being offered them. It follows that the refusal to accept the transfer could
not have amounted to insubordination or willful disobedience to lawful
orders.

For its part, PT&T Corp. maintains that the transfers were made in lawful exercise of its management prerogative and were done in good faith.
The transfers were aimed at decongesting surplus employees and detailing them to a more demanding branch.

Labor Arbiter dismissed the complaint of the private respondents;

NLRC reversed the LA and declared that there was illegal dismissal;

CA dismissed the petition for certiorari filed by PT&T and affirmed the
decision of the NLRC.

*Digest not mine, taken from Working Conditions section.


Facts

Operator/JG
2

work

Issue/Holding/Ratio
WON the private respondents were illegally dismissed Yes. The dismissal is
not based on any of the just or valid grounds under Art. 282 of the Labor
Code, the NLRC correctly ordered the private respondents reinstatement
without loss of seniority rights and the payment of backwages from the time of
their dismissal up to their actual reinstatement.

As maintained by PT&T Corp. when the private respondents were transferred they were also promoted.

An employee cannot be promoted without his consent. A transfer


that results in promotion or demotion, advancement or reduction or a transfer that aims to lure the employee away from his
39

53.

permanent position cannot be done without the employees consent.

W/N Artificio was validly placed under preventive suspension pending


investigation of the charges?YES

A promotion is the advancement from one position to another with an increase in duties and responsibilities as authorized by law, and usually
accompanied by an increase in salary. The indispensable element of promotion is the advancement from one position to another or an upward
vertical movement of the employees rank or position. Any increase in
salary should only be considered incidental.

W/N Artificio was illegally dismissed? NO


HELD:
SEC. 8. Preventive suspension. The employer may
place the worker concerned under preventive suspension if his
continued employment poses a serious and imminent threat to
the life or property of the employer or of his co-workers.

Considering further that a promotion is in the nature of a gift or reward,


an employee cannot be compelled to accept a promotion. In other words,
he has a right to refuse, and the exercise of this right cannot be considered in law as insubordination or willful disobedience of a lawful order of
the employer.

SEC. 9. Period of suspension. No preventive


suspension shall last longer than thirty (30) days. The employer
shall thereafter reinstate the worker in his former or in a
substantially equivalent position or the employer may extend
the period of suspension provided that during the period of
extension, he pays the wages and other benefits due to the
worker. In such case, the worker shall not be bound to
reimburse the amount paid to him during the extension if the
employer decides, after completion of the hearing, to dismiss the
worker.

Artificio v NLRC, NLRC, GR No. 172988 (2010)

JOSE P. ARTIFICIO V NLRC- Santos


FACTS:

ISSUE:

Jose P. Artificio was employed as security guard by RP Guardians Security Agency, Inc
June 2002: Artificio had a heated argument with a fellow security guard,
Merlino B. Edu (Edu).
July 2002, Edu submitted a confidential report to Antonio A. Andres
(Andres), Administration & Operations Manager, (Artificio be investigated for maliciously machinating Edus hasty relief from his post and for
leaving his post during night shift duty to see his girlfriend at a nearby
beerhouse)
July 2002, another security guard, Gutierrez Err (Err), sent a report to
Andres stating that Artificio arrived at the office under the influence of
liquor. When Artificio learned that no salaries would be given that day,
he bad-mouthed the employees of RP Guardians Security Agency, Inc.
and threatened to arson their office.
Andres issued Memorandum: temporarily relievied Artificio from his post
and placed him under preventive suspension pending investigation for
conduct unbecoming a security guard abandonment of post during
night shift duty, light threats and irregularities in the observance of
proper relieving time.
o He also directed Artificio to report to the office and submit his
written answer
Another MEMO: hearing will be held on 12 August 2002
W/out waiting for hearing to be held, Artificio filed complaint for Illegel
Dismissal
LA found RP guilty of illegal suspension and dismissal
NLRC Set aside LA decision. RP to reinstate Artificio w/out loss of seniority rights. MR Denied
CA Affirmed NLRC

Artificios preventive suspension was justified since he was employed as a


security guard tasked precisely to safeguard respondents client.
His continued presence in respondents or its clients premises poses a serious threat to respondents, its employees and client in light of the serious
allegation of conduct unbecoming a security guard such as abandonment of
post during night shift duty, light threats and irregularities in the observance of proper relieving time.
Management has the prerogative to discipline its employees and to impose
appropriate penalties on erring workers pursuant to company rules and
regulations (always respected by the courts as long as exercised in good
faith for the advancement of the employers interest and not for the purpose of defeating or circumventing the rights of the employees under special laws or under valid agreements)

On the alleged lack of due notice:

Artificio regrettably chose not to present his side at the administrative


hearing scheduled to look into the factual issues that accompanied the
accusation against him.
He avoided the investigation into the charges by filing his illegal dismissal complaint ahead of the scheduled investigation; he thus preempted
the investigation that could have afforded him the due process

On the benefits due:

Since he has worked with the company for 16 years with no other derogatory record, he was granted equitable relief in the form of separation pay.
No backwages since no ID
40


54.

Paid the rate of one (1) month pay for every year of service reckoned from
the start of his employment with the respondents in 1986 until 2002

Sexual Harassment Act of 1995, or R.A. No. 7877. Rule II, Section 1
of the MIT Rules and Regulations provides:

Gatbonton v NLRC, 479 SCRA 416 (2006)


Section 1. Preventive Suspension of Accused in
Sexual Harassment Cases. Any member of the
educational community may be placed immediately
under preventive suspension during the pendency of the
hearing of the charges of grave sexual harassment
against him if the evidence of his guilt is strong and the
school head is morally convinced that the continued
stay of the accused during the period of investigation
constitutes a distraction to the normal operations of the
institution or poses a risk or danger to the life or
property of the other members of the educational
community.

Renato Gatbonton v. NLRC


Jan. 23, 2006; GR No. 146779; Austria-Martinez, J.
Facts

Petitioner Renato S. Gatbonton is an associate professor of


respondent Mapua Institute of Technology (MIT), Faculty of
Civil Engineering.
A civil engineering student of respondent MIT filed a lettercomplaint against petitioner for unfair/unjust grading system,
sexual harassment and conduct unbecoming of an academician.
Pending investigation, MIT placed petitioner under a 30-day
preventive suspension.
Petitioner filed with the NLRC a complaint for illegal
suspension, damages and attorneys fees.
LA: ruled for petitioner, that the suspension was illegal, but it
dismissed claim for damages.
NLRC: reversed.
CA: affirmed NLRC.

Issue/Holding/Ratio
WON the preventive suspension is legal? No. First, the administrative
rules were not duly published. Moreover, there was no ground for such
suspension.

R.A. No. 7877 imposed the duty on educational or training institutions


to promulgate rules and regulations in consultation with and jointly
approved by the employees or students or trainees, through their duly
designated representatives, prescribing the procedures for the
investigation of sexual harassment cases and the administrative
sanctions therefor.
Preventive suspension is a disciplinary measure for the protection of the
companys property pending investigation of any alleged malfeasance or
misfeasance committed by the employee.
However, when it is determined that there is no sufficient basis
to justify an employees preventive suspension, the latter is
entitled to the payment of salaries during the time of preventive
suspension.
Petitioners preventive suspension was based on respondent
MITs Rules and Regulations for the Implemention of the Anti-

It must be noted however, that respondent published said


rules and regulations only on February 23, 1999.
Taada vs. Tuvera: Administrative rules and regulations must
also be published if their purpose is to enforce or implement
existing law pursuant also to a valid delegation.
The Mapua Rules is one of those issuances that should be
published for its effectivity, since its purpose is to enforce and
implement R.A. No. 7877, which is a law of general application.
o
The Mapua Rules itself explicitly required publication of
the rules for its effectivity, as provided in Section 3, Rule IV
(Administrative Provisions), which states that [T]hese
Rules and Regulations to implement the Anti-Sexual
Harassment Act of 1995 shall take effect fifteen (15) days
after publication by the Committee.
Thus, at the time of the imposition of petitioners preventive
suspension on January 11, 1999, the Mapua Rules were not
yet legally effective, and therefore the suspension had no
legal basis.
Moreover, there is no sufficient basis to justify his preventive
suspension. Under the Mapua Rules, an accused may be placed
under preventive suspension only if the evidence of guilt is strong, or
the accused poses a risk or danger to the life or property of other
members of the community.
In petitioners case, there is no indication that petitioners preventive
suspension may be based on the foregoing circumstances
The schools resolution on his case does not show that evidence of
petitioners guilt is strong and that the school head is morally
convinced that petitioners continued stay during the period of
investigation constitutes a distraction to the normal operations of
the institution; or that petitioner poses a risk or danger to the life or
property of the other members of the educational community.
Even under the Labor Code, petitioners preventive
suspension finds no valid justification. As provided in Section 8,
41

Rule XXIII, Book V of the Omnibus Rules Implementing the Labor


Code:
Sec. 8. Preventive Suspension. The employer may
place the worker concerned under preventive suspension
if his continued employment poses a serious threat to the
life or property of the employer or of his co-workers.

Consequently, the payment of wages during his 30-day


preventive suspension, i.e., from January 11, 1999 to February
10, 1999, is in order.
With regard to petitioners claim for damages, the Court finds
the same to be without basis. While petitioners preventive
suspension may have been unjustified, this does not
automatically mean that he is entitled to moral or other
damages. The records of this case are bereft of any
evidence showing that respondent MIT acted in bad faith
or in a wanton or fraudulent manner in preventively
suspending petitioner.

55.
Manila Pavilion Hotel, etc. vs. Henry Delada, G.R. No. 189947,
January 25, 2011

safety reasons, i.e. "he might sabotage the food" he was placed on a 30-day
preventive suspension.

On 8 Jun 2007, MPH issued a decision which found him guilty of insubordination based on his repeated and willful disobedience of the transfer order. He was imposed a 90-day suspension as penalty.

On 14 Dec, the PVA issued its decision and ruled that Delada's transfer
was a valid exercise of management prerogative. The PVA found no malice
or bad faith in MPH's decision to transfer him, and that the real reason
why he refused to obey the transfer was that he asked for additional monetary benefits as a condition for the transfer. Furthermore, his transfer did
not prejudice, inconvenience nor cause diminution of salaries or demotion
in rank. Thus Delada had no justifiable reason to refuse or delay compliance with the management directive.

PVA also ruled that there was no legal or factual basis on the imposition of
preventive suspension on Delada. The mere assertion that it is not farfetched for Delada to sabotage the food because of the hostile relationship
was more imagined than real.

PVA also found that MPH went beyond the 30-day period of preventive
suspension prescribed by the IRR of the Labor Code when MPH proceeded to impose a 90-day suspension on him.

Also the PVA ruled that MPH lost authority to continue with the administrative proceedings for insubordination and willful disobedience, because it
acquired exclusive jurisdiction over the issue because the issue of the validity of the transfer order necessarily encompassed the issue of insubordination and willful disobedience of the transfer order. Thus MPH relinquished
its power to impose disciplinary action on Delada.

CA affirmed PVA decision.

Manila Pavilion v. Delada


GR No. 189947 / 25 Jan 2012 / J. Sereno
FACTS

Henry Delada was union president of the Manila Supervisors Association


at the Manila Pavilion Hotel. He was originally Head Waiter of Rotisserie,
a fine dining restaurant operated by MPH.

Pursuant to a personnel reorganization program he was reassigned as


Head Waiter of Seasons Coffee Shop, another restaurant at the same hotel.

He declined the transfer and requested a grievance meeting pursuant to


their CBA.

MPH replied and told him to report to his new assignment without prejudice to the resolution of the grievance. He continued to refuse and MPH
sent him several memos requiring him to explain in writing why he should
not be penalized for the ff offenses: serious misconduct, willful disobedience
of the lawful orders of the employer, gross insubordination, gross and habitual neglect of duties and willful breach of trust.

The parties failed to reach a settlement at the grievance meeting, and the
matter eventually reached the National Conciliation and Mediation Board.

While the validity of his transfer was pending before the Panel of Voluntary Arbitrators (PVA), MPH continued disciplinary action for his refusal
to report to his new post at the Seasons Coffee Shop. Citing security and

ISSUE

W/N MPH retained the authority to continue the administrative case


against Delada for insubordination and willful disobedience of the transfer
order.

W/N the 90-day suspension is valid.

HELD 1. Yes, they did.

The rule is that voluntary arbitrators can assume jurisdiction and make a
final settlement on related issues of questions expressly delineated by the
submission agreement. However, in this case that is not applicable because
the PVA did not make a ruling on the specific issue of insubordination and willful disobedience, but rather its disagreement with the
42

90-day penalty of suspension that went beyond the 30 day limit for
preventive suspension.

Quoting the PVA "MPHs authority to impose disciplinary action to Delada


must now give way to the jurisdiction of this panel of arbitrators to rule on
the issues at hand. By necessary implication, this Panel is thus constrained
to declare both the preventive suspension and the separate suspension of
90-days meted to Delada to be not valid and justified."

56.

2. Yes it is valid.

Quoting the PVA "The preventive suspension of Delada is supposed to expire on 09 June 2007, but without notifying Delada, the MPH proceeded to
impose a separate penalty of 90-day suspension to him which took effect
only on 18 June 2007, or way beyond the 30-day rule mandated by the
Rules. While the intention of the MPH is to impose the 90-day suspension
as a separate penalty against Delada, the former is already proscribed from
doing so because as of 05 June 2007, the dispute at hand is now under the
exclusive jurisdiction of the panel of arbitrators. In fact, by its own admission, the MPH categorically stated in its Position Paper that as of 25 May
2007, or before the suspension order was issued, MPH and Delada had already formulated and submitted the issues for arbitration."

The basis of the 30 day preventive suspension must be distinguished from


the basis of the 90 day penalty of suspension.

Preventive suspension is a disciplinary measure resorted to by the employer pending investigation of an alleged malfeasance or misfeasance committed by an employee. On the other hand, the penalty of suspension refers to
the disciplinary action imposed on the employee after an official investigation or administrative hearing is conducted.

Thus a finding of validity of the 90-day suspension does not embrace the
validity of the 30-day preventive suspension. In this case MPH does not
anymore assail the validity of the 30-day suspension.

Applying the doctrine in Allied Banking Corporation v. Court of Appeals,


employees may object to negotiate and seek redress against employers for
rules or orders that they regard as u njust or illegal. However, until and
unless these rules or orders are declared illegal or improper by competent
authority, the employees ignore or disobey them at their peril.

Quoting the PVA decision again "In fact, Delada cannot hide under the legal cloak of the grievance machinery of the CBA or the voluntary arbitration proceedings to disobey a valid order of transfer from the management
of the hotel. While it is true that Deladas transfer to Seasons is the subject
of the grievance machinery in accordance with the provisions of their CBA,
Delada is expected to comply first with the said lawful directive while
awaiting the results of the decision in the grievance proceedings."

Thus, his defiance being unjustified, MPH had the right to impose the 90day suspension, and no back wages can be awarded for this time.
Edge Apparel Inc v NLRC, 286 SCRA 302 (1998)

Edge Apparel vs. NLRC (Digest by Robby Solis)


(Vitug, J. ;G.R. No. 121314. February 12, 1998)
(CAUTION: As we have 3 cases each on short notice, I utilized a Reviewer Digest
to help make this. Credit to the source, and please note that the digest may not be
as comprehensive. Thanks.)
Facts:

Pursuing its retrenchment program, petitioner Edge Apparel, Inc., dismissed


the 6 private respondents herein, Antipuesto et al.
The respondents consulted with the DOLE who advised them that it would be
best for them to receive the separation pay being offered by the corporation,
an advice which they took. However, the subsequent receipt of their separation pay benefits, nevertheless, did not deter Antipuesto, et al., from filing a
complaint for illegal dismissal against the corporation.
They claim that the retrenchment program was a mere misrepresentation by
Edge Apparel.
Edge Apparel countered that its financial obligations, amounting to about P8
Million, had begun to eat up most of its capital outlay and resulted in unabated losses of P681,280.00 in 1989, P262,741.00 in 1990, P162,170.00 in
1991 and P749,294.00 in 1992, constraining the company to adopt and implement a retrenchment program.
LA: Dismissed the Complaint.
NLRC: Affirmed with Modification. It found that "There is basis in the retrenchment of these workers.
o These workers were assigned to row #8 of the sewing line for simple
garments which was phased out due in fact to the dropping of this particular line of business.
o Termination of an employee's services because of a reduction of work
force due to a decrease in the scope or volume of work of the employer is
synonymous to termination because of redundancy under Article 283
o "In case of termination due to the installation of labor saving devices or
redundancy, the worker affected thereby shall be entitled to at least one
(1) month pay or to at least one month pay for every year of service,
which ever is higher. (Art. 283, Labor Code).
o "Under the circumstances obtaining in this case, the termination of the
retrenched employees is considered a redundancy. Hence, the complainants, who were already paid the separation pay equivalent to 1/2 month
pay per year of service, are entitled to be paid the additional separation
pay equivalent to 1/2 month pay for every year of service.

Issues/Held:

WON Respondents were illegally dismissed. NO.


WON redundancy was the cause of their termination. NO, Retrenchment.
43

Ratio:

Article 283 LC specifies the authorized causes for the termination of employment.
The installation of labor-saving devices contemplates the installation of
machinery to effect economy and efficiency in its method of production.
Redundancy exists where the services of an employee are in excess of what
would reasonably be demanded by the actual requirements of the enterprise.
A position is redundant when it is superfluous. An employer has no legal obligation to keep on the payroll employees more than the number needed for
the operation of the business.
Retrenchment, in contrast to redundancy, is an economic ground to reduce
the number of employees. In order to be justified, the termination of employment by reason of retrenchment must be due to business losses or reverses
which are serious, actual and real.
It is an act of the employer of reducing the work force because of losses in the
operation of the enterprise, lack of work, or considerable reduction on the
volume of business. Retrenchment is, in many ways, a measure of last resort.
The payment of separation pay would be due when a dismissal is on account
of an authorized cause. The amount of separation pay depends on the ground
for the termination of employment. A dismissal due to the installation of labor saving devices, redundancy (Article 283) or disease (Article 284), entitles
the worker to a separation pay equivalent to "one (1) month pay or at least
one (1) month pay for every year of service, whichever is higher." When the
termination of employment is due to retrenchment to prevent losses, or to
closure or cessation of operations of establishment or undertaking not due to
serious business losses or financial reverses, the separation pay is only an
equivalent of "one (1) month pay or at least one-half (1/2) month pay for every
year of service, whichever is higher." In the above instances, a fraction of at
least six (6) months is considered as one (1) whole year.
In this case, the Labor Arbiter and the NLRC both concluded that there had
been a valid ground for the retrenchment of private respondents. The documents presented in evidence were found to "conclusively show that (petitioner) suffered serious financial losses."
The elements needed for the retrenchment to be valid - i.e., that the
losses expected are substantial and not merely de minimis in extent; that the
expected losses are reasonably imminent such as can be perceived objectively
and in good faith by the employer; that the retrenchment is reasonably necessary and likely to effectively prevent the expected losses; and that the imminent losses sought to be forestalled are substantiated -were adequately
shown in the present case.
Procedurally, in order to validly effect retrenchment, the employer must observe two other requirements, viz: (a) service of a prior written notice of at
least one month on the workers and the Department of Labor and Employment, and (b) payment of the due separation pay. Petitioner has been found
to have complied with the above requirements of the law, including the payment of separation pay.
The NLRC has gravely abused its discretion. The law acknowledges the right
of every business entity to reduce its work force if such measure is made necessary or compelled by economic factors that would otherwise endanger its

stability or existence. In exercising its right to retrench employees, the firm


may choose to close all, or a part of, its business to avoid further losses or
mitigate expenses. (Caffco International Limited vs. Office of the MinisterMinistry of Labor and Employment)
The Court sustains the position taken by the Labor Arbiter that private respondents should only be entitled to severance compensation equivalent to
one-half (1/2) month pay for every year of service.

Dispositive: WHEREFORE, the appealed decision, promulgated on 26 April


1995, is MODIFIED by deleting the additional award of separation pay to private
respondents decreed by the NLRC. No costs.
57.

FASAP v PAL, 559 SCRA 252 (2008)

58.

FASAP v PAL, GR No. 178083 (2009)

FASAP vs. PAL, PATRIA CHIONG and CA


FACTS:

Exercising its management prerogative and sound business judgment,


PAL decided to cut its fleet of aircraft in order to minimize its operating
losses and rescue itself from total downfall; which meant that a
corresponding company-wide reduction in manpower necessarily had to
be made. As a result, 5,000 PAL employees (including the herein 1,400
cabin attendants) were retrenched. PAL argues that aside from the
confluence of simultaneous unfortunate events that occurred during the
time, like successive strikes, peso depreciation and the Asian currency
crisis, there was a serious drop in passenger traffic which necessitated
the closure of PALs entire European, Australian, and Middle
East operations and numerous Asian stations, as well as some of its
domestic stations.
FASAP never assailed the economic basis for the retrenchment, but only
the allegedly discriminatory and baseless manner by which it was
carried out.
PAL significantly changed its stance and blamed the June 5, 1998 pilots
strike as the real culprit which drove it to undertake the massive
retrenchment under scrutiny. This time, PAL characterizes the
retrenchment scheme and the downsizing of aircraft as mere necessary
reactions to or unfortunate consequences of the pilots strike, which it
claims likewise necessitated a disregard of all previous negotiations for
the implementation of cost-cutting measures that could have rendered
the retrenchment scheme unnecessary, and which cost-cutting measures
it no longer found necessary to undertake.

ISSUE/S: whether the retrenchment of the 1,400 cabin personnel was warranted
HELD: NO.

The strike was a temporary occurrence that did not necessitate the
immediate and sweeping retrenchment of 1,400 cabin or flight
attendants. There was no reason to drastically implement
44

a permanent retrenchment scheme in response to a temporary strike,


which could have ended at any time, or remedied promptly, if
management acted with alacrity. Juxtaposed with its failure to
implement the required cost-cutting measures, the retrenchment scheme
was a knee-jerk solution to a temporary problem that beset PAL at the
time.
PAL cannot be allowed to conveniently blame the striking pilots for
causing the massive retrenchment of cabin personnel. Using them as
scapegoats to validate a comprehensive retrenchment scheme of cabin
personnel without observing the requirements set by law is both unfair
and underhanded. PAL must still prove that it implemented cost-cutting
measures to obviate retrenchment, which under the law should be
the last resort. By PALs own admission, however, the cabin personnel
retrenchment scheme was one of the first remedies it resorted to, even
before it could complete the proposed downsizing of its aircraft fleet.
Again, it must be emphasized that in order for a retrenchment scheme to
be valid, all of the following elements under Article 283 of the Labor
Code must concur or be present, to wit:
(1) That retrenchment is reasonably necessary and likely to
prevent business losses which, if already incurred, are not
merely de minimis, but substantial, serious, actual and real, or
if only expected, are reasonably imminent as perceived
objectively and in good faith by the employer;
(2) That the employer served written notice both to the
employees and to the Department of Labor and Employment at
least one month prior to the intended date of retrenchment;
(3) That the employer pays the retrenched employees
separation pay equivalent to one (1) month pay or at least onehalf () month pay for every year of service, whichever is higher;
(4) That the employer exercises its prerogative to retrench
employees in good faith for the advancement of its interest and
not to defeat or circumvent the employees right to security of
tenure; and,

In the absence of one element, the retrenchment scheme becomes an


irregular exercise of management prerogative. The employers obligation
to exhaust all other means to avoid further losses without retrenching its
employees is a component of the first element as enumerated above. To
impart operational meaning to the constitutional policy of providing full
protection to labor, the employers prerogative to bring down labor costs
by retrenching must be exercised essentially as a measure of last resort,
after less drastic means have been tried and found wanting.
In the instant case, PAL admitted that since the pilots strike allegedly
created a situation of extreme urgency, it no longer implemented costcutting measures and proceeded directly to retrench. This being so, it
clearly did not abide by all the requirements under Article 283 of the
Labor Code. At the time it was implemented, the retrenchment scheme
under scrutiny was not triggered directly by any financial difficulty PAL
was experiencing at the time, nor borne of an actual implementation of
its proposed downsizing of aircraft. It was brought about by and
resorted to as an immediate reaction to a pilots strike which, in strict
point of law and as herein earlier discussed, may not be considered as a
valid reason to retrench, nor may it be used to excuse PAL for its nonobservance of the requirements of the law on retrenchment under the
Labor Code.

Therefore, the Court finds the retrenchment of the flight attendants was illegally
executed. PAL failed to observe the procedure and requirements for a valid
retrenchment. Assuming that PAL was indeed suffering financial losses, the
requisite proof therefor was not presented before the NLRC which was the proper
forum. More importantly, the manner of the retrenchment was not in accordance
with the procedure required by law. Hence, the retrenchment of the flight
attendants amounted to illegal dismissal. Consequently, the flight attendants
affected are entitled to the reliefs provided by law, which include backwages and
reinstatement or separation pay, as the case may be.
59.
Lambert Pawnbrokers & Jewelry Corp. v Binamira GR No.
170464 (2010)
60.
Philippine Carpet Manufacturing Corporation, et al. v. Ignacio B.
Tagyamon, et al.,
61.
Ruben L. Andrada v NLRC, Subic Legend Resorts And Casino,
Inc., G.R. No. 173231
ANDRADA v. NLRC & SUBIC LEGEND RESORTS
J. Velasco
G.R. No. 173231

(5) That the employer uses fair and reasonable criteria in


ascertaining who would be dismissed and who would be retained
among the employees, such as status, efficiency, seniority,
physical fitness, age, and financial hardship for certain workers.

TOPIC: Redundancy; Under the Labor Code, retrenchment and redundancy are
authorized causes for separation from service. However, to protect labor,
dismissals due to retrenchment or redundancy are subject to strict requirements
under Article 283 of the Labor Code.

45

FACTS: Andrada and six other petitioners were employees of Legend as


architects, engineers, surveyors, etc etc under the Project Development Division.
Legend sent a notice of retrenchment to the DOLE and petitioners and gave the
petitioners an option to enter into a (1) temporary retrenchment and lay-off; (2)
permanent retrenchment and payment of separation pay and other benefits after
lapse of 30 days; or (3) immediate retrenchment and payment of SP and OB. On
the same day, the Labor and Employment Center of SBMA advertised that
Legend was in need of employees for positions the petitioners recently vacated.
After lapse of 30 days, Legend informed petitioners of their retrenchment.
Petitioners signed a quitclaim, reserving their right to sue Legend. Petitioners
filed a case for illegal dismissal and money claims for payment of their share in
service charges, unused leaves, and salaries for the unexpired portions of their
contracts.
LA: In favour of petitioners; no proof of actual business losses or redundancy in
positions; quitclaims were part of clearance forms; damages were due because of
petitioners humiliation and social trauma as well as bad faith attending their
dismissals; bad faith for advertising petitioners former posts

whichever is higher
4.

Redundancy as distinguished from retrenchment


Retrenchment

exists where the


services of an employee are in excess of what is
reasonably
demanded by the
actual
requirements of the enterprise.

A position is redundant where it


is
superfluous,
and superfluity of
a position or positions may be the
outcome
of
a
number of factors,
such as over hiring of workers,
decreased volume
of business, or
dropping of a particular
product
line or service activity previously
manufactured or
undertaken
by
the enterprise.

CA: In favour of Legend; redundancy existed (basis not shown)

RESOLUTION: Petition GRANTED; CA decision REVERSED


Redundancy and retrenchment; When valid; Redundancy and retrenchment
are valid exercises of management prerogative. Under the LC, they are
authorized causes for separation from service. To be valid, they are subject to the
ff requirements
Retrenchment

Redundancy

1.

Proof of losses

1.

2.

Written notice to
employee
and
DOLE at least 1
month prior too intended date of retrenchment

Written notice to
employee
and
DOLE at least 1
month prior to intended date of retrenchment

2.

Separation
pay
equivalent
to
1
month pay/ 1 month
salary for each year
of service, whichever is higher

3.

Separation
pay
equivalent
to
1
month
pay/

month salary for


each year of service,

3.

Good faith in abol-

Adoption of fair and


reasonable criteria
which positions to
be declared redundant and accordingly abolished

Legend failed to show it was in a poor financial condition prior to and at the
time of implementing its retrenchment program. No audited financial
statements regarding financial losses.

NLRC: In favour of Legend; there were actual business losses and redundancy;
not Legend but a certain Gaehin looking for employees in the SBMA
advertisement.
ISSUE: Whether or not valid grounds for retrenchment or redundancy existed
(NO)

ishing
redundant
positions

Redundancy

the termination
of
employment
initiated by the
employer through
no fault of the
employees
and
without prejudice
to the latter, resorted to by management during
periods of business
recession,
industrial
depression, or seasonal
fluctuations, or during
lulls occasioned
by lack of orders,
shortage of materials, conversion
of the plant for a
new production
program or the
introduction
of
new methods or
more
efficient
machinery, or of
automation.

According to the CA, Legend proved the existence of redundancy when it


submitted a status review of its project division where it reported that the 78-man
personnel exceeded the needs of the company. The report further stated that
there was duplication of functions and positions, or an over supply of employees,
46

especially among architects, engineers, draftsmen, and interior designers. These


are mere allegations, as they did not even explain how petitioners functions were
superfluous or even necessary. Substantial evidence is necessary to prove
redundancy.
62.
Smart Communications Inc v Regina Astorga, GR No. 148132
(2008)
Smart Communications vs Astorga 542 scra 153 (2007)
Source: Internet
FACTS:

Regina M. Astorga (Astorga) was employed by respondent Smart


Communications, Incorporated (SMART) on May 8, 1997 as District
Sales Manager of the Corporate Sales Marketing Group/ Fixed Services
Division (CSMG/FSD).
As District Sales Manager, Astorga enjoyed additional benefits, namely,
annual performance incentive equivalent to 30% of her annual gross
salary, a group life and hospitalization insurance coverage, and a car
plan in the amount of P455,000.00.
In February 1998, SMART launched an organizational realignment to
achieve more efficient operations. This was made known to the
employees on February 27, 1998. Part of the reorganization was the
outsourcing of the marketing and sales force. Thus, SMART entered into
a joint venture agreement with NTT of Japan, and formed SMART-NTT
Multimedia, Incorporated (SNMI). Since SNMI was formed to do the
sales and marketing work, SMART abolished the CSMG/FSD, Astorgas
division.
SNMI agreed to absorb the CSMG personnel who would be recommended
by SMART. SMART then conducted a performance evaluation of CSMG
personnel and those who garnered the highest ratings were favorably
recommended to SNMI. Astorga landed last in the performance
evaluation, thus, she was not recommended by SMART. SMART offered
her a supervisory position in the Customer Care Dept but she refused the
offer.
On March 3, 1998, SMART issued a memorandum advising Astorga of
the termination of her employment on ground of redundancy, effective
April 3, 1998. Astorga received it on March 16, 1998.
The termination of her employment prompted Astorga to file a
Complaint for illegal dismissal, non-payment of salaries and other
benefits with prayer for moral and exemplary damages against SMART.
She claimed that abolishing CSMG and, consequently, terminating her
employment was illegal for it violated her right to security of tenure.
On May 18, 1998, SMART sent a letter to Astorga demanding that she
pay the current market value of the Honda Civic Sedan which was given
to her under the companys car plan program, or to surrender the same
to the company for proper disposition. Astorga, however, failed and
refused to do either, thus prompting SMART to file a suit for replevin
with the Regional Trial Court of Makati (RTC) on August 10, 1998.

ISSUE:

Whether the dismissal of Astorga is valid


Whether or not the RTC has jurisdiction over the complaint for
recovery of a car which Astorga acquired as part of her employee
benefit.

HELD:
1. Astorga is declared validly dismissed.
Astorga was terminated due to redundancy, which is one of the authorized
causes for the dismissal of an employee. Redundancy in an employers
personnel force necessarily or even ordinarily refers to duplication of work.
The characterization of an employees services as superfluous or no longer
necessary and, therefore, properly terminable, is an exercise of business
judgment on the part of the employer. An employer is not precluded from
adopting a new policy conducive to a more economical and effective
management even if it is not experiencing economic reverses. Neither does
the law require that the employer should suffer financial losses before he can
terminate the services of the employee on the ground of redundancy.
But while tilting the scales of justice in favor of workers, the fundamental
law also guarantees the right of the employer to reasonable returns for his
investment. In this light, we must acknowledge the prerogative of the
employer to adopt such measures as will promote greater efficiency, reduce
overhead costs and enhance prospects of economic gains, albeit always within
the framework of existing laws.
However, SMART failed to comply with the mandated one (1) month notice
prior to termination. The record is clear that Astorga received the notice of
termination only on March 16, 1998 or less than a month prior to its
effectivity on April 3, 1998. Likewise, the Department of Labor and
Employment was notified of the redundancy program only on March 6, 1998.
Article 283 of the Labor Code clearly provides:
Art. 283. Closure of establishment and reduction of personnel. The
employer may also terminate the employment of any employee due to the
installation of labor saving devices, redundancy, retrenchment to prevent
losses or the closing or cessation of operation of the establishment or
undertaking unless the closing is for the purpose of circumventing the
provisions of this Title, by serving a written notice on the workers and
the Ministry of Labor and Employment at least one (1) month before the
intended date thereof x x x.
SMARTs assertion that Regina cannot complain of lack of notice because the
organizational realignment was made known to all the employees as early as
February 1998 fails to sway. Reginas actual knowledge of the reorganization
cannot replace the formal and written notice required by the law.
Notwithstanding her knowledge of the reorganization, she remained uncertain
about the status of her employment until SMART gave her formal notice of
termination.
47

The SC also ruled that it is proper to increase the amount of the penalty on
SMART to P50,000.00. However, the award of backwages to Regina by the CA
should be deleted for lack of basis. Backwages is a relief given to an illegally
dismissed employee. Since her dismissal is for an authorized cause, Regina is not
entitled to backwages. The CAs award of backwages is totally inconsistent with
its finding of valid dismissal.
2. The RTC rightfully assumed jurisdiction over the suit and acted well within its
discretion in denying Astorgas motion to dismiss. SMARTs demand for payment
of the market value of the car or, in the alternative, the surrender of the car, is
not a labor, but a civil, dispute. It involves the relationship of debtor and creditor
rather than employee-employer relations. As such, the dispute falls within the
jurisdiction of the regular courts.
Replevin is a possessory action, the gist of which is the right of
possession in the plaintiff. The primary relief sought therein is the
return of the property in specie wrongfully detained by another person. It
is an ordinary statutory proceeding to adjudicate rights to the title or
possession of personal property. The question of whether or not a party
has the right of possession over the property involved and if so, whether
or not the adverse party has wrongfully taken and detained said property
as to require its return to plaintiff, is outside the pale of competence of a
labor tribunal and beyond the field of specialization of Labor Arbiters.
63.
General Milling Corporation vs. Violeta L. Viajar. G.R. No.
181738, January 30, 2013
GENERAL MILLING CORPORATION vs VIOLETA VIAJAR
Violeta Viajar, part of GMC's purchasing staff, was one of 13 employees
terminated due to redundancy. GMC claims that the company's profitability has
suffered due to economic setbacks and redundancy was part of its concrete and
actual cost reduction measures. She was given notice of such on October 30,2003,
termination to be effective on November 30, 2003.
However, as early as October 31, she was already barred from entering the
premises. She was also asked to sign an Application for Retirement and Benefits
on the premise that such was needed to process her separation pay. Viajar thus
filed this case for illegal dismissal against GMC and its HR Manager (Almocera)
and Purchasing Manager (Paulino).
ISSUE: w/n Viajar was validly terminated
HELD: NO, her termination is invalid.
Based on Art. 283 of the Labor Code, the ff. are the elements of a valid
redundancy program:
(a) the employer must serve a written notice to the affected employees and the
DOLE at least one
(1) month before the intended date of retrenchment;
(b) the employer must pay the employees a separation pay equivalent to at least
one month pay or
at least one month pay for every year of service,
whichever is higher;

(c) the employer must abolish the redundant positions in good faith; and
(d) the employer must set fair and reasonable criteria in ascertaining which
positions are redundant and may be abolished.
GMC was not able to prove its good faith, nor its act of fairly ascertaining which
positions are redundant and should be abolished. It merely presented the ff
proofs: Establishment Termination Report to the DOLE Office, 2 checks to Viajar
(P440,253.02 and P21,211.53), and the list of terminated EE's as of June 2006.
GMC did not:
evaluate existing positions and their effect on the company
present tangible proof (feasibility studies, audited financial statements,
etc) of its business slow down or over hiring
which would have successfully discharged its burden of proving valid dismissal.
On the other hand, it was proven by respondent that GMC was in bad faith when
it:

fired old EE's and hired new ones

prevented Viajar from entering work premises even before termination


was effective

compelled her to sign an application for retirement, which is completely


different from retrenchment

64.
Eastridge Golf Club v Eastridge Labor Union -SUPER, 563 SCRA
93 (2008)
65.

JPL Marketing Promotions v CA, 463 SCRA 136 (2005)

JPL Marketing Promotions v CA


Tinga, J.
FACTS: JPL Marketing and Promotions (hereinafter referred to as JPL) is a
domestic corporation engaged in the business of recruitment and placement of
workers. On the other hand, private respondents Noel Gonzales, Ramon Abesa III
and Faustino Aninipot were employed by JPL as merchandisers on separate dates
and assigned at different establishments in Naga City and Daet, Camarines
Norte as attendants to the display of California Marketing Corporation (CMC),
one of petitioners clients.
On 13 August 1996, JPL notified private respondents that CMC would stop its
direct merchandising activity in the Bicol Region, Isabela, and Cagayan Valley
effective 15 August 1996. They were advised to wait for further notice as they
would be transferred to other clients. However, on 17 October 1996, private
respondents Abesa and Gonzales filed before the NLRC complaints for illegal
dismissal, praying for separation pay, 13th month pay, and service incentive
leave pay and payment for moral damages. Aninipot filed a similar case
thereafter.
ISSUE: WON private respondents are entitled to separation pay, 13th month pay
and service incentive leave pay.
HELD: Respondents are NOT entitled to separation pay. However, they are
entitled to 13th month pay and service incentive leave pay.
48

RATIO: The common denominator of the instances where payment of separation


pay is warranted is that the employee was dismissed by the employer. In the
instant case, there was no dismissal to speak of. Private respondents were simply
not dismissed at all, whether legally or illegally. What they received from JPL
was not a notice of termination of employment, but a memo informing them of the
termination of CMCs contract with JPL. More importantly, they were advised
that they were to be reassigned. At that time, there was no severance of
employment to speak of.

66.
Sangwoo Philippines,
Employees Union-OLALIA,

Furthermore, Art. 286 of the Labor Code allows the bona fide suspension of the
operation of a business or undertaking for a period not exceeding six (6) months,
wherein employee/employees are placed on the so-called floating status. When
that floating status of an employee lasts for more than six months, he may be
considered to have been illegally dismissed from the service. Thus, he is entitled
to the corresponding benefits for his separation, and this would apply to
suspension either of the entire business or of a specific component thereof.

As clearly borne out by the records of this case, private respondents sought
employment from other establishments even before the expiration of the six (6)month period provided by law. As they admitted in their comment, all three of
them applied for and were employed by another establishment after they received
the notice from JPL. JPL did not terminate their employment; they themselves
severed their relations with JPL. Thus, they are not entitled to separation pay.
The Court is not inclined in this case to award separation pay even on the ground
of compassionate justice. The Court of Appeals relied on the cases wherein the
Court awarded separation pay to legally dismissed employees on the grounds of
equity and social consideration. Said cases involved employees who were actually
dismissed by their employers, whether for cause or not. Clearly, the principle
applies only when the employee is dismissed by the employer, which is not the
case in this instance. In seeking and obtaining employment elsewhere, private
respondents effectively terminated their employment with JPL.
Nonetheless, JPL cannot escape the payment of 13th month pay and service
incentive leave pay to private respondents. Said benefits are mandated by law
and should be given to employees as a matter of right. While computation for the
13th month pay should properly begin from the first day of employment, the
service incentive leave pay should start a year after commencement of service, for
it is only then that the employee is entitled to said benefit. On the other hand, the
computation for both benefits should only be up to 15 August 1996, or the last day
that private respondents worked for JPL. To extend the period to the date of
finality of the NLRC resolution would negate the absence of illegal dismissal, or
to be more precise, the want of dismissal in this case. Besides, it would be unfair
to require JPL to pay private respondents the said benefits beyond 15 August
1996 when they did not render any service to JPL beyond that date. These
benefits are given by law on the basis of the service actually rendered by the
employee, and in the particular case of the service incentive leave, is granted as a
motivation for the employee to stay longer with the employer. There is no cause
for granting said incentive to one who has already terminated his relationship
with the employer.

67.

Inc.

v.

Sangwoo

Philippines,

Inc.

Crayons Processing v Pula, GR No. 167727 (2007)

Crayons Processing Inc v. Felipe Pula and CA


July 30, 2007
Facts:

Petitioner Crayons Processing, Inc. (Crayons) employed respondent Felipe


Pula (Pula) as a Preparation Machine Operator beginning June 1993.
On 27 November 1999,Pula, then aged 34, suffered a heart attack and was
rushed to the hospital, where he was confined for around a week. Pulas wife
duly notified Crayons of her husbands medical condition
Upon his discharge from the hospital, Pula was advised by his attending
physician to take a leave of absence from work and rest for three (3)
months. Subsequently, on 25 February 2000, Pula underwent an Angiogram
Test at the Philippine Heart Center under the supervision of a Dr. Recto, who
advised him to take a two-week leave from work.
Following the angiogram procedure, respondent was certified as fit
to work by Dr. Recto. On 11 April 2000, Pula returned to work, but 13
days later, he was taken to the company clinic after complaining of
dizziness. Diagnosed as having suffered a relapse, he was advised by his
physician to take a leave of absence from work for one (1) month.
Pula reported back for work on 13 June 2000, armed with a certification from
his physician that he was fit to work. However, Pula claimed that he was
not given any post or assignment, but instead, on 20 June 2000, he was
asked to resign with an offer from Crayons of P12,000 as financial
assistance.Pula refused the offer and instead filed a complaint for
illegal dismissal
The respondents filed a Position Paper alleging that Pula had not been
dismissed at all, but had only been offered a less strenuous job. They prayed
that Pula be ordered to report for work without loss of seniority rights
LA ruled that Pula had been illegally dismissed and ordered reinstatement.
LA rejected Crayons contention that Pulas ailment was a proper reason to
dismiss him, he stressed that no evidence was presented to show that his
illness could not be cured within the period of six months. It was pointed out
that under Section 8, Rule I, Book VI of the Omnibus Rules Implementing
the Labor Code, implementing in particular Article 284 of the Labor Code,
termination on the ground of disease is prohibited unless there is a
certification by a competent public health authority that the disease
is of such nature or at such a stage that it cannot be cured within a
period of six months even with proper medical treatment
NLRC ruledthat there was valid cause to terminate Pulas employment
considering that he had a heart attack that kept him out of work for
more than six (6) months. According to the NLRC, the fact
that Pula was on leave for more than six months due to his illness
rendered unnecessary the certification from a public health
authority as required under the Omnibus Implementing Rules.
49

Court of Appeals annulled the NLRC Decision and reinstated the ruling of
the Labor Arbiter.

terminate the employee but shall ask the employee to take a leave. The
employer shall reinstate such employee to his former position immediately
upon the restoration of his normal health. (Emphasis supplied)

Issue #1: WON the court should give credence to the Calauag report [HR
Manager]? NO.

The report narrated that during the time Pula was purportedly
dismissed, Crayons had told him that it was willing to allow him to
return to work, provided that he undergo a medical examination by a
certain Dr. Ting, who was to prepare a certification as to his fitness to
return to work. After Pula had an initial consultation with Dr. Ting, he failed
to submit the medical findings prepared by the Philippine Heart Center
which would serve as basis for the medical certification. Instead, Pula filed
the instant complaint for illegal dismissal.
CA refused to acknowledge the CALAUAG REPORT since it was not
notarized.

For a dismissal on the ground of disease to be considered valid, two requisites


must concur:

This report emerged at first instance only in the proceedings before the
Court of Appeals. No reference was made to it before the Labor
Arbiter or the NLRC. The report is undated and unverified. It is addressed
to no one in particular, certainly not to any court or tribunal, and is not
accompanied by any motion or pleading seeking its admission as evidence. It
is hearsay in character.

Assuccinctly stressed in Tan v. NLRC it is only where there is a


prior certification from a competent public authority that the disease afflicting
the employee sought to be dismissed is of such nature or at such stage that it
cannot be cured within six (6) months even with proper medical treatment that
the latter could be validly terminated from his job.

Issue #2: WON the court should give credence to the Calauag report [HR
Manager]? NO.
The termination as upheld by the NLRC was grounded on Article 284 of the
Labor Code, which reads:
An employer may terminate the services of an employee who has been found to be
suffering from any disease and whose continued employment is prohibited by law
or is prejudicial to his health as well as to the health of his co-employees:
Provided, That he is paid separation pay equivalent to at least one (1) month
salary or to one-half (1/2) month salary for every year of service, whichever is
greater, a fraction of at least six (6) months being considered as one (1) whole
year.
The particular manner by which it is determined that the employee is suffering
from the disease of such character as expressed in Article 284 is in turn spelled
out in Section 8, Rule I, Book VI of the Omnibus Rules Implementing the Labor
Code, which provides:
Sec. 8. Disease as a ground for dismissal. Where the employee suffers
from a disease and his continued employment is prohibited by law or
prejudicial to his health or to the health of his co-employees, the employer
shall not terminate his employment unless there is a certification by a
competent public health authority that the disease is of such
nature or at such a stage that it cannot be cured within a period
of six (6) months even with proper medical treatment. If the disease
or ailment can be cured within the period, the employer shall not

(a) the employee must be suffering from a disease which cannot be cured
within six months and his continued employment is prohibited by law or
prejudicial to his health or to the health of his co-employees; and
(b) a certification to that effect must be issued by a competent public
health authority.
The burden falls upon the employer to establish these requisites,and in the
absence of such certification, the dismissal must necessarily be declared illegal.

Without the required certification, the characterization or even diagnosis of the


disease would primarily be shaped according to the interests of the parties rather
than the studied analysis of the appropriate medical professionals. The
requirement of a medical certificate under Article 284 cannot be
dispensed with; otherwise, it would sanction the unilateral and arbitrary
determination by the employer of the gravity or extent of the employee's
illness and thus defeat the public policy in the protection of labor.
The NLRCs conclusion that no such certification was required
since Pula had effectively been absented due to illness for more than six
(6) months is unsupported by jurisprudence and plainly contrary to the
language of the Implementing Rules. The indefensibility of such conclusion is
further heightened by the fact that Pula was able to obtain two
different medical certifications attesting to his fitness to resume work. Even
absent the certifications favorable to Pula, Crayons would still be unable
to justify his dismissal on the ground of ill health or disease, without the
necessary certificate from a competent public health authority.
CA Affirmed. Petition DENIED.
68.

Wuerth Philippines Inc. v Rodante Ynson GR No. 175932 (2012)

Wuerth Philippines v. Rodante Ynson (2012)

50

Wuerth PH hired Rodanty Ynson in 2001 as a National Sales Manager for the
Automotive business. His work required him to travel throughout the country as
well as supervise sales managers. In January 2003, he suffered a stroke and was
confined in Davao Doctors Hospital. Medical certification was provided by his
doctors to the company. He underwent therapy and rehabilitation for six months
until June 2003 and was advised not to travel a fact which he communicated to
Wuerth. Nevertheless, he was called to the Manila for an investigation on several
occasions, the last being August 2003, for unexplained absences since January
2003 and abandonment of work. He was later terminated.

Padillo v Rural Bank of Nabunturan


January 21, 2013 | PERLAS-BERNABE, J.
FACTS:

ACTION: Illegal dismissal. DEFENSE: He was not terminated because of his


disease, but due to gross dereliction of duties. CA: Ruled that Rodante did not
comply with medical certificate requirement which is to be offset with his right
under law to take a 6 month leave of absence to recover from illness.

LAW:

Section 8, Rule I, Book VI of the Omnibus Rules Implementing the Labor Code
requires that:
Disease as a ground for dismissal. Where the employee suffers from a disease
and his continued employment is prohibited by law or prejudicial to his health or
to the health of his co-employees, the employer shall not terminate his
employment unless there is a certification by a competent public health authority
that the disease is of such nature or at such a stage that it cannot be cured within
a period of six (6) months even with proper medical treatment. If the disease or
ailment can be cured within the period, the employer shall not terminate the
employee but shall ask the employee to take a leave. The employer shall reinstate
such employee to his former position immediately upon the restoration of his
normal health.
SC: NOT ILLEGALLY DISMISSED. He failed to provide competent evidence that
he was unable to return to work due to medical conditions. Ynsons conduct shows
his indifference and utter disregard of his work and his employer's interest, and
displays his clear, deliberate, and gross dereliction of duties. COURTS
FINDINGS:
1.
2.

He was already cleared to return to work by June 2003, although he was


advised to continue with his rehab regimen for at least 1 more months,
and not to travel.
He failed to attend the investigation even as late as August 2003 when
he already completed the regimen prescribed by the doctor.

Other issues: (1) he was management employee and company has more leeway
with respect to termination than with rank and file (2) The period from Jan to
June 2003 should be considered sick leave and he is entitled to salaries charged to
leave or whatever company policy may be. (3) also a long discussion on damages
awarded.
69.
Eleazar S. Padillo vs. Rural Bank of Nabunturan, Inc., et al. G.R.
No. 199338.
Termination of Employment>Authorized Causes>Disease

Petitioner, the late Eleazar Padillo (Padillo), was employed by respondent Rural Bank of Nabunturan, Inc. (Bank) as its SA Bookkeeper
Due to liquidity problems, the Bank took out retirement/insurance plans
with Philam Life for all its employees in anticipation of its possible closure and the concomitant severance of its personnel
Respondent Mark Oropeza (Oropeza), the President of the Bank, bought
majority shares of stock in the Bank and took over its management
which brought about its gradual rehabilitation. The Banks finances improved and eventually, its liquidity was regained
Later, Padillo suffered a mild stroke due to hypertension which consequently impaired his ability to effectively pursue his work. He then wrote
a letter addressed to respondent Oropeza expressing his intention to avail
of an early retirement package. Despite several follow-ups, his request
remained unheeded.
Subsequently, Padillo was separated from employment due to his poor
and failing health. Padillo filed a complaint for the recovery of unpaid retirement benefits. Respondents countered that the claim of Padillo for retirement benefits was not favorably acted upon for lack of any basis to
grant the same.
LA dismissed Padillos complaint (it found Padillo disqualified to receive
any benefits under LC Art 300 as he was only 55 y.o. when he resigned,
while the law specifically provides for an optional retirement age of 60
and compulsory retirement age of 65) but directed the Bank to pay him
the amount of P100,000.00 as financial assistance, treated as an advance
from the amounts receivable under the Philam Life Plan.
NLRC, applying LC Art 297 (provision on termination on the ground of
disease), set aside LAs ruling and ordered payment of separation pay, on
top of the Philam Life Plan benefit.
CA reinstated LAs decision with modification (directed payment of an
additional 50K as financial assistance). It held that Padillo could not, absent any agreement with the Bank, receive any retirement benefits pursuant to Article 300; evidence to prove that the Bank has an existing
company policy of granting retirement benefits to its aging employee was
insufficient and; (citing the case of Villaruel v. Yeo Han Guan) separation
pay on the ground of disease under LC Art 297 should not be given to
Padillo because he was the one who initiated the severance of his employment
Hence this petition (Padillo was now substituted by his legal heirs due to
his death)

ISSUE/HELD:
1.

W/N respondent is entitled to separation pay based on Labor Codes provision on termination on the ground of disease? NO.

51

2.

W/N respondent is entitled to retirement benefits? NO. Nevertheless,


CAs decision was MODIFIED, increasing the award of financial assistance of P50K to P75K, exclusive of the P100K benefit under the Phil am
Life Plan.

Art. 300. Retirement. Any employee may be retired upon reaching


the retirement age established in the collective bargaining agreement or
other applicable employment contract.
In case of retirement, the employee shall be entitled to receive such
retirement benefits as he may have earned under existing laws and any
collective bargaining agreement and other agreements: Provided,
however, That an employee's retirement benefits under any collective
bargaining and other agreements shall not be less than those provided
herein.
In the absence of a retirement plan or agreement providing for
retirement benefits of employees in the establishment, an employee upon
reaching the age of sixty (60) years or more, but not beyond sixty-five
(65) years which is hereby declared the compulsory retirement age, who
has served at least five (5) years in the said establishment, may retire
and shall be entitled to retirement pay equivalent to at least one-half
(1/2) month salary for every year of service, a fraction of at least six (6)
months being considered as one whole year.
Unless the parties provide for broader inclusions, the term one half (1/2)
month salary shall mean fifteen (15) days plus one-twelfth (1/12) of the
13th month pay and the cash equivalent of not more than five (5) days of
service incentive leaves.

RATIO:
1.

Labor Code provision on termination on the ground of disease under Article


297 does not apply in this case, considering that it was the petitioner and not
the Bank who severed the employment relations. It necessarily follows that
petitioners claim for separation pay anchored on such provision must be denied.
As borne from the records, the clear import of Padillos letter and the fact
that he stopped working before the foregoing date and never reported for
work even thereafter, show that it was Padillo who voluntarily retired
and that he was not terminated by the BankAs held in Villaruel, a
precedent which the CA correctly applied, LC Art 297 contemplates a
situation where the employer, and not the employee, initiates the
termination of employment on the ground of the latters disease or
sickness This is precisely the reason why Section 8, Rule 1, Book VI of the
Omnibus Rules Implementing the Labor Code, directs that an employer shall
not terminate the services of the employee unless there is a certification by a
competent public health authority that the disease is of such nature or at
such a stage that it cannot be cured within a period of 6 months even with
proper medical treatment.
Note: SC also said that NLRCs application of Abaquin was gravely misplaced
- a careful reading of Abaquin shows that the Court merely awarded
termination pay on the ground of disease in favor of security guard Antonio
Jose because he belonged to a "special class of employees deprived of the right
to ventilate demands collectively. Thus, notwithstanding the fact that it was
Antonio Jose who voluntarily resigned because of his sickness and it was not
the security agency which terminated his employment, the Court held that
Jose "deserve[d] the full measure of the laws benevolence" and still granted
him separation pay because of his situation

2.

In the absence of any applicable contract or any evolved company policy,


Padillo should have met the age and tenure requirements set forth under Article 300* of the Labor Code to be entitled to the retirement benefits provided
therein. Unfortunately, while Padillo was able to comply with 5 year tenure
requirement as he served for 29 years he, however, fell short with respect
to the 60 year age requirement given that he was only 55 y.o. when he retired. Therefore, without prejudice to the proceeds due under the Philam Life
Plan, petitioners claim for retirement benefits must be denied.
Nevertheless, SC, in light of the dictates of social justice, concurred with the
CA that financial assistance should be awarded and at an increased amount.

*Article 300 of the Labor Code as amended by Republic Act Nos. 764132 and
855833 partly provides:

70.

lnguillo v First Phil. Scales Inc. GR No. 165407 (2009)

Herminigildo Inguillo and Zenaida Bergante vs. First Philippine Scales, Inc.
(FPSI) and/or Amparo Policarpio, manager
G.R. No. 165407 (June 5, 2009)
In 1991, FPSI and First Philippine Scales Industries Labor Union (FPSILU)
entered into a Collective Bargaining Agreement (CBA) for a period of five (5)
years in a document entitled RATIPIKASYON NG KASUNDUAN. Bergante and
Inguillo, who were members of FPSILU, signed the said document.
Bergante, Inguillo and several FPSI employees joined another union,
the Nagkakaisang Lakas ng Manggagawa (NLM). [The latter] filed with the
Department of Labor and Employment (DOLE) an intra-union dispute against
FPSILU and FPSI. Meanwhile, on March 29, 1996, the executive board and
members of the FPSILU addressed a document dated March 18, 1996
denominated as Petisyon to FPSI's general manager, Amparo Policarpio
(Policarpio), seeking the termination of the services of [several employees,
including herein petitioners. This was granted upon by FPSI, which terminated,
among others, herein petitioners.]
In their Petition, Bergante and Inguillo assail the legality of their termination
based on the Union Security Clause in the CBA between FPSI and FPSILU.
[(1) Was there a valid ground for termination?
(2) Was there compliance with the procedural due process to the termination?]
(1) Yes. The Labor Code of the Philippines has several provisions under which an
employee may be validly terminated, namely: (1) just causes under Article
52

282; (2) authorized causes under Article 283; (3) termination due to disease under
Article 284; and (4) termination by the employee or resignation under Article
285. While the said provisions did not mention as ground the enforcement of the
Union Security Clause in the CBA, the dismissal from employment based on the
same is recognized and accepted in our jurisdiction.
Union security is a generic term, which is applied to and comprehends closed
shop, union shop, maintenance of membership or any other form of
agreement which imposes upon employees the obligation to acquire or retain
union membership as a condition affecting employment. There is union shop
when all new regular employees are required to join the union within a certain
period as a condition for their continued employment. There is maintenance of
membership shop when employees, who are union members as of the effective
date of the agreement, or who thereafter become members, must maintain union
membership as a condition for continued employment until they are promoted or
transferred out of the bargaining unit or the agreement is terminated.[40] A
closed-shop, on the other hand, may be defined as an enterprise in which, by
agreement between the employer and his employees or their representatives, no
person may be employed in any or certain agreed departments of the enterprise
unless he or she is, becomes, and, for the duration of the agreement, remains a
member in good standing of a union entirely comprised of or of which the
employees in interest are a part.[
Bergante and Inguillo assail the legality of their termination based on the Union
Security Clause in the CBA between FPSI and FPSILU. Article II[42] of the CBA
pertains to Union Security and Representatives, which provides:
The Company hereby agrees to a UNION SECURITY [CLAUSE] with the
following terms:
1. All bonafide union members x x x x shall, as a condition to their
continued employment, maintain their membership with the UNION;
xxx
5. Any employee/union member who fails to retain union membership in
good standing may be recommended for suspension or dismissal by the
Union Directorate and/or FPSILU Executive Council x x x
Verily, the aforesaid provision requires all members to maintain their
membership with FPSILU during the lifetime of the CBA. Failing so, and for any
of the causes enumerated therein, the Union Directorate and/or FPSILU
Executive Council may recommend to FPSI an employee/union member's
suspension or dismissal. Records show that Bergante and Inguillo were former
members of FPSILU based on their signatures in the document which ratified the
CBA. It can also be inferred that they disaffiliated from FPSILU when the CBA
was still in force and subsisting, as can be gleaned from the documents relative to
the intra-union dispute between FPSILU and NLM-KATIPUNAN. In view of
their disaffiliation, as well as other acts allegedly detrimental to the interest of
both FPSILU and FPSI, a Petisyon was submitted to Policarpio, asking for the

termination of the services of employees who failed to maintain their Union


membership.
In terminating the employment of an employee by enforcing the Union Security
Clause, the employer needs only to determine and prove that: (1) the union
security clause is applicable; (2) the union is requesting for the enforcement of the
union security provision in the CBA; and (3) there is sufficient evidence to
support the union's decision to expel the employee from the union or company. All
the requisites have been sufficiently met and FPSI was justified in enforcing the
Union Security Clause.
The stipulations in the CBA authorizing the dismissal of employees are of equal
import as the statutory provisions on dismissal under the Labor Code, since a
CBA is the law between the company and the Union, and compliance therewith is
mandated by the express policy to give protection to labor. In Caltex Refinery
Employees Association (CREA) v. Brillantes, the Court expounded on the
effectiveness of union security clause when it held that it is one intended to
strengthen the contracting union and to protect it from the fickleness or perfidy of
its own members. For without such safeguards, group solidarity becomes
uncertain; the union becomes gradually weakened and increasingly vulnerable to
company machinations. In this security clause lies the strength of the union
during the enforcement of the collective bargaining agreement. It is this clause
that provides labor with substantial power in collective bargaining.
(2) No. Nonetheless, while We uphold dismissal pursuant to a union security
clause, the same is not without a condition or restriction. The enforcement of
union security clauses is authorized by law, provided such enforcement is not
characterized by arbitrariness, and always with due process. There are two (2)
aspects which characterize the concept of due process under the Labor Code: one
is substantivewhether the termination of employment was based on the
provisions of the Labor Code or in accordance with the prevailing jurisprudence;
the other is procedural - the manner in which the dismissal was effected.
Procedural due process in the dismissal of employees requires notice and hearing.
The employer must furnish the employee two written notices before termination
may be effected. The first notice apprises the employee of the particular acts or
omissions for which his dismissal is sought, while the second notice informs the
employee of the employers decision to dismiss him. The requirement of a hearing,
on the other hand, is complied with as long as there was an opportunity to be
heard, and not necessarily that an actual hearing was conducted.
In the present case, the required two notices that must be given to herein
petitioners Bergante and Inguillo were lacking. Respondents, however, aver that
they had furnished the employees concerned, including petitioners, with a copy of
FPSILU's Petisyon. While the Petisyon enumerated the several grounds that
would justify the termination of the employees mentioned therein, yet such
document is only a recommendation by the Union upon which the employer may
base its decision. It cannot be considered a notice of termination. A perusal of
each of [the grounds stated therein] leads Us to conclude that what was stated
were general descriptions, which in no way would enable the employees to
intelligently prepare their explanation and defenses.
53

Policarpio's allegations are self-serving. Except for her claim as stated in the
respondent's Position Paper, nowhere from the records can We find that Bergante
and Inguillo were accorded the opportunity to present evidence in support of their
defenses. Policarpio relied heavily on the Petisyon of FPSILU. She failed to
convince Us that during the dialogue, she was able to ascertain the validity of the
charges mentioned in the Petisyon. In her futile attempt to prove compliance
with the procedural requirement, she reiterated that the objective of the dialogue
was to provide the employees the opportunity to receive the act of grace of FPSI
by giving them an amount equivalent to one-half () month of their salary for
every year of service. We are not convinced. We cannot even consider the demand
and counter-offer for the payment of the employees as an amicable settlement
between the parties because what took place was merely a discussion only of the
amount which the employees are willing to accept and the amount which the
respondents are willing to give. Such non-compliance is also corroborated by
Bergante and Inguillo in their pleadings denouncing their unjustified dismissal.
In fine, We hold that the dialogue is not tantamount to the hearing or
conference prescribed by law.

dismissal. When the results came out, 7 tested positive and were suspended. Of
the 7, only 2 were allowed back to work. The other 5 were not admitted by AER
without first submitting a fit-to-work certificate. While they were securing their
fit-to-work certificates, AER charged them with insubordination and being
AWOL. They complied with the demand for a written explanation but were not
reinstated.

71.
NUWHRAIN-APL-IUF Dusit Hotel Nikko Chapter v. Court of
Appeals

Illegal acts of AER:

72.

Alex Q. Naranjo, et al. vs. Biomedica Health Care, Inc., et al.

73.

Automotive Engine Rebuilders, GR 160138, 16 Jan 2013

Automotive Engine Rebuilders vs Progresibong UNYON ng mga


Manggagawa sa AER
Mendoza, J.
Facts:

AER Is a company engaged in the automotive engine repair and


rebuilding business. Progresibong Unyon (UNYON) is the legitimate
labor union of the rank and file employees of AER.
Both parties filed complaints against each other. AER accused UNYON
of illegal concerted activities while UNYON accused AER of unfair labor
practice, illegal suspension and illegal dismissal.

AERs version: 18 employees suddenly and without reason staged a walkout and
assembled illegally in the company premises. They then proceeded to the AER
Performance and Service Center (AER-PSC) located on another street and
collectively tried to cart away an equipment. They also urged other employees to
stop working. They occupied the AER-PSC premises for several hours and only
left when the police intervened. They were issued a memo to explain the illegal
workout, finding their explanation unsatisfactory, AER terminated them.
Almost a month later, they started a wildcat strike, barricaded company
premises, and prevent free ingress and egress of employees, clients and visitors.
They only stopped when the NLRC issued a TRO.
UNYONs version: UNYON filed a petition for certification election before the
DOLE. Resenting what they did, AER forced all of its employees to submit their
urine samples for drug testing. Those who refused were threatened with

Meanwhile, UNYON found out that AER was trying to engage in a runaway shop
by transferring equipment to the AER-PSC. They prevented the transfer. AER
accused those involved with gross insubordination and work stoppage. The
affected workers were denied entry so they picketed in front of AER premises.
Issue: Whether or not strike was illegal? Held: Yes, however, the workers
temporarily walked out of their jobs because they strongly believed that
management was committing an unfair labor practice.
Both parties are in pari delicto, they should be restored to their respective
positions prior to the illegal strike and illegal lockout
AERs fault is obvious from the fact that a day after the union filed a petition for
certification election before the DOLE, it hit back by requiring all its employees to
undergo a compulsory drug test. It was AERs first drug test in 35 years so the
timing was suspicious. Morever, AER failed to show proof that the drug test
conducted on its employees was performed by an authorized drug testing center.
Furthermore, AER engaged in a runaway shop when it began pulling out
machines from the main AER building to the AER-PSC compound. AER
committed another infraction when it refused to admit back those employees who
were not included in its complaint against the union.
Illegal acts of UNYON:
The affected workers were also at fault for resorting to a concerted work
slowdown and walking out of their jobs of protest for their illegal suspension. It
was also wrong for them to have forced their way to the AER-PSC premises to try
to bring out the boring machine. The picketing employees prevented the entry
and exit of non-participating employees and possibly AERs clients. Although the
unions sudden work stoppage lasted a day, it surely caused serious disturbance
and tension within AERs premises and could have adversely affected AERs
clients and business in general.
In Pari Delicto:
The findings show that both parties are in pari delicto, a situation which
warrants the maintenance of the status quo. This means that the contending
parties must be brought back to their respective positions before the controversy;
that is, before the strike. Employees should be reinstated without backwages
Backwages (MR, 16, 2013):
UNYON filed partial MR for 14 of its employees who were reinstated without
backwages.

54

32 employees filed a complaint against AER for illegal suspension and unfair
labor practices but of the 32, only 18 were charged by AER with illegal strike,
excluding 14 from its complaint. Out of the 14, 5 failed to write their name and
affix their signature in the petition filed with the CA. Because of their failure to
affix their signature, the 5 are not entitled to the relief prayed for by UNYON.
Prayer granted as to the 9 employees who were covered by the Partial MR and
who signed the petition.
74.

Nacague v Suplicio lines Inc GR No. 172589 (2010)

Nacague v Sulpicio Lines

Nacague was already questioning the credibility of S.M. Lazo Clinic as


early as the proceedings before the Labor Arbiter. In fact, the Labor
Arbiter declared that the S.M. Lazo Clinic drug test result was doubtful
since it is not under the supervision of the Dangerous Drug Board.

However, we find that Sulpicio Lines failed to clearly show that Nacague
was guilty of using illegal drugs. We agree with the Labor Arbiter that
the lack of accreditation of S.M. Lazo Clinic made its drug test results
doubtful.

Section 36 of R.A. No. 9165 provides that drug tests shall be performed
only by authorized drug testing centers. Moreover, Section 36 also
prescribes that drug testing shall consist of both the screening test and
the confirmatory test.

The law is clear that drug tests shall be performed only by authorized
drug testing centers. In this case, Sulpicio Lines failed to prove that S.M.
Lazo Clinic is an accredited drug testing center

FACTS:

Sulpicio Lines received an anonymous letter reporting the use of illegal


drugs on board the ship.

Ceasar T. Chico, a housekeeper on the ship, submitted a report regarding


the drug paraphernalia found inside the Mopalla Suite Room and the
threat on his life made by Nacague and Chief Mate Reynaldo Doroon
after he found the drug paraphernalia.

On 15 February 2003, Sulpicio Lines sent a notice of investigation to


Nacague informing him of the charges against him for use of illegal
drugs and threatening a co-employee.
A random drug test was performed. The result of the random drug test
revealed that Nacague was positive for methamphetamine hydrochloride
or shabu.

On 20 February 2003, Sulpicio Lines subjected Nacague to a formal


investigation. Nacague denied using illegal drugs.

On 23 February 2003, Nacague went to Chong Hua Hospital in Cebu


City to undergo a voluntary drug test. The drug test with Chong Hua
Hospital yielded a negative result. Nacague submitted this test result to
Sulpicio Lines.

However, on 7 March 2003, Sulpicio Lines sent a memorandum to


Nacague terminating him from the service.

ISSUE: WON Nacague was validly terminated?


HELD/RATIO: NO

Under Article 279 of the Labor Code, an employer may terminate the
services of an employee for just causes or for authorized causes.
Furthermore, under Article 277(b) of the Labor Code, the employer must
send the employee who is about to be terminated, a written notice
stating the causes for termination and must give the employee the
opportunity to be heard and to defend himself. Thus, to constitute
valid dismissal from employment, two requisites must concur: (1)
the dismissal must be for a just or authorized cause; and (2) the
employee must be afforded an opportunity to be heard and to
defend himself.

75.

Baron v NLRC GR No. 182299 (2010)

BARON v NLRC (2010)


FACTS:
Magic Sales Inc (MSI) is a domestic corp engaged in the business of trading
consumer goods such as soap, biscuits, candy, coffee, and juice, among other
things. Jose Sy is president and GM.
On Jan 18, 2000, Sy ordered an inventory and audit after noticing a steady
increase in the companys payables and a decline in investments. In a memo on
Feb 18, 2000, employees were instructed to: 1) give all the support needed by the
audit team; 2) surrender all keys and documents; 3) not to bring out anything
belonging to management; and 4) undergo a search before leaving the office.
Subject employees, including Wilfredo Baron (operations manager) refused to
cooperate in the audit process and thereafter refrained from reporting for work.
According to audit team, there were several irregularities in the operations of
MSI. Barons accounting system was generally weak and compliance with
procedures was not strictly implemented. The team was likewise convinced that
there was a conspiracy among employees and Baron to benefit from the
irregularities.
Consequently, management informed employees of the charges against them of
serious misconduct and willful disobedience, fraud or willful breach of trust, and
abandonment.
Employees were required to explain and refute the charges. They neither
rebutted the same nor attended the investigation. MSI decided hence to
terminate their services.
ISSUES & RULING
1)

Were Wilfredo Baron, dela Rosas, Junatas, Ballesca, and Rabago validly
dismissed?
55

2)

3)

76.

YES. Despite managements memo, keys were not surrendered.


Baron pulled out some records without allowing the audit team to inspect
them and also deleted some files from the computer which could no
longer be retrieved.
Cynthia Junatas (another employee) was also seen carrying some
documents. When asked to present the documents, she refused and tore
them.
Conspiracy to orchestrate massive irregularities and grand scale fraud
were discovered by the audit team. Unauthorized taking of company
documents, failing to pay unremitted collections, failure to surrender
keys, concealment of shortages, and failure to record inventory
transactions pursuant to a fraudulent scheme are acts of grave
misconduct and grounds for loss of trust and confidence under Art 282 of
the Labor Code. Petitioner employees in this case held positions imbued
with trust and confidence.
Was there employment relationship for Puzon, Gemino, Mangsat, Barry
Baron, and Cayago?
NO. They admitted that they were probationary employees of Superb
Trading and Services (STSI) and not MSI. There was no proven
connection between STSI and MSI.
Were workers denied of their right to due process when they were
terminated?
NO. In dismissal of employees, the twin requirements of notice and
hearing are essential elements of due process. Employer must furnish
the worker with 2 written notices a notice apprising the employee of
the particular acts or omissions for which dismissal is sought and a
subsequent notice informing the employee of the employers decision to
dismiss him. As for hearing, the essence of due process lies simply in an
opportunity to be heard, and not that an actual hearing should always
and indispensably be held.
Likewise, there is no requirement that the notices should be couched in
the form and language of judicial or quasi-judicial decisions. What is
required is that the employer conduct a formal investigation process,
with notices duly served. Through the investigation process, employee
must be accorded the right to present his side, which must be considered
and weighed by the employer. The employee must be sufficiently
apprised of the nature of the charge, so as to be able to intelligently
defend himself against the charges.
In this case, the twin requirements of notice and hearing were complied
with. Notices were issued on various dates. Employees failed to respond
and did not attend the scheduled investigation. When parties have been
given an opportunity to be heard and to present their case, there is no
denial of due process.
Mantle Trading Services Inc. v NLRC GR No. 166705 (2009)

MANTLE TRADING SERVICES, INC. V. NLRC (CELEBRADO)


Mantle Trading Services, Inc. and/or Bobby Del Rosario vs. National
Labor Relations Commission and Pablo S. Madriaga

G.R. No. 166705 | 28 July 2009 | 1st Division | Puno, C.J.


FACTS:
Mantle Trading Services, Inc., is engaged in the fishing business. It hired
Madriaga as a batilyo or fish hauler. Subsequently, he became a tagapuno
(someone who filled up tubs with fish).
He was reported by Gallos (a fish broker) to have received money from a fish
trader, Alfaro. As consideration, Madriaga would put more fish in Alfaros
tubs. Formal incident reports were submitted to the company.
Madriaga was allegedly barred by the payroll master, Baqued, from reporting
for work. The company alleged that Madriaga abandoned his work when he was
about to be investigated for the incident reports.
Madriaga filed a complaint before the LA. The latter ruled that hes an illegally
dismissed regular employee. NLRC ruled that hes not illegally dismissed nor
abandoned his work. CA held that theres abandonment but because due process
requirement is not complied with, hes still illegally dismissed.
ISSUE:
Whether or not theres illegal dismissal (even though theres abandonment of
work) due to due process lapses? [No.]
HELD:
NO. It is settled that to effect a valid dismissal, the law requires that:
a) there be just and valid cause as provided under Article 282 of the
Labor Code; and,
b) the employee be afforded an opportunity to be heard and to defend
himself.
The two-notice requirement must be complied with, to wit:
a) a written notice containing a statement of the cause for the
termination to afford the employee ample opportunity to be heard and defend
himself with the assistance of his representative, if he so desires; and,
b) if the employer decides to terminate the services of the employee, the
employer must notify him in writing of the decision to dismiss him, stating clearly
the reason therefore.
The case of Agabon v. NLRC applies to the case at bar. In Agabon, the dismissal
was found by the Court to be based on a just cause because the employee
abandoned his work. But it also found that the employer did not follow the notice
requirement demanded by due process. It ruled that this violation of due process
on the part of the employer did not nullify the dismissal, or render it illegal, or
ineffectual. Nonetheless, the employer was ordered to indemnify the employee for
the violation of his right to due process. It further held that the penalty should be
in the nature of indemnification, in the form of nominal damages and should
depend on the facts of each case, taking into special consideration the gravity of
the due process violation of the employer. The amount of such damages is
56

addressed to the sound discretion of the court, considering the relevant


circumstances.
77.

Agabon v NLRC, 442 SCRA 573 (2004)

Agabon vs. NLRC Case Digest


Jenny Agabon & Virgilio Agabon vs. NLRC
G.R. No.158693
November 17, 2004
Facts: Private respondent Riviera Home Improvements, Inc. is engaged in the
business of selling and installing ornamental and construction materials. It
employed petitioners Virgilio Agabon and Jenny Agabon as gypsum board and
cornice installers on January 2, 1992 until February 23, 1999 when they were
dismissed for abandonment of work.
Petitioners then filed a complaint for illegal dismissal and payment of money
claims and on December 28, 1999, the Labor Arbiter rendered a decision declaring
the dismissals illegal and ordered private respondent to pay the monetary
claims.
Issue: Whether or not respondents dismissal is illegal and if not, entitles them
benefits.
Ruling: The Court ruled that the dismissal is legal and entitles them of payment
of benefits.
Dismissals based on just causes contemplate acts or omissions attributable to the
employee while dismissals based on authorized causes involve grounds under the
Labor Code which allow the employer to terminate employees. A termination for
an authorized cause requires payment of separation pay. When the termination of
employment is declared illegal, reinstatement and full back wages are mandated
under Article 279. If reinstatement is no longer possible where the dismissal was
unjust, separation pay may be granted.
Procedurally, (1) if the dismissal is based on a just cause under Article 282, the
employer must give the employee two written notices and a hearing or
opportunity to be heard if requested by the employee before terminating the
employment: a notice specifying the grounds for which dismissal is sought a
hearing or an opportunity to be heard and after hearing or opportunity to be
heard, a notice of the decision to dismiss; and (2) if the dismissal is based on
authorized causes under Articles 283 and 284, the employer must give the
employee and the Department of Labor and Employment written notices 30 days
prior to the effectivity of his separation.
From the foregoing rules four possible situations may be derived: (1) the dismissal
is for a just cause under Article 282 of the Labor Code, for an authorized cause
under Article 283, or for health reasons under Article 284, and due process was
observed; (2) the dismissal is without just or authorized cause but due process
was observed; (3) the dismissal is without just or authorized cause and there was
no due process; and (4) the dismissal is for just or authorized cause but due
process was not observed.

In the fourth situation, the dismissal should be upheld. While the procedural
infirmity cannot be cured, it should not invalidate the dismissal. However, the
employer should be held liable for non-compliance with the procedural
requirements of due process.
The present case squarely falls under the fourth situation. The dismissal should
be upheld because it was established that the petitioners abandoned their jobs to
work for another company. Private respondent, however, did not follow the notice
requirements and instead argued that sending notices to the last known
addresses would have been useless because they did not reside there anymore.
Unfortunately for the private respondent, this is not a valid excuse because the
law mandates the twin notice requirements to the employees last known address.
Thus, it should be held liable for non-compliance with the procedural
requirements of due process.
The Court ruled that respondent is liable for petitioners holiday pay, service
incentive leave pay and 13th month pay without deductions. The evident
intention of Presidential Decree No. 851 is to grant an additional income in the
form of the 13th month pay to employees not already receiving the same so as to
further protect the level of real wages from the ravages of world-wide inflation.
Clearly, as additional income, the 13th month pay is included in the definition of
wage under Article 97(f) of the Labor Code.
An employer is prohibited under Article 113 of the same Code from making any
deductions without the employees knowledge and consent.
78.

Jaka Food Processing Corp. v Pacot, G.R. No. 151378 (2005)

79.

Serrano v NLRC 323 SCRA 445 (2000)

Serrano v. NLRC and Isetann


FACTS:
-

Serrano was the head of security checkers of Isetann in charge of apprehending shoplifters and preventing pilferages
For reasons of retrenchment, he was given a letter informing him of the
immediate termination of his services
Isetann had opted to hire an independent security agency as a costcutting measure
Serrano filed a complaint for illegal dismissal, unfair labor practice, underpayment, notpayment of salary and OT pay
LA: Serrano was illegally dismissed:
Isetann failed to establish that it had retrenched its security division;
petitioner was not accorded due process
Isetann failed to employ a security supervisor with similar duties
NLRC: reversed
Phase-out was a valid exercise of management prerogative
Absence of bad faith or abuse of discretion
Security and safety supervisors position was long in place prior to his
separation or the phase-out

ISSUE(S):
57

w/n dismissal was valid

PUNO, dissenting

HOLDING:

Under the Wenphil doctrine, enforcement became a dismiss now, pay later
scheme, in circumvention of the due process requisites. It becomes convenient to
employers to dismiss summarily than to comply with the 30-day notice.
Therefore, dismissal, without compliance to procedural requirements renders the
dismissal illegal. Legislatie, Executive and Judicial Proceedings that deny due
process do so under the pain of nullity.

No, valid but ineffectual (without legal effect)


Dismissal in this case is due to an authorized cause under Art. 283 of LC, i.e.
redundancy. However, Isetann failed to follow the procedural requirement, that
is, to give a written notice of termination to the employee concerned and to the
DOLE at least 30 days prior to its ineffectivity.
What is the effect of failure to comply with the procedural requirement?
Previously, dismissal became invalid. However, in Wenphil Corp. v. NLRC, the
dismissal was held to be valid BUT the employer was sanctioned by way of
payment of indemnity thru damages depending on the circumstances such as
gravity of offense committed by employer. In this case, a reexamination of the
Wenphil doctrine is necessary. Here, dismissal of Serrano is ineffectual, not void.

PANGANIBAN, dissenting
Monetary sanction is too insignificant, niggardly, sometimes even late. Every
decision that denies due process renders the decisions and proceddings void for
lack of jurisdiction. Dismissal should be held illegal.
80.

Salaw v NLRC 202 SCRA 7 (1991)

1991 SALAW v NLRC and ASSOCIATED BANK et al

MAJORITY addressing dissenting of Puno and Panganiban:

1967 Salaw, was employed by Assoc Bank a credit investigator-appraiser

Violation of procedural requirement of termination is not a denial of the right to


due process.

His duties = inspecting, appraising company's foreclosed assets; giving valuation


to its real properties, and verifying the genuineness and encumbrances of the
titles of properties mortgaged to the bank

1.

2.

3.
4.

5.

The due process clause is a limitation on governmental powers, inapplicable to the exercise of private power, such as in this case. The provision
No person shall be deprived of life, liberty and property without due
process of law pertains only to the State, as only it has the authority to
do the same.
The purpose of the notice and hearing under the due process clause is to
provide an opportunity for the employee to be heard before the power of
the organized society is brought upon the individual. Under Art. 283,
however, the purpose is to give him time to prepare for the eventual loss
of his job and for DOLE to determine whether economic causes exist to
justify termination. It is not to give opportunity to be heard there is no
charge against the employee under Art. 283
The employer cannot be expected to be an impartial judge of his own
cause.
Not all notice requirements are requisites of due process. Some are simply a part of a procedure to be followed before a right granted to party
can be exercised; others are an application of the Justinian precept. Such
is the case here. The failure of the employer to observe a procedure for
the termination of employment which makes the termination of employment merely ineffectual.
Art. 279 of the LC provides that only dismissal without just or authorized cause renders such dismissal illegal. To consider termination without observing procedural requirements as also ID is to add another
ground for ID, thereby amending Art. 279.; further, there is a disparity
in legal treatment, as employees who resign without giving due notice
are only liable for damages; it does not make their resignation void.

In this case, the separation pay was a distinct award from the payment of
backwages as a way of penalty.

1984, the Criminal Investigation Service (CIS) of the Philippine Constabulary


extracted from the Salaw without the assistance of counsel a Sworn
Statement which made it appear that the petitioner, in cahoots with a coemployee, Madrigal, sold sewing machines and electric generators which had
been foreclosed by the bank from Worldwide Garment and L.P. Money Garment,
for P60,000.00, and divided the proceeds between the two of them.
When Salaw signified his readiness to appear before the banks Investigation
committee, he was sent a letter stating --You are requested to come to (venue of hearing) without
counsel or representative
Petitioner was thereafter terminated for alleged serious misconduct or willful
disobedience and fraud or willful breach of the trust
Petitioner filed complaint for illegal dismissal
LA dismissal was illegal
NLRC reversed and dismissed the case
ISSUE - WON dismissal was legal
Held: NO. No due process
Right to counsel violated. When he was called to present his defences
to the allegations against him, he was asked to come to the hearing
without counsel or representative
It is true that administrative and quasi-judicial bodies are not
bound by the technical rules of procedure in the adjudication
58

cases. However, the right to counsel, a very basic requirement of


substantive due process, has to be observed. Indeed, rights to
counsel and to due process of law are two of fundamental rights
guaranteed by the 1987 Constitution. Thus, Section 12(1), Article III
thereof specifically provides: "Any person under investigation for the
commssion of an offense shall have the right to ... have compete and
independent counsel preferably of his own choice. If the person cannot
afford the service of counsel, he must be provided with one. These rights
cannot be waived except in writing in the presence of counsel."

The supposed admission on which bank based his dismissal was


of no probative value because it was signed without assistance of
counsel.
Note: The ff are the Cardinal primary rights which must be respected
even in proceedings of administrative character as enunciated in the
classic landmark decision of Justice Laurel in Ang Tibay:
(1) right to a hearing, which includes the right of the party
interested or affected to present his own case and submit
evidence in support thereof

xxx

(4) Not only must there be some evidence to support a finding or


conclusion but the evidence must be "substantial." "Substantial
evidence is more than a mere scintilla. It means such relevant
evidence as a reasonable mind might accept adequate to support
a conclusion."

(5) The decision must be rendered on the evidence presented the


hearing, or at least contained in the record and disclosed to
parties affected.
(6) The NLRC or any of its judges, therefore, must act on its or
his own independent consideration of the law and facts of
controversy, and not simply accept the views of a subordinate
arriving at a decision
(7) NLRC should, in controversial questions, render its decision
in such a manner that parties to the proceeding can know the
various issues involved, the reasons for the decisions rendered.
The performance of this duty inseparable from the authority
conferred upon it.
LA reinstated. Dismissal illegal.
81.

King of Kings Transport v MAmac, GR No. 166208 (2007)

King of Kings Transport v. Mamac


J. Velasco
Facts:

Petitioner KKTI is a corporation engaged in public transportation while


respondent is a conductor of the KKTI.
Upon audit of the October 28, 2001 Conductors Report of respondent,
KKTI noted an irregularity. It discovered that respondent declared
several sold tickets as returned tickets causing KKTI to lose an income of
eight hundred and ninety pesos.
While no irregularity report was prepared on the October 28, 2001
incident, KKTI nevertheless asked respondent to explain the
discrepancy. In his letter, respondent said that the erroneous declaration
in his October 28, 2001 Trip Report was unintentional. He explained that
during that days trip, the windshield of the bus assigned to them was
smashed; and they had to cut short the trip in order to immediately
report the matter to the police.
As a result of the incident, he got confused in making the trip report.
On November 26, 2001, respondent received a letter terminating his
employment effective November 29, 2001. The dismissal letter alleged
that the October 28, 2001 irregularity was an act of fraud against the
company.
KKTI also cited as basis for respondents dismissal the other offenses he
allegedly committed since 1999.
On December 11, 2001, respondent filed a Complaint for illegal
dismissal, illegal deductions, nonpayment of 13th-month pay, service
incentive leave, and separation pay. He denied committing any infraction
and alleged that his dismissal was intended to bust union activities.
Moreover, he claimed that his dismissal was effected without due
process.
In its April 3, 2002 Position Paper, KKTI contended that respondent was
legally dismissed after his commission of a series of misconducts and
misdeeds. It claimed that respondent had violated the trust and
confidence reposed upon him by KKTI. Also, it averred that it had
observed due process in dismissing respondent and maintained that
respondent was not entitled to his money claims such as service incentive
leave and 13th-month pay because he was paid on commission or
percentage basis.
Labor Arbiter dismissed respondents Complaint for lack of merit.
NLRC modified the decision and awarded P10,000 to respondent for
plaintiffs failure to comply with due process prior to termination.
CA upheld the dismissal but also sustained that petitioners failed to
comply with the required procedural due process prior to respondents
termination.
It also modified the award of P10,000 as indemnification by awarding full
backwages from the time respondents employment was terminated until
finality of the decision. Moreover, the CA held that respondent is entitled
to the 13th-month pay benefit.

Issues:
1.
2.

WON KKTI did not comply with the requirements of procedural due
process before dismissing the services of the respondent.
WON CA erred in awarding in favor of the respondent, full back wages
59

3.

WON CA rendered an incorrect decision in awarding 13th month pay


benefits to respondent.

Held:
1.

2.

3.

82.

Yes, KKTI did not comply with the due process requirement.
First, respondent was not issued a written notice charging him of
committing an infraction. The law is clear on the matter. A verbal
appraisal of the charges against an employee does not comply with the
first notice requirement. In Pepsi Cola Bottling Co. v. NLRC, the Court
held that consultations or conferences are not a substitute for the actual
observance of notice and hearing. Also, in Loadstar Shipping Co., Inc. v.
Mesano, the Court, sanctioning the employer for disregarding the due
process requirements, held that the employees written explanation did
not excuse the fact that there was a complete absence of the first notice.
Second, even assuming that petitioner KKTI was able to furnish
respondent an Irregularity Report notifying him of his offense, such
would not comply with the requirements of the law. We observe from the
irregularity reports against respondent for his other offenses that such
contained merely a general description of the charges against him. The
reports did not even state a company rule or policy that the employee
had allegedly violated. Likewise, there is no mention of any of the
grounds for termination of employment under Art. 282 of the Labor
Code. Thus, KKTIs standard charge sheet is not sufficient notice to the
employee.
Third, no hearing was conducted. Regardless of respondents written
explanation, a hearing was still necessary in order for him to clarify and
present evidence in support of his defense. Moreover, respondent made
the letter merely to explain the circumstances relating to the irregularity
in his October 28, 2001 Conductors Trip Report. He was unaware that a
dismissal proceeding was already being effected. Thus, he was surprised
to receive the November 26, 2001 termination letter indicating as
grounds, not only his October 28, 2001 infraction, but also his previous
infractions.
Yes, the award of backwages is in error. The CA awarded full backwages
in favor of respondent in accordance with the doctrine in Serrano v.
NLRC. However, the doctrine in Serrano had already been abandoned in
Agabon v. NLRC by ruling that if the dismissal is done without due
process, the employer should indemnify the employee with nominal
damages.
Yes, the award of 13th month pay is also in error. Respondent is paid by
commission only and this fact is supported by his payslips. Thus he is
ecluded from receiving the 13th-month pay benefit.

Magro Placement and General Services is the local agency of Orbit


Recruitment Office of Jeddah, Kingdom of Saudi Arabia (K.S.A.).

November 1999: Cresenciano E. Hernandez then an Aircon Electrical


Technician of Toyota Pasong Tamo, Inc filed an application for
employment abroad as Auto Electrician or Air-Conditioning Technician

Cresenciano was successfully hired as Auto Electrician of Al Yamama


Est. (Al Yamama) in Jeddah, K.S.A. for a two-year contract with a basic
monthly salary of US$450.00 for 10 hours a day, 6 days a week regular
working hours, 15 days vacation leave and 15 days sick leave with full
pay per year of service, and free food allowance of US$50.00 a month
with free suitable housing. He resigned from TOYOTA.

While in Jeddah, he encountered difficulties in working on American cars


and failed a trade test using an American car.

January 30 Statement: Other problems: Communication barriers (cant


speak Arabic), not enough equipment and manuals, isufficient food
allowance, no electric fans

Feb 10 Statement: He expressed desire to change employers.

Feb 16 Statement: recruited for Al Yamama as Auto Electrician, but he


was not qualified since he had no experience as Auto Electrician; he was
allowed to go for a trade test but failed; he was allowed to find a new job,
but he was not qualified to work in Budget Rent-A-Car Company &
Nissan; he had no complaints against his agent in the K.S.A. or his
Philippine agent, herein petitioner; and respondent requested that he be
sent back to the Philippines as early as possible

March 2000: He was repatriated to the Phil, given only 2k as financial


assistance

He filed complaint for illegal dismissal.

LA: NO ID- He was ill-equipped to work as an Auto Electrician for


American cars, per his revelation and admission; he asked to be
repatriated, without any complaint against his foreign employer or
agency

BUT: granted pay of US$185.00, or its peso equivalent, for the


15-day period (January 16 to 30, 2000) when he worked with Al Yamama

NLRC affirmed LA

CA reversed: Yes there was just cause for dismissal but, such dismissal
was ineffectual, since it did not comply with the due process
requirements. It held Magro liable for backwages from the time
respondent was terminated until it is determined that said termination
is for just cause.

Magro Placement v Hernandez GR No. 156964 (2007)

MAGRO PLACEMENT AND GENERAL SERVICES VS CRESENCIANO E.


HERNANDEZ
FACTS:

ISSUE: W/N respondent was accorded procedural due process before his
separation from work. NO
HELD
60

83.

RULE: employer has the burden of proving that the dismissed worker
has been served two notices: (1) the first to inform the employee of the
particular acts or omissions for which the employer seeks his dismissal;
and (2) the second to inform the employee of his employers decision to
terminate him.

The first notice must state that the employer seeks dismissal for the act
or omission charged against the employee, otherwise, the notice does not
comply with the rules.

Al Yamama failed to satisfy the two-notice requirement. Without prior


notice or explanation, Al Yamama took EEs passport and simply brought
him to Magros foreign principal, Orbit, and told the latter that EE did
not know his job as electrician. EE heard his employer's complaint
against him at that instance only.

UNDER Agabon v. NLRC: the employers violation of the employees


right to statutory due process warrants the payment of indemnity in the
form of nominal damages. (amount addressed to the sound discretion of
the Court, taking into account the relevant circumstances)

Serrano doctrine which awarded full backwages in ineffectual dismissal


cases where an employee dismissed for cause was denied due process,
which was applied by the CA, has been abandoned.

SO: SC granted amount of P30,000.00 as nominal damages for failure to


comply fully with the notice requirement as part of due process, in
addition to payment of respondent's one half month salary in the sum of
US$185.00 or its peso equivalent
Erector Advertising Sign Inc v Cloma, GR No. 167218 (2010)

(Clavacio), personnel and production manager of petitioner


company, and approved by Architect Jimmy C. Amoroto
(Amoroto), president and chief executive officer.
o The former happened between May 12 and May 15,
2000 when Cloma supposedly failed to report for work
without prior notice and prior leave approval which
thus effectively prevented the other workers from being
transported to the job site as there was no other driver
available.
o The latter incident happened on May 11, 2000 when
allegedly, Cloma, without authority, suddenly barged
into the premises of the Outright Division and, without
being provoked, threatened the employees with bodily
harm if they did not stop from doing their work.
When he returned for work, the security guard prevented him
from entering the companys premises and handed him a
termination letter dated May 20, 2000.
Cloma was dismissed in May 2000.
He filed a complaint with the NLRC for illegal suspension,
illegal dismissal, and other unpaid monetary benefits.
LA: dismissed complaint for lack of merit
NLRC: reversed LA
CA: affirmed NLRC

Issue/Holding/Ratio
WON Cloma was dismissed with just cause and with due process of law?
No. Petitioner did not comply with the rules of procedural due process.
Moreover, there was no just cause.

Erector Advertising Sign Group v. NLRC


July 2, 2010; GR No. 167218; Peralta, J.

Doctrine: Suspension orders, however, hardly constitute the


first notice required by law prior to termination. Here is why: a
fleeting glance at these two orders readily reveals that the
alleged offenses mentioned therein were not to be used as
grounds for termination, but rather merely for suspension.
Facts

Petitioner Erector Advertising Sign Group, Inc. is engaged in


the business of constructing billboards and advertising signs.
Petitioner engaged the services of Expedito Cloma (Cloma) in
1996 as company driver.
Petitioner served on Cloma two (2) Suspension Orders dated
May 15, 2000 and May 17, 2000, both signed by Nelson Clavacio

With respect to due process requirement, the employer is bound to


furnish the employee concerned with two (2) written notices before
termination of employment can be legally effected. One is the
notice apprising the employee of the particular acts or omissions
for which his dismissal is sought and this may loosely be considered
as the proper charge. The other is the notice informing the
employee of the managements decision to sever his employment.
o This decision, however, must come only after the
employee is given a reasonable period from receipt of the
first notice within which to answer the charge, thereby giving
him ample opportunity to be heard and defend himself with the
assistance of his representative should he so desire.
Clomas dismissal from service did not comply with this basic precept.
The notice of termination served by petitioner on Cloma cites three
reasons why the management has arrived at the decision to dismiss him
from service: first, his absence from work for two (2) days without prior
61

notice and approval; second, his act of barging into the premises of the
Outright Division and threatening the members of the said division with
bodily harm if they did not stop doing their work; and third, his frequent
tardiness in reporting for work.
The records do not contain any suggestion that petitioner, with
respect to these three grounds with which Cloma is charged, has
tried to notify the latter of the said charges. What is clear from the
records is that the only notice that was given to Cloma prior to his
termination is the May 20, 2000 notice of termination informing him
that his employment in the company has been severed for the causes
mentioned.
Petitioner insists that Cloma has been sufficiently informed of the acts
constituting the grounds for his termination by referring to the May 15
and May 17, 2000 Suspension Orders which it previously served on
Cloma. Suspension orders, however, hardly constitute the first
notice required by law prior to termination. Here is why: a
fleeting glance at these two orders readily reveals that the
alleged offenses mentioned therein were not to be used as
grounds for termination, but rather merely for suspension.
Moreover, the May 15, 2000 Order, in particular, could not have
constituted the first notice relative to the charge that Cloma has incurred
unauthorized absences for two days as stated in the notice of
termination. This, inasmuch as the order refers to a four (4)day absence
supposedly incurred between May 12, 2000 and May 15, 2000 for which
Cloma has actually been sanctioned with suspension.
The same is true with the third ground of termination, i.e., that Cloma
has frequently been late in reporting for work. Observably, aside from
the fact that Cloma, with respect to this ground, has not been furnished a
pre-dismissal notice, the notice of termination does not state the
inclusive dates on which Cloma actually reported late for his work.
Moreover, petitioner has not overcome the quantum of
substantial evidence needed to establish the existence of just
causes for dismissal in this case.
o With respect to the charges of frequent tardiness and incurring
an unauthorized two-day leave of absence, it is plain in the
records that the same have not been sufficiently proved by
petitioner.
o For one, petitioner could not identify the dates when
Cloma incurred the alleged tardiness in reporting for work.
Add to that the fact that Clomas daily time records, which
would have been the best evidence on the matter, have
not been made of record when they are actually within
petitioners power to produce and submit at the trial. The same
applies to the charge of unauthorized absences.
On the charge that Cloma had terrorized the staff of the Outright
Division and incited a work stoppage, it is clear, from the May 17, 2000

suspension order, that he has already been penalized with


suspension for this offense.
84.
Perez v Philippine Telegraph and Telephone Co, GR no. 152048
(2008)
GR No. 152048 / 7 Apr 2009 / J. Corona
Facts:

Petitioners - Felix Perez was a shipping clerk while Amante Doria was a
supervisor in respondent Philippine Telephone & Telegraph's shipping section.

PT&T acted on an unsigned letter mentioning anomalous transactions at


the shipping section and formed a special audit team to investigate. It
learned that the shipping section jacked up freight costs for goods shipped
and that duplicates of shipping documents showed traces of altering and
tampering.

On 3 Sep 1993, petitioners were placed on 30-day preventive suspension for


their alleged involvement. Their suspension was extended for 15 days
twice.

On 29 Oct, PT&T issued a memo that in line with the recommendation of


the AVP-Audit and subsequent criminal charges filed against petitioners,
they are hereby dismissed from service.

On 9 Nov they filed a complaint for illegal suspension and dismissal, alleging that they were dismissed on 8 Nov, the date they received the memo.

LA's ruling: the extensions of the suspension and the subsequent dismissal were illegal.

NLRC: 15 day extension of suspension illegal, but dismissal was legal for
just cause.

CA: affirmed NLRC, but said dismissal, though for just cause, had no due
process.

Issue:

W/N there was just cause in dismissal of petitioners. NO

W/N there was due process in dismissal of petitioners. NO

W/N they were illegally suspended for 30 days (not 15). YES

Held:
Just cause
62

There was no just cause. Evidence was insufficient to attribute the anomalies to petitioners.

PT&T should have provided evidence of petitioners' functions, extent of


duties, procedures in handling shipping requests, and the fact that no other
personnel was involved.

Impartial tribunals should not rely only on the statement of the employer
that it lost confidence in its employee.

The standard for the hearing requirement, ample opportunity, is


couched in general language revealing legislative intent to give some
degree of flexibility. Also, Sec. 2(d), Rule I of the omnibus rules provides that the standards of due process outlined therein shall be observed substantially, not strictly. This is a recognition that while a
formal hearing or conference is ideal, it is not an absolute,
mandatory or exclusive avenue of due process.

Instead of a formal hearing, employee may submit a written explanation upon receipt of first notice and offer evidence in support, like
relevant company records and sworn statements of witnesses. He
may prepare this personally or with assistance from a representative
or counsel. He may ask employer to provide him copies of records
material to his defense.

When formal hearing becomes mandatory:

Willful breach of trust by employee is a just cause for termination. But it


should not be used as a subterfuge for causes which are improper, illegal or
unjustified. Burden of proof rests on employer to prove cause, and the evidence must clearly and convincingly show the facts on which loss of
confidence rests.

Due process

To meet the requirements of due process, employer must furnish two written notices:
1.

Written notice specifying grounds for termination and giving to said


employee a reasonable opportunity to explain his side

2.

Another written notice indicating that, upon due consideration of all


the circumstances, grounds have been established to justify employer's decision to dismiss the employee.

None of that here. Petitioners were simply separated from work and served
notices of termination.

But as to petitioners' contention that there was no due process due


to absence of a hearing:

There is no need for a hearing or conference for due process.

Omnibus rules implementing LC require a hearing and conference


during which the employee concerned is given the opportunity to respond to the charge, present his evidence or rebut the evidence presented against him.

But LC Art. 277(b) only requires that an employee given "ample


opportunity to be heard and to defend himself."

Test for fair procedure under Art. 277(b) cannot be whether there
has been a formal pretermination confrontation between employer
and employee. The "ample opportunity to be heard" standard is
neither synonymous nor similar to a formal hearing.

Ample opportunity to be heard means any meaningful opportunity


(verbal or written) given to the employee to answer the charges
against him and submit evidence in support of his defense, whether
in a hearing, conference or some other fair, just and reasonable way.

If in his written explanation he requests a formal hearing

Where there exists substantial evidentiary disputes

Where company rules or practice require actual hearing as


part of employment pretermination procedure

Illegal suspension

Valid suspension for just cause may only be for 30 days, after which the
employee shall either be reinstated or paid his wages during the extended
period.

There was no evidence that petitioners were paid during the two 15-day extensions of their preventive suspension. Thus, PT&T must pay their wages
for this period.

Dispositive:

Original LA decision reinstated, modified that since reinstatement is impossible as 14 years have passed since the dismissal, they are to be paid
separation pay.

85.

Technol Eight Phil. Corp. v NLRC GR No. 187605 (2010)

Technol Eight Philippines Corporation v. NLRC and Dennis Amular


(digest by Robby Solis)
(Brion, J.; GR 187605; 13 April 2010)
Facts:

Technol Eight Philippines Corporation, located at 127 East Main Avenue,


Laguna Technopark, Bian, Laguna, manufactures metal parts and motor
vehicle components. It hired the Dennis Amular in March 1998 and assigned
him to Technols Shearing Line, together with Clarence P. Ducay. Rafael
Mendoza was the lines team leader.
63

April 16, 2002 at about 5:30 p.m., Mendoza went to the Surf City Internet
Caf at a mall in Balibago, Sta. Rosa, Laguna. As Mendoza was leaving the
establishment, he was confronted by Amular and Ducay who engaged him in
a heated argument regarding their work in the shearing line, particularly Mendozas report to Avelino S. De Leon, Jr., Technols Production Control and Delivery (PCD) assistant supervisor, about Amulars and Ducays
questionable behavior at work. The heated argument resulted in a fistfight
that required the intervention of the barangay tanods in the area.

While the mauling incident happened outside the company premises and after working hours, no disruption of work operations and it did not result in a
hostile environment in the company, nevertheless, there is work-connection
because the motivation behind the confrontation concerned respondents
questionable work behavior. The motivation behind the confrontation was
rooted on workplace dynamics as Mendoza, Amular and Ducay interacted
with one another in the performance of their duties.

On Misconduct, Respondent and his companion were purposely at the mall to


confront Mendoza about their work-related problem, when the latter appeared, they accosted and mauled him respondent committed misconduct or
exhibited improper behavior that constituted a valid cause for his dismissal
under the law and jurisprudential standards.

The essence of due process is simply an opportunity to be heard. It is the denial of this opportunity that constitutes violation of due process of law. Amular chose not to present his side by filing a complaint before the date set for
his administrative hearing. The essence of due process is simply an opportunity to be heard; it is the denial of this opportunity that constitutes violation
of due process of law, not refusal to avail it.

Upon learning of the incident, Technols management sent to Amular and


Ducay a notice of preventive suspension/notice of discharge dated May 18,
2002 advising them that their fistfight with Mendoza violated Section 1-k of
Technols Human Resource Department (HRD) Manual. The two were given
forty-eight (48) hours to explain why no disciplinary action should be taken
against them for the incident. They were placed under preventive suspension
for thirty (30) days, from May 19, 2002 to June 17, 2002 for Ducay, and May
21, 2002 to June 20, 2002 for Amular. Amular submitted a written statement
on May 20, 2002.

Dispositive:

Technol Management said that they will conduct an administrative hearing


on the matter. Amular failed to attend the administrative hearing. Consequently, Technol sent him a Notice of Dismissal.

Petition MERITORIOUS, GRANTED; the assailed decision of CA REVERSED


and SET ASIDE; the complaint for illegal dismissal is DISMISSED for lack of
merit. Costs against respondent AMULAR.

LA: Amulars preventive suspension and subsequent dismissal were illegal;


was based solely on unsubscribed written statements by Mendoza, Rogelio
Garces and Mary Ann Palma. Technol did not afford him procedural due
process

86.

Seastar Marine Services Inc v Bui-an 444 SCRA 140 (2004)

87.

ABO Overseas v NLRC 286 SCRA 454 (1998)

88.

Macasero v Southern Industrial Gases Phil, GR No. 178524 (2009)

NLRC: Affirmed LA decision

CA: Affirmed decision; Technol failed to comply with jurisprudential guidelines that misconduct warranting a dismissal: (1) must be serious; (2) must
relate to the performance of the employees duties; (3) must know that the
employee has become unfit to continue working for the employer; mauling incident happened outside office premises and outside working hours.

PANFILO
MACASERO,
versus
SOUTHERN
GASES PHILIPPINES and/or NEIL LINDSAY

FACTS:

Issues:
1. WON there was illegal dismissal NO, he was validly dismissed
2. WON Amular was not afforded procedural due process NO, he was given a
chance to explain

Held/Ratio:

SC finds no procedural impediment to the petition. An objective reading of


the petition reveals that Technol largely assails the correctness of the conclusions drawn by the CA from the set of facts it considered. The question therefore is one of law and not of fact, as we ruled in Cucueco v. Court of Appeals.

INDUSTRIAL

The services of Panfilo Macasero were engaged by Southern Industrial


Gases, Philippines as Carbon Dioxide Bulk Tank Escort since September
1995. For every 24-hour work rendered by him in escorting respondent
companys tanks while they were being shipped from Cebu and to other
areas in the Visayas and Mindanao, petitioner earned P200, aside from
receiving transportation, accommodation, and meal allowances.
On January 5, 1999, petitioner Macasero filed before the NLRC a
Complaint against respondent company and/or its co-respondent General
Manager Neil Lindsay, for illegal dismissal with prayer for
reinstatement, backwages, unpaid benefits, and attorneys fees, alleging
that in September 1998, he was advised that his services were no longer
needed and was in fact prevented from entering the company premises.
Respondents contended that no employer-employee relationship existed
between respondent company and petitioner because his services were
only occasionally required, he having worked 287 days in the 3 years that
he was connected with it; that petitioner was never subject to respondent
64

companys supervision and/or control; and that petitioner had no fixed


work schedule, hence, at most, he was an unsupervised pakiaw or task
worker.
Respondents, on appeal, insisted that no employer-employee
relationship existed between respondent company and petitioner who it
claimed was actually an independent contractor or, at best, a task
worker.

ISSUE/S: whether petitioner was dismissed and, if in the affirmative, if it was


legally effected.
HELD: petitioner was, dismissed without just cause.
In illegal dismissal cases, the onus of proving that the employee was
not dismissed or, if dismissed, that the dismissal was not illegal, rests on
the employer, failure to discharge which would mean that the dismissal
is not justified and, therefore, illegal.
Indeed, a party alleging a critical fact must support his allegation with
substantial evidence, for any decision based on unsubstantiated allegation cannot
stand without offending due process.
Respondents claim that there was a business slump, hence, petitioner
could not be given any escorting assignment has remained just that. The records
are bereft of any documentary evidence showing that it was indeed suffering
losses or a decline in orders which justified its admitted failure to give
assignments to petitioner.
89.
Philippine Rural Reconstruction Movement v Pulgar GR No.
169227 (2010)
90.

Kulas Ideas & Creations v Alcoseba GR No. 180123 (2010)

KULAS IDEAS & CREATIONS, GIL FRANCIS MANINGO AND MA. RACHEL
MANINGO, Petitioners, vs. JULIET ALCOSEBA AND FLORDELINDA ARAOARAO, Respondents.
FACTS
In 1996, respondents Juliet Alcoseba (Juliet) and Flordelinda Arao-arao
(Flordelinda) were employed as sales attendants of herein petitioner KULAS
Ideas & Creations (KULAS).
As part of their duties and responsibilities, Juliet and Flordelinda were tasked to
sell KULASs products, prepare weekly sales reports and assist the clerk in the
monthly inventory of saleable goods.
In February 2000, the Department of Labor and Employment (DOLE) inspected
the outlet of KULAS in Ayala Center in Cebu where Juliet and Flordelinda were
assigned and found that it violated several labor standards laws. The DOLE later
sent KULAS a Notice of Summary Investigation dated September 11, 2000

directing it to pay the salary differential of its employees from January to August
2000 amounting to P173,003.28.
KULAS subsequently directed Juliet and Flordelinda, by Memorandum of
November 23, 2000, to explain and/or investigate an alleged inventory
discrepancy which entailed the amount of P48,179.30. And it thereafter
suspended Juliet and Flordelinda for seven days, by Memorandum of November
29, 2000, starting December 1, 2000 for gross negligence of duties and
responsibilities.
Both Juliet and Flordelinda thus filed a complaint for illegal suspension and
withholding of salaries before the National Labor Relations Commission (NLRC)
Regional Arbitration Branch No. VII on December 5, 2000.
By Reconciliation Report of December 7, 20007 sent to Juliet and Flordelinda,
KULAS advised them that discrepancies in its inventory were noted and that
both of [them] were assigned at the Ayala Boutique to diligently monitor all
stocks and to report any stock discrepancy to the office, if there were any, so that
the proper action may be taken, [but] [t]here never was any report made
regarding stock shortage. KULAS accordingly directed them to explain the
discrepancies.
After serving their suspension, Juliet and Flordelinda, by letter of December 11,
2000, inquired with KULAS the status of their employment since they were told
not to report for work until they were able to explain the discrepancies.
After another investigation finding a shortage of items, respondents were sent a
memo giving them 48 hours to explain why they should not be terminated from
employment.
ISSUE: WON respondents were illegally dismissed.
HELD: YES. KULAS failed to provide substantial evidence of the allegations
against them. For a termination based on gross and habitual neglect to prosper,
there must be a showing that the character of the negligence must not be only
gross but also habitual. There was also a lack of due process, as respondents did
not receive due notice regarding investigation of the charges, and that they were
not fully apprised thereof.
Article 282 (b) and (c) 29 of the Labor Code provide that an employer may
terminate an employee for "gross and habitual neglect by the employee of his
duties" and for "fraud." In both instances, substantial evidence is necessary for an
employer to effectuate any dismissal. Uncorroborated assertions and accusations
by the employer do not suffice, otherwise the constitutional guaranty of security
of tenure of the employee30 would be jeopardized.
Article 282 (b) imposes a stringent condition before an employer may terminate
an employment due to gross and habitual neglect by the employee of his duties.
To sustain a termination of employment based on this provision of law, the
negligence must not only be gross but also habitual.
91.

Abosta Shlpmanagement Corp. v NLRC GR No. 163252 (2011)

ABOSTA SHIP MGT CO v. NLRC and FLORES


65

J. Brion
G.R. No. 163252
TOPIC: Procedure; Although Flores merely issued a vehement denial, Capt. B.H.
Mun should have given him a reasonable time to explain, if necessary, in writing.
While this lapse in procedure cannot negate the existence of a valid cause
for Flores dismissal, as discussed above, the violation of his right to procedural
due process warrants the payment of indemnity in the form of nominal damages,
as we held in Agabon v. National Labor Relations Commission.
FACTS: Arnulfo Torres entered into a 12-month contract with Abosta (local
agency), in behalf of Panstar (foreign principal), as a radio officer and joined the
vessel M/V Morning Charm in 1997, with B.H. Mun as Captain and Gowang Gun
Lee as Chief Engineer (from SoKor), as well as other Filipino employees. He was
repatriated due to alleged infractions committed while on board the vessel.
FLORES: He was asked to coordinate with several crew members who wanted to
be re-assigned or terminated due to mismanagement of the vessel. He acted as
coordinator, but was surprised when he was one of those whose resignations were
accepted. Abosta said Panstar held Flores liable for instigating the resignation en
masse of the PH crew.
Abosta and Panstar: Flores, while in their employ, insistently and rudely
questioned the crews working schedule, including the propriety of requiring them
to render overtime services. They claimed that Flores instigated the crew to rebel
against the authority of the Master, under the guise of questioning social security
and income tax deductions. The crew members became unruly, arrogant, and
impolite, and were even violent in expressing their views. They even refused to
obey the lawful orders of the Master and the senior officers, thus causing
dissension on board the vessel.
LA: Dismissed Flores complaint for lack of merit; evidence was convincing
enough to prove Flores was a threat to the safety of the vessel and its crew; Flores
failed to refute agencys accusations that he incited the crew to rebel
NLRC: Panstar failed to prove his dismissal was due to a just/authorized cause
and that petitioners observed due process; reports from the chief mate and 1st
assistant engineer did not constitute proof of the validity of the dismissal
CA: Dismissed due to insufficiency of substance; failure to observe due process
and to prove dismissal was due to a just/authorized cause
ISSUE: Whether or not Abosta and Panstar observed procedural due process in
terminating Flores (NO)
RESOLUTION: CA decision SET ASIDE; DISMISSAL VALID but AWARD him
ND for violation of his due process rights
Procedural due process; Not observed in this case; The records bear out
that Flores was not given a reasonable opportunity to present his side vis--vis
the charges at the time he was dismissed. As the NLRC noted, Floreswas
immediately dismissed after Capt. B.H. Mun conducted his inquiry on November

17, 1997. Although Flores merely issued a vehement denial, Capt. B.H. Mun
should have given him a reasonable time to explain, if necessary, in writing.
While this lapse in procedure cannot negate the existence of a valid cause
for Flores dismissal, as discussed above, the violation of his right to procedural
due process warrants the payment of indemnity in the form of nominal damages,
as we held in Agabon v. National Labor Relations Commission. Given the
circumstances in the present case, we deem an award of nominal damages
to Flores in the amount of P30,000.00 to be appropriate.
92.

Jarl Construction and Armando K. Tejada vs. Simeon A. Atencio.

93.

Concepcion v Minex Import Corp GR No. 153569 (2012)

CONCEPCION vs MINEX IMPORT CORP.


Minex Import-Export Corp. (Minex), a retail business of semi-precious stones,
hired petitioner as its sales girl, rotated in its kiosk outlets in various shopping
centers in Manila. Petitioner reported the theft of P50,912 worth of sales proceeds
to Mimex's Assistant Manager Mariano, explaining that she had placed the
amount in a plastic bag at the bottom of the kiosk's cabinet the night before, but
that it was missing the following morning. Mariano then had petitioner placed
under arrest and filed a complaint of qualified theft against her.
Petitioner was released on the instruction of the inquest prosecutor, and she
subsequently filed a case for illegal dismissal against Minex.
ISSUES:
1. w/n there was just cause for Concepcion's dismissal
2. w/n Concepcion was afforded due process
HELD:
1. YES. The quantum of proof for dismissing an employee is substantial evidence.
The employer need not prove her guilt beyond reasonable doubt (as in a criminal
proceeding), it being enough that there was justified loss of confidence. In this
case, Concepcion was negligent as a supervisor in handling the large amount by
simply leaving it in the cabinet drawer. At the very least, she could have resorted
to the SOP of asking the main office how to secure the amount since she couldn't
deposit in a bank on a sunday.
2. NO. Concepcion was immediately arrested and investigated for qualified theft
after she reported the loss. Even before her release, MINEX's decision to
terminate her had already become final, as relayed to her over the phone by her
supervisor. For this, the court awarded her P30,000 in nominal damages.
**Prescription (inferred, not expressly stated in the case): Concepcion brought the
action for illegal dismissal shortly after she was arrested and dismissed, thus it
fell within the 4 year period prescribed by CC 1146 for (1) Upon an injury to
the rights of the plaintiff and (2) Upon a quasi-delict.
94.

Talam v NLRC GR No. 175040 (2010)


FRANCIS RAY TALAM vs.
RELATIONS
COMMISSION,

NATIONAL LABOR
THE
SOFTWARE
66

FACTORY, INC. and/or TERESA GRAPILON, Office


Manager, and WOLFGANG HERMLE, Chief Executive
Officer
FACTS: The respondent, The Software Factory, Inc. (TSFI), is a domestic
corporation engaged in providing information technology and computer
consultancy to the public. It holds office in Makati City. In April 2001, it
employed Talam as a full-time programmer.
In the latter part of 2001 and in 2002, TSFI suffered financial reverses. Its
external financial auditor advised that it cut on its payroll expenses which
accounted for 41% of its total operating costs. TSFI heeded the advice and decided
to retrench some of its employees, using as basis its employees' service income and
contribution margins to the company. TSFI found that Talam was one of two
employees with the least or with no income contribution for the year 2002.
Consequently, respondents Teresa Grapilon (Grapilon), TSFI's Office Manager,
and Wolfgang Hermle (Hermle), Chief Executive Officer, verbally informed Talam
that his services with the company would be terminated thirty (30) days after
September 27, 2002. Thereafter, TSFI notified Talam in writing of the
termination of his employment. The notice was dated October 1, 2002, but
received by Talam on October 4, 2002. On November 6, 2002, or after a month,
Talam signed a Release and Quitclaim in consideration and receipt of P89,954.00
in compensation and other benefits.
ISSUE: Whether or not the execution of quitclaim is a bar to the filing of the
complaint for illegal dismissal
HELD: YES. IT BARS FILING OF THE COMPLAINT
RATIO:
The CA erred in glossing over the legal effect of Talam's release and quitclaim. It
should not have been nullified. Talam was not an unlettered employee; he was an
information technology consultant and must have been fully aware of the
consequences of what he was entering into. The quitclaim was a voluntary act as
there is no showing that he was coerced into executing the instrument; he
received a valuable consideration for his less than two years of service with the
company. Thus, from all indications, the release and quitclaim was a valid and
binding undertaking that should have been recognized by the labor authorities
and the CA.
While the law looks with disfavor upon releases and quitclaims by employees who
are inveigled or pressured into signing them by unscrupulous employers seeking
to evade their legal responsibilities, a legitimate waiver representing a voluntary
settlement of a laborer's claims should be respected by the courts as the law
between the parties. Talam's release and quitclaim fall into the category of
legitimate waivers as defined by the Court. ECDaAc
In executing the release and quitclaim, Talam had unequivocably signified his
acceptance of his separation from the service as communicated to him in writing
by TSFI on October 1, 2002, after the company management verbally discussed
the matter with him. In fact, on the day he received the written notice of his
separation (October 4, 2002), he was issued, upon his request, a certification that

he "is a former employee of The Software Factory Inc." who joined the
company "on April 15, 2001 until October 31, 2002 as a Programmer."
With the foregoing backdrop in Talam's execution of the release and quitclaim, we
find the filing of the illegal dismissal case tainted with bad faith on his part for he
has already "released and forever discharged" the company "from any and all
claims of damages and other liability, any from any and all manner of claims,
cause or causes of actions whatsoever . . . against them."
Given the release and quitclaim, we do not see how TSFI can be made to answer
for failure to afford Talam procedural due process. The release and quitclaim, to
our mind, erased whatever infirmities there might have been in the notice of
termination as Talam had already voluntarily accepted his dismissal through the
release and quitclaim. With this acceptance, the written notice became academic;
the notice, after all, is merely a protective measure put in place by law and serves
no useful purpose after protection has been assured. We thus find no basis for the
conclusion that TSFI violated procedural due process and should pay nominal
damages.
95.

Aujero v Phil. Satellite Corp. GR No. 193484 (2012)

HYPTE R. AUJERO v. PHILIPPINE COMMUNICATIONS SATELLITE


CORPORATION (2012)
REYES, J:
FACTS: It was in 1967 that the petitioner started working for respondent
Philippine Communications Satellite Corporation (Philcomsat) as an accountant
in the latter's Finance Department. On August 15, 2001 or after thirty-four (34)
years of service, the petitioner applied for early retirement. His application for
retirement was approved, effective September 15, 2001, entitling him to receive
retirement benefits at a rate equivalent to one and a half of his monthly salary for
every year of service. At that time, the petitioner was Philcomsat's Senior VicePresident with a monthly salary of Two Hundred Seventy-Four Thousand Eight
Hundred Five Pesos (P274,805.00).
On September 12, 2001, the petitioner executed a Deed of Release and Quitclaim
in Philcomsats favor, following his receipt from the latter of a check in the
amount of Nine Million Four Hundred Thirty-Nine Thousand Three Hundred
Twenty-Seven and 91/100 Pesos (P9,439,327.91).
Almost three (3) years thereafter, the petitioner filed a complaint for unpaid
retirement benefits, claiming that the actual amount of his retirement pay is
Fourteen Million Fifteen Thousand and Fifty-Five Pesos (P14,015,055.00) and the
P9,439,327.91 he received from Philcomsat as supposed settlement for all his
claims is unconscionable, which is more than enough reason to declare his
quitclaim as null and void. According to the petitioner, he had no choice but to
accept a lesser amount as he was in dire need thereof and was all set to return to
his hometown and he signed the quitclaim despite the considerable deficiency as
no single centavo would be released to him if he did not execute a release and
waiver in Philcomsat's favor.

67

ISSUE: WON the quitclaim executed by the petitioner in Philcomsats favor is


valid, thereby foreclosing his right to institute any claim against Philcomsat.
HELD: The quitclaim is VALID.
RATIO: Philcomsat alleged that the petitioner's execution of the subject
quitclaim was voluntary and he made no claim that he did so. Philcomsat likewise
argued that the petitioner's educational attainment and the position he occupied
in Philcomsat's hierarchy militate against his claim that he was pressured or
coerced into signing the quitclaim. In Goodrich Manufacturing Corporation, v.
Ativo, the Court reiterated the standards that must be observed in determining
whether a waiver and quitclaim has been validly executed:
Not all waivers and quitclaims are invalid as against public policy. If the
agreement was voluntarily entered into and represents a reasonable settlement,
it is binding on the parties and may not later be disowned simply because of a
change of mind. It is only where there is clear proof that the waiver was wangled
from an unsuspecting or gullible person, or the terms of settlement are
unconscionable on its face, that the law will step in to annul the questionable
transaction. But where it is shown that the person making the waiver did so
voluntarily, with full understanding of what he was doing, and the consideration
for the quitclaim is credible and reasonable, the transaction must be recognized
as a valid and binding undertaking.
While the law looks with disfavor upon releases and quitclaims by employees who
are inveigled or pressured into signing them by unscrupulous employers seeking
to evade their legal responsibilities, a legitimate waiver representing a voluntary
settlement of a laborer's claims should be respected by the courts as the law
between the parties. Considering the petitioner's claim of fraud and bad faith
against Philcomsat to be unsubstantiated, this Court finds the quitclaim in
dispute to be legitimate waiver.
While the petitioner bewailed as having been coerced or pressured into signing
the release and waiver, his failure to present evidence renders his allegation selfserving and inutile to invalidate the same. That no portion of his retirement pay
will be released to him or his urgent need for funds does not constitute the
pressure or coercion contemplated by law.
That the petitioner was all set to return to his hometown and was in dire need of
money would likewise not qualify as undue pressure sufficient to invalidate the
quitclaim. "Dire necessity" may be an acceptable ground to annul quitclaims if the
consideration is unconscionably low and the employee was tricked into accepting
it, but is not an acceptable ground for annulling the release when it is not shown
that the employee has been forced to execute it. While it is the courts duty to
prevent the exploitation of employees, it also behooves it to protect the sanctity of
contracts that do not contravene our laws.
The petitioner is not an ordinary laborer. He is mature, intelligent and educated
with a college degree, which cannot be easily duped or tricked into performing an
act against his will. As no proof was presented that the said quitclaim was
entered into through fraud, deception, misrepresentation, the same is valid and
binding. The petitioner is estopped from questioning the said quitclaim and
cannot renege after accepting the benefits thereunder. This Court will never

satisfy itself with surmises, conjectures or speculations for the purpose of giving
imprimatur to the petitioner's attempt to abdicate from his obligations under a
valid and binding release and waiver.
The petitioner's educational background and employment stature render it
improbable that he was pressured, intimidated or inveigled into signing the
subject quitclaim. The Court cannot permit the petitioner to relieve himself from
the consequences of his act, when his knowledge and understanding thereof is
expected. Also, the period of time that the petitioner allowed to lapse before filing
a complaint to recover the supposed deficiency in his retirement pay clouds his
motives, leading to the reasonable conclusion that his claim of being aggrieved is
a mere afterthought, if not a mere pretention.
96.
Philippine Carpet Manufacturing Corporation, et al. v. Ignacio B.
Tagyamon
97.

Jobel Enterprises v NLRC GR No. 194031 (2011)

Jobel Enterprises v. NLRC and Martinez (2011)


Jobel Enterprises hired Martinez as a driver. Initially he performed well, but
later became stubborn and got into a fight with a co-worker. He was told to go
home and he never reported for work again. He was dismissed. Martinez then
demanded money and filed an illegal dismissal case when the company did not
pay him.
LA: Ruled for Martinez. Awarded
NLRC: Appeal was denied because company failed to post a surety bond (amount
which the NLRC increased, then denied motion to reduce bond amount).
CA: Dismissed petition for certiorari because of petitioners failure to attach to
the petition a duplicate original or certified true copy of the assailed NLRC
decision.
SC: REVERSED. We note that this case was dismissed on purely technical
grounds at both the NLRC and the CA levels, in total disregard of the merits of
the case. The case was not unmeritorious on its face and the lower courts should
have at least heard the case to establish the facts. The CA precipitately denied
the petition for certiorari based on an overly rigid application of the rules of
procedure. In effect, it sacrificed substance to form in a situation where the
petitioners recourse was not patently frivolous or meritless.
98.

Lacorte v Inciong 166 SCRA 1 (1988)

Termination of Employment>Technical ground for dismissal frowned upon (not


sure why this case is under this topic :/)
Lacorte v Inciong
September 27, 1988 | FERNAN, C.J.:
FACTS:

68

Petitioner Lacorte is a warehouseman in private respondent corporation,


Asean Fabricators, Inc (Asean), whose duties included receiving and storing raw and junk materials used by respondent in its business
One day, he offered to purchase some obsolete defective and non-usable
junk materials from Asean. The respondent agreed but when petitioner
tried to bring out these items, it was discovered that the items weighed
more than what he actually purchased. Furthermore, it was found out
that the items were not junk since some parts were brand new and usable
Asean filed a case for qualified theft against petitioner, however, complaint was dismissed for insufficiency of evidence. Nevertheless, application for clearance to terminate the services of petitioner filed by Asean
was granted
On appeal, order was affirmed by public respondent Inciong. Hence, this
petition.

ISSUE/HELD: W/N public respondents acted arbitrarily and/or with grave abuse
of discretion in connection with the grant of the application for clearance to
terminate the employment of petitioner. NO
There is substantial basis for the Orders issued by respondent labor
officials.
Petitioner: The criminal complaint for qualified theft based on the alleged
attempted stealing of some company property was dismissed by the fiscal for
insufficiency of evidence
SC: The purpose of the proceedings before the fiscal is to determine if there is
sufficient evidence to warrant the prosecution and conviction of the accused. In
assessing the evidence before him, the fiscal considers the basic rule that to
successfully convict the accused the evidence must be beyond reasonable doubt
and not merely substantial. On the other hand, to support findings and conclusion
of administrative bodies only substantial evidence is required. It does not follow
that once the fiscal dismisses the complaint for qualified theft, respondent
officials should also have decided in favor of petitioner. For one, the evidence
presented before the two bodies may not be necessarily Identical. Secondly, the
appreciation of the facts and evidence presented is an exercise of discretion on the
part of administrative officials over which one cannot impose his conclusion on
the otherthe conviction of an employee in a criminal case is not indispensable to
warrant his dismissal, and the fact that a criminal complaint against the
employee has been dropped by the fiscal is not binding and conclusive
upon a labor tribunal.
Petitioner: It took private respondent several months after the incident before
seeking clearance to terminate his services during which he was allowed to
remain in his job, negating any claim of loss of confidence arising
therefrom
SC: Respondent company conducted its investigation on the alleged theft before
filing the criminal charges and the application for clearance, and only after
having been convinced of the veracity of the reported attempt to steal. That the
company investigated the incident first while allowing petitioner to stay

on his job pending the investigation is not only proper but in accord
with fair process. That the investigation took time is understandable,
considering that it was not the only preoccupation of respondent corporation
Petitioner: Order failed to consider the affidavits of petitioner and a fellow
employee claiming that the real motive for dismissing him is not loss of
confidence but his union activities; and, that petitioner was allegedly not
aware of the weighing and examination of the withheld boxes containing
the scrap materials conducted by respondent corporation after he was accosted by
the company guards, in violation of his right to due process
SC: It defies explanation other than that it was a mere afterthought why it took
petitioner so much time to prepare affidavits (presented when the case was
already on appeal) which contain nothing more than the bare allegation,
obviously self-serving, that his union activities prompted his termination.
Petitioner's imputation of omissions must therefore fail. He has not succeeded in
overcoming the presumption of regularity in the performance of respondent labor
officials' functions in issuing the orders.
As to petitioners claim that there was no actual weighing and examination of the
boxes containing the scrap materials, SC said that it is now too late in the day for
petitioner to raise these matters of facts.. At any rate, his evidence does not
substantiate his claim.
99.

Quiambao v NLRC 254 SCRA 211 (1996)

QUIAMBAO vs.NLRC
Rodolfo Quiambao was hired as officer-in-charge of private respondent Central
Cements Tuguegarao Branch. Six months later, he was made permanent Branch
Manager. Petitioner, together with the Branch Cashier was in charge of credit
collections. He submitted monthly reports on the operations of the branch and the
outstanding balances of its customers and was also required to attend regular
monthly meetings. A financial and performance audit showed the Tuguegarao
Branch to be in a state of disarray and chaos. Petitioner was suspended for an
indefinite period for poor performance in extending credit to customers, violation
of company rules and regulations and gross negligence. Petitioner was charged
with estafa, while a civil case for collection was brought against him. Both cases
were dismissed. Petitioner demanded reinstatement with backwages. But Central
Cement ignored his demand and instead served him with a notice of termination
on the ground of loss of confidence. Petitioner therefore filed a complaint for
illegal dismissal. Labor Arbiter found petitioner to have been illegally dismissed.
Central Cement appealed to the NLRC. Petitioner moved to dismiss appeal on the
ground that Cement Central had not posted a supersedeas bond as required by
Art. 223 of Labor Code, but the NLRC did not act on his motion.
Quiambao filed this petition for certiorari. He filed a supplemental petition,
alleging that the NLRC acted without jurisdiction and contrary to law in taking
cognizance of the appeal of Central Cement from the decision of the Labor Arbiter
despite the fact that Central Cement had not posted a supersedeas bond.
SC: The petition is well taken.
69

Filing of a supersedeas bond is indispensable to the perfection of an appeal in


cases which involve monetary awards. Because Central Cement failed to comply
with this requirement, the decision of the Labor Arbiter, finding Central Cement
guilty of the illegal dismissal of petitioner, became final and executory.
The intention of the lawmakers to make the bond an indispensable requisite for
the perfection of an appeal is clearly limned in the provision that the appeal by
the employer may be perfected only upon the posting of a cash or surety bond.
It is true that, in some cases, this Court relaxed the requirement of posting
supersedeas bond for the perfection of an appeal. But the decisions in those cases
were justified by the fact that there was substantial compliance with the rule, so
that on balance, technical considerations had to give way to considerations of
equity and justice. In the case at bar, no similar justifications exist excusing
Central Cements failure to comply with the rule on mandatory posting of
supersedeas bond.

favor of respondent and authorizing the NLRC to release the same in the
event that the Labor Arbiters Decision becomes final and executory.
The NLRC reversed the LA ruling and on the issue of the posting of cash
or security bonds, the NLRC held that there was substantial compliance
with the appeal bond requirement.
Initially, the CA dismissed Confesors appeal but upon MR, the CA held
that the ruling of the LA became final and executory and reinstated the
LAs decision. The CA reinstated the LA ruling because the bank deposit
of MTC failed to substantially comply with the appeal bond requirement.

Issue: Whether or not there was substantial compliance with the appeal bond
requirement. NO.
An appeal from a judgment as that involved in the present case is perfected only
upon the posting of a cash or surety bond.

The consequence of private respondents failure to comply with the mandatory


requirement for the perfection of the appeal was to render the decision of the
Labor Arbiter final and executory, and to place it beyond the power of the NLRC
to review and, even more so, to reverse.

Article 223of the Labor Code provides that an appeal by the employer to the
NLRC from a judgment of a labor arbiter which involves a monetary award may
be perfected only upon the posting of a cash or surety bond issued by a reputable
bonding company duly accredited by the NLRC, in an amount equivalent to the
monetary award in the judgment appealed from.

100.

Cruz v Coca Cola Bottlers Phlls. Inc. 460 SCRA 340 (2005)

Section 4 of the New Rules of Procedure of the NLRC echoes the provision:

101.

Barairo v OP & MST Marine GR No. 189314 (2011)

102.

Mindanao Times Corp v Confesor, GR No. 183 417 (2010)

Mindanao Times Corporation vs. Mitchel Confesor


Carpio-Morales, J.
Facts:

Mitchel Confesor was the Associate Editor of Mindanao Times


Corporation (MTC), a publisher of a newspaper of general circulation in
Mindanao and Davao City.
Confesor alleges that he was forced to resign by the President and COO
of MTC when he wrote articles accusing then Presidential Assistant
Dominador Boy Zuo, Jr., Cong. Prospero Nograles and Cong. Corazon
Malanyaon of being involved in some anomalies.
He was told that if he resigned, he would be entitled to separation pay
and other benefits. When the promised benefits were not forthcoming, he
filed a complaint for payment of separation pay and pro-rated 13th month
pay which he later amended from one of money claim to illegal dismissal.
The LA found for Confesor and held that he was constructively
dismissed. Both parties appealed to the NLRC. Confesor contending that
he was entitled to moral and exemplary damages, and MTC questioning
the constructive dismissal.
In compliance with the appeal bond requirement, MTC deposited the
amount of P71,909.77 with the United Coconut Planters Bank and
surrendered to the NLRC the passbook covering the deposit, along with a
Deed of Assignment it executed assigning the proceeds of the deposit in

SECTION 4. REQUISITES FOR PERFECTION OF APPEAL. The


appeal shall be filed within the reglementary period as provided in
Section 1 of this Rule; shall be verified by appellant himself in
accordance with Section 4, Rule 7 of the Rules of Court, with proof of
payment of the required appeal fee and the posting of a cash or surety
bond as provided in Section 6 of this Rule;
SECTION 6. BOND. In case the decision of the Labor Arbiter or the
Regional Director involves a monetary award, an appeal by the employer
may be perfected only upon the posting of a cash or surety bond. The
appeal bond shall either be in cash or surety in an amount equivalent to
the monetary award, exclusive of damages and attorneys fees.
As previously stated in Accessories Specialist, Inc. v. Alabanza,
The posting of a bond is indispensable to the perfection of an appeal in
cases involving monetary awards from the decision of the LA. The word
"only" makes it perfectly plain that the lawmakers intended the posting
of a cash or surety bond by the employer to be the essential and exclusive
means by which an employer's appeal may be perfected. The filing of the
bond is not only mandatory but also a jurisdictional requirement that
must be complied with in order to confer jurisdiction upon the NLRC.
Non-compliance therewith renders the decision of the LA final and
executory.
Cash, means a sum of money. In the present case, the Deed of Assignment, as
well as the passbook, which petitioner submitted to the NLRC is neither a cash
nor a surety bond. Petitioners appeal to the NLRC was thus not duly perfected,
thereby rendering the Labor Arbiters Decision final and executory.
70

103.

University Plans Inc v Solano GR No. 170416 (2011)

the bond may be reduced albeit only on meritorious grounds and upon
posting of a partial bond in a reasonable amount in relation to the
monetary award. Suffice it to state that while said Rules allows the
Commission to reduce the amount of the bond, the exercise of the
authority is not a matter of right on the part of the movant, but lies
within the sound discretion of the NLRC upon a showing of meritorious
grounds

[T]he bond requirement on appeals involving monetary awards has been


and may be relaxed in meritorious cases. These cases include instances
in which

University Plans Inc. v Solano


FACTS:

Respondents filed before the Labor Arbiter complaints for illegal


dismissal, illegal deductions, overriding commissions, unfair labor
practice, moral and exemplary damages, and actual damages against
petitioner University Plans Incorporated.
LA rendered in favour of the respondents. Thus, Petitioner filed before
the NLRC its Memorandum on Appeal[6] as well as a Motion to Reduce
Bond.[7] Simultaneous with the filing of said pleadings, it posted a cash
bond in the amount of P30,000.00.

Petitioner alleged that it was under receivership and that it cannot


dispose of its assets at such a short notice.

In an Order[8] dated April 25, 2001, the NLRC denied petitioners


Motion to Reduce Bond and directed it to post an additional appeal bond
in the amount of P3,013,599.50

the CA held that the NLRC in meritorious cases and upon motion by the
appellant may reduce the amount of the bond. However, in order for the
NLRC to exercise this discretion, it is imperative for the petitioner to
show veritable proof that it is entitled to the same.

Petitioner stresses that it is under receivership pursuant to Presidential


Decree No. 902-A. As such, all pending actions for claims are
automatically stayed to enable the management committee or the
rehabilitation receiver to effectively exercise its powers free from any
judicial or extrajudicial interference.

ISSUE: WON Reduction of Bond is allowed?


HELD/RATIO: Yes

Article 223. Appeal. Decisions, awards, or orders of the Labor Arbiter


are final and executory unless appealed to the Commission by any or both
parties within ten (10) calendar days from receipt of such decisions,
awards, or orders. x x x

In case of a judgment involving a monetary award, an appeal by the


employer may be perfected only upon the posting of a cash or surety bond
issued by a reputable bonding company duly accredited by the
Commission in the amount equivalent to the monetary award in the
judgment appealed from.

Sections 4 and 6, Rule VI of the Revised Rules of Procedure of the NLRC


read: ii) posting of a cash or surety bond as provided in Section 6 of this
Rule
o

(1) there was substantial compliance with the Rules,

(2) surrounding facts and circumstances constitute meritorious


grounds to reduce the bond,

(3) a liberal interpretation of the requirement of an appeal bond


would serve the desired objective of resolving controversies on
the merits, or
(4) the appellants, at the very least, exhibited their willingness
and/or good faith by posting a partial bond during the
reglementary period.

It is clear that the petitioner is under receivership. However, the


NLRC ignored petitioners allegations and instead remained
adamant that since the amount of bond is fixed by law, petitioner
must post an additional bond of more than P3 million. This, to us,
is an utter disregard of the provision of the Labor Code and of the
NLRC Revised Rules of Procedure allowing the reduction of bond
in meritorious cases. While the NLRC tried to correct this error in
its March 21, 2003 Resolution[32] by further explaining that it
was not persuaded by petitioners alleged incapability of posting
the required amount of bond for failure to submit financial
statement, list of sources of income and other details with respect
to the alleged receivership, we still find the hasty denial of the
motion to reduce bond not proper.

No motion to reduce bond shall be entertained except on


meritorious grounds, and only upon the posting of a bond in a
reasonable amount in relation to the monetary award
71

You might also like